SlideShare uma empresa Scribd logo
1 de 99
Chapter I 
Review topics in 
Algebra 1 
1
2 
1.Set of real numbers 
 The word algebra originated from the Arabic word “al-jabr” which means 
the science of reduction and cancellation. The algebraic symbolism used 
to generalize the operatiob of arithmetic was formulate in the sixteenth 
and seventeenth centuries. 
Real number 
 set of rational numbers and the set of irrational numbers make up. It 
consists of the set of real numbers and two operations called addition and 
multiplication. Addiotion is denoted by the symbol “+” and multiplication 
is denoted by the symbol “x” or “”. If a nd b are real numbers, a+b 
denotes the sum of a and b, and ab or (ab) denotes their products. 
If the numbers are repeating or terminating decimal they called rational number. The 
square roots of perfect squares also name rational number. 
Examples: 
1) √0.16 
2) 0.666 
3) 
1 
3 
4) 
10 
9 
5) 
9 
6
If the numbers are not repeating or terminating decimals. They called irrational number. 
3 
For examples: 
1) π 
2) √2 
3) 0.61351 
4) √8 
5) √11 
Properties of real numbers 
Let us denote the set of real numbers by 푅. These properties are statement derived from 
the basic axioms of the real numbers system. Axioms are assumptions on operation with 
numbers. 
Axioms of Equality 
Let a, b, c, d ∈ R 
1. Reflexive Law 
If a=a 
2. Symmetric Law 
If b=c then c=b 
3. Transitive Law 
If b=c and c=d then b=d 
4. Additional Law of Equality 
If a=b then a+c=b+c 
5. Multiplication Law of Equality 
If a=b then a.c=b.c 
Axioms for Addition and Multiplication 
Let a, b, c, d, ∈ R 
1) A. Closure property for addition 
a+b ∈ R 
Examples: 
1) 3+3=6 
2) 7+(-4)=3 
3) -8+4=-4 
B. Closure property for multiplication 
a.b ∈ R 
Examples: 
1) 3(7)=21 
2) -8(3)=-24 
3) 0.11=0
2) A. Commutative prroperty for addition 
4 
a+b=b+a 
Examples: 
1) 
1 
2 
+ 7 = 7 + 1 
2 
2) 0.3 + (− 5 
6 
) = − 5 
6 
+ 0.3 
3) 
1 
3 
+ 21 = 21 + 1 
3 
B. Commutative prroperty for multiplication 
a.b=b.a 
Examples: 
1) 
4 
5 
(22) = 22 (4 
5 
) 
2) 6.3=3.6 
3) 
10 
9 
(−25) = −25 (10 
9 
) 
3) A. Associative property for addition 
(a+b)+c=a+(b+c) 
Examples: 
1) (3+7)+0.4=3+(7+0.4) 
2) (0.36+89)+ 
1 
2 
= 0.36 + (89 + 1 
2 
) 
3) (3 
5 
+ 0.8) + 3 
8 
= 3 
5 
+ (0.8 + 3 
8 
) 
B. Associative property for multiplication 
(a.b).c=a.(b.c) 
Examples: 
1) (3.x).y=3.(x.y) 
2) [5(7)] 1 
4 
= 5 [7 (1 
4 
)] 
3) [3푥(6푥)]]5 = 3푥[6푥(5)] 
4) Identity property for multiplication 
a.1=a 
Examples: 
1) 1.a3=a3 
2) 
3 
7 
(1) = 3 
7 
3) 3.1=3 
5) A. Inverse property for addition 
a+(-a)=0 
Examples: 
1) 6+(-6)=0 
2) 10+(-10)=0 
3) -3+3=0
B. Inverse property for multiplication 
5 
푎. 1 
푎 
= 1 
Examples: 
1) -2(− 1 
)=1 
2 
1 
8 
2) 8( 
)=1 
3) -6(- 
1 
6 
)=1 
6) Distributive property of multiplication over addition 
a(b+c)=ab+ac 
Examples: 
1) 3(4+6)=3(4)+3(6) 
2) -6(7+1)=-6(7)+[-6(1)] 
3) a(7+5)=7a+5 
1.2 Exponents and Radicals 
In the expression 훼푛 , α is the base and 혯 is the exponent. The expression 훼푛 
means that the value α is multiplied 혯 times by itself. 
Examples: 
1) 63= 6.6.6 
=216 
2) 56= 5.5.5.5.5 
=15625 
3) 42= 4.4 
=16 
Integral and zero exponents 
Laws of Integral and Zero Exponents 
Theorem 1: 
For any real number α, (α≠ 0) 
푎0 = 1 
Examples: 
1) (6푎0 + 3)0=1 
2) 6α0+70=6(1)+1=7 
3) 2α0+70=2(1)+1=3 
Theorem 2: 
For any real numbers α, 
αm. α혯= αm+n 
where m and n are integers. 
Examples: 
1) α5.α4=푎5+4 = 푎9 
2) 4푥푦2(2푥2푦2) = 8푥 1+2푦2+2 = 8푥 3푦4 
3) 푥 푎+3. 푥 푎+4 = 푥 2푎 +7
6 
Theorem 3: 
For any real numbers a+b, 
(ab)n=anbn, 
where n is any integer. 
Examples: 
1) (5x)2=55x2=25x2 
2) (-2x)3=-23x3=-8x3 
3) [x(x-3)]2=x2(x-3)2 
=x2(x2-6x+9) 
=x4-6x3+9x2 
Theorem 4: 
For any real numbers a 
(am)n=amn 
where m and n are integers. 
Examples: 
1) (-x2)3=-x2(3)=-x6 
2) [(3x+4)2]3=(3x+4)6 
3) (-x2y3z)4=-x8y12z4 
Theorem 5: 
For any real numbers a and b (b≠0), 
( 
푎 
푏 
)푛 = 
푎푛 
푏푛 
where n is any integer. 
Examples: 
1) ( 
푎2 
푏3 )2 = 푎4 
푏6 
3 
4 
2) ( 
)3 = 33 
43 = 27 
64 
푥 
푦 +2 
3) ( 
)2= 
푥2 
2 
= 푥(푦 +2)2 푦2 +4푦 +4 
Theorem 6: 
For any real numbers a(a≠0), 
푎푚 
푎푛 = 푎푚−푛 
where m and n are integers. 
Examples: 
1) 
푎7 
푎5 = 푎7−5=푎2 
2) 
푥3푦4 푧5 
푥푦푧 
= 푥 3−1푦4−1 푧5−1 = 푥 2푦3푧4 
3) 
푥4푦4 
푥4푦4 = 푥 4−4푦4− 4 = 푥 0푦0 = 1(1) = 1
7 
Theorem 7: 
For any real numbers a(a≠0), 
푎−푛 = 
1 
푎푛 
Where n is any positive integer. 
Examples: 
1) 3푥 3푦−2= 
3푥3 
푦2 
2) (4푥 2푦)−2 = 1 
(4푥2푦)2 = 1 
8푥4 푦2 
3) (푥 2 + 푦)−2 = 1 
(푥2+푦)2 = 1 
푥4+푦2 
Fractional Exponents: Radicals 
Since not all numbers are integers, we can’t expect exponents to always 
whole number or zero. Exponents can be form fractional. Fractional exponents 
may seem unfamilliar for they are usually expressed as radicals. 
For expression 푥 
1 
2 is the same as √2 (read as square root of 2), and 푥 
2 
3 is 
the same as 3√푥2 
(read as cube root of x squared). The expression 푛√푎푚 is called a 
radical. The symbol √ is called a radical sign, where n is the index, a is the 
radicand and m is the power of the radicand. 
푎 
푚 
푛 
=푛√푎푚 
Laws of Radicals 
Theorem 1: 
For any real numbers a, 
√푎푛 = 푎 푛 
Examples: 
1) √42 = 4 
2) 3√(푥2푦)3=푥 2푦 
3 
3) √33 =3 
Theorem 2: 
For any real numbers a,and b. 
√푎 푛 . √푏 푛 = √푎푏 푛 
Examples: 
1) √3. √3 = √3.3 = √9=3 
2) √4. √3 = √4.3 = √12 
3) √푎. √푏 = √푎. 푏
8 
Theorem 3: 
For any real numbers a,and b, (b≠0) 
√푎 푛 
√푏 푛 = √ 
푎 
푏 
푛 
Examples: 
1) 3√푎 
√푏 3 = √ 
푎 
푏 
3 
2) √4 
√5 
= √4 
5 
3) 4√푥 
√푦 4 = √ 
푥 
푦 
4 
Theorem 4: 
For any real numbers a , 
푚 
√푎 푚푛 = √ √푎 푛 
푛 
= √ √푎 푚 
Examples: 
3 
1) √64 6 = √√64 
= √8 3 = 2 
2 
2) √16 4 = √√16 
= √4 2 =2 
3) √100 3 = √100 3 =√100 = 10 
Theorem 5: 
For any real numbers a 
k √푎푘푚 푛 = 푛√푎푚 
Examples: 
1) √24 6 
= √22.2 2.3 = 3√22 = √4 3 
2) √93 6 = 3.2√93.1 = √9 2 =3 
3) 
Addition and Sutraction of Radicals 
To add and subtract radicals, first we need to combine the like terms with 
similar radicals. 
Examples: 
1) √2 + 3√2 − 2√2 = 2√2 
2) √8 + √18 + √32 = √4.2 + √9.2+√16.2 = 2√2 + 3√2 + 4√2 = 9√2 
3) 푦√푥 3푦 − √푥 3푦3 + 푥√푥푦3 = 푦√푥 2. 푥푦 − √푥 2. 푥. 푦2. 푦 + 푥√푥. 푦2. 푦 = 
푥푦√푥푦 − 푥푦√푥푦 + 푥푦√푥푦 = 푥푦√푥푦
Multiplication and Division of Radicals 
To multiply and divide radicals with the same index, multiply, or divide 
the radicals and copy the common index. 
Examples: 
1) √3.√3 = √32 = 3 
2) 3√푥푦. √푥 2푦 3 . √푥푧 3 =√푥푦. 푥 2푦. 푥푧 3 = √푥 4푦2푧 3 = 푥 √푥푦2푧 3 
3) √16 3 ÷ √−2 3 =√16 ÷ (−2 3 )= √−8 3 = −2 
9 
1.3 Polynomials 
Polynomials was used to describe any algebraic expression. The algebraic 
expression, 5x+4 and x3+x2+1 are examples of polynomials in variable x. A polynomial 
with just one term 2x is called a monomial. If the polynomial is the sum or difference of 
two terms as in -9x+7, then it is called a binomial. If it has three terms like x2+2x+1, then 
it is called a trinomial. In general a polynomial consisting of a sum of any numbers of 
terms is called a multinomial. 
In the binomial, 5x+4 the number 5 is called the numerical coefficient of x while x 
is the literal coefficient and the numbers 4 is the constant term. 
Addition and Sutraction of Polynomials 
To determined the sums and differences of polynomials, only the coefficients are 
combined. By similar terms are refer to the terms with the same coefficients. Those with 
different literal coefficient are called dissimilar or unlike terms. 
Examples: 
1. Find the sum of 2x-3y+5 and x+2y-1, 
=(2x-3y+5)+( x+2y-1) 
=2x+x-3y+2y+5-1 
=3x-y++4 
2. Find the differences between 2x-3y+5 and x+2y-1 
=(2x-3y+5)-( x+2y-1) 
=2x-3y+5+(-x-2y+1) 
=2x-x-3y-2y+5+1 
=x-5y+6 
3. Subtract 2(4x+2y+3) from 5(2x-3y+1) 
=5(2x-3y+1)- 2(4x+2y+3) 
=10x-15y+5-8x+4y+6 
=2x-11y+11
10 
Multiplication of Polynomials 
Examples: 
1) 푥 푚.푥 푛 = 푥 푚+푛 
2) 푥 −2.푥 2=푥 0 = 1 
3) Multiply a+2b+3c by 5m. 
= a+2b+3c(5m) in multiplication, we apply the 
=5am+10bm+15cm distributive property 
Division of Polynomials 
To divide a polynomial by a monomial, divide each term of the polynomial by the 
monomial. 
푥 푚 
푥 푛 = 푥 푚−푛 푎푛푑 푥 −푛 = 
1 
푥 푛 
Examples: 
1) 
푥5 
푥2 = 푥 3 
1 
푥5 
2) 푥 −5= 
3) Divide 7푥 2 − 5푥 푏푦 푥 
푥 is the divisor and 7푥 2 − 5푥 as the dividend, we have 
7푥2 −5푥 
푥 
= 
7푥2 
푥 
- 
5푥 
푥 
=7푥 − 5 
1.4 Factoring 
Factors and Greatest Common Denominator 
If the two of more numbers are multiplied, each number is a factor of the product. 
In the example above, 18 is expressed as the product of different pair of whole numbers. 
18=2.9 
18=3.6 
18=18.1 
A prime number is a whole number, greater than1, whose only factors are 1 and 
itself. A composite number is a whole number greater than 1, that is not prime. 
Examples: 
1) Find the prime factorization of 84. 
84=2.42 the least prime factor of 84 is 2 
=2.2.24 the least prime factor of 42 is 2 
=2.2.3.7 the least prime factor of 21 is 3 
All of the factors in tha last row are prime. Thus, the prime factorization of 84 is 
2.2.3.7 or 22.3.7. 
2) Factor 20a2b 
20a2b=2.10.a.a.b 
=2.2.5.a.a.b
The greatest common factor of two or more integer is the product of the prime 
11 
factors common to the integers. 
Examples: 
1) Find the GCF of 54, 63, and 180. 
54=2.③.③.3 factor each number 
63③.③7 
180=2.2.③.③.5 then circle the common factors 
The GCF of 54, 63, and 180 is 3.3 or 9. 
2) 8푎2푏 푎푛푑 18푎2푏2푐 
8푎2푏=②. 2.2. 푎 ⃝ . 푎 ⃝ . 푏 ⃝ 
18푎2푏2 푐=②.3.3. 푎 ⃝ . 푎 ⃝ . 푏 ⃝. 푏. 푐 
= 2푎2푏 
The GCF of 8푎2 푏 푎푛푑 18푎2푏2푐 is 2푎2푏. 
Factoring Using the Distributive Property 
To multiplied a polynomial by a monomial by using the distributive property. 
Multiplying Polynomials Factoring Polynomials 
3(a+b)=3a+3b 3a+3b=3(a+b) 
x(y-z)=xy-xz xy-xz= x(y-z) 
3y(4x+2)=3y(4x)+3y(2) 12xy+6y=3y(4x)+3y(2) 
=12xy+6y =3y(4x+2) 
Examples: 
1) Use the distributive property to factor 10푦2 + 15푦 
10푦2=2.⑤. 푦 ⃝. 푦 
15푦 =3.⑤.푦 
The GCF is 5y 
10푦2 + 15푦=5y(2y)+5y(3) 
=5y(2y+3) distributive property 
2) Factor 21푎푏2 − 33푎2푏푐 
21푎푏2=③.7. a ⃝ . 푏 ⃝. 푏 
33푎2푏푐=③.11. a ⃝ . 푎. 푏 .⃝c the GCF is 3ab 
21푎푏2 − 33푎2푏푐= 3ab(7b)-3ab(11ac) 
=3ab(7b-11ac) distributive property 
Factoring by Grouping 
Polynomial with four or more terms, like 3xy-21y+5x-35, can sometimes be 
factored by grouping terms of the polynomials. One method is to group the terms into 
binomials that can each be factored using the distributive property. Then use the the 
distributive property again with a binomial as the common factor.
12 
Examples: 
1) Factor 3xy-21y+5x-35 
3xy-21y+5x-35= (3xy-21y)+(5x-35) 
=3y(x-7)+5(x-7) 
=3y+5(x-7) 
Check by using FOIL ; 
(3y+5)(x-7)=3y(x)+3y(-7)+5(x)-5(7) 
=3xy-21y+5x-35 
2) Factor 8푚2 푛 − 5푚 − 24푚푛 + 15 
8푚2 푛 − 5푚 − 24푚푛 + 15=(8푚2푛 − 5푚) + (−24푚푛 + 15 
= 푚(8푚푛 − 5) + (−3)(8푚푛 − 5) 
=푚 − 3(8푚푛 − 5) 
Check: 
푚 − 3(8푚푛 − 5) = 푚(8푚푛) + 푚(−5) + (−3)(8푚푛) + (−3)(-5) 
= 8푚2 푛 − 5푚 − 24푚푛 + 15 
Factoring Trinomials 
When two numbers are multiplied each number is a factor of the product. 
Similarly if two binomials are multiplied, each binomials is factor of the product. 
Consider the binomials 5x+2 and 3x+7. You can use the FOIL method to find their 
product. 
(5x+2)( 3x+7)=(5x)(3x)+(5x)(7)+(2)(3x)+(2)(7) 
=15x2+35x+6x+14 
=15x2+(35+6)x+14 
=15x2+41x+14 
You can be use this pattern to factor quadratic trinomials, such as 2푦2 + 7푦 + 6 
Factors of 12 Sum of Factors 
1.12 1+12=13 no 
2.6 2+6=8 no 
3.4 3+4=7 yes 
2푦2 + (3 + 4)푦 + 6 Select the factors 3 and 4. 
2푦2 + 3푦 + 4푦 + 6 
(2푦2 + 3푦) + (4푦 + 6) Group terms that have a 
푦(2푦 + 3) + 2(2푦 + 3) common monomials factor 
(푦 + 2)(2푦 + 3) Factor (use the distirbutive property) 
Therefore 2푦2 + 7푦 + 6= (푦 + 2)(2푦 + 3)
13 
Example: 
Factor 5푥 − 6 + 푥 2 
The trinomials 5푥 − 6 + 푥 2 can be written as 푥 2 + 5푥 − 6. For this 
trinomials, the constant terms is -6 and the coefficient of x is 5. Thus, we need ti 
find two factors two factors of -6 whose sum is 5. 
Factors of -6 Sum of factors 
1, -6 1+(-6)=-5 no 
-1, 6 -1+6=5 yes 
Select the factors -1 and 6 
Therefore, 푥 2 + 5푥 − 6 = (푥 − 1)(푥 + 6) 
Factoring Differences of Square 
The product of the sum and ifference of two expressions is called the differences 
of squares. The process for finding this product can be reversed in order to factor the 
differenceof squres. Factoring the difference of square can also be modeled 
geometrically. 
푎2 − 푏2 = (푎 − 푏)(푎 + 푏) 
Examples: 
1) 푓푎푐푡표푟 푎2 − 64 
푎2 − 64 = (푎)2 − (8)2 
= (푎 − 8)(푎 + 8) 
푎. 푎 = 푎2 푎푛푑 8.8 = 64 use the difference of square 
2) 푓푎푐푡표푟 푎푥 2 − 100푦2 
푎푥 2 − 100푦2 = (3푥)2 − 10푦2 
= (3푥 − 10푦)(3푥 + 10푦) 
3푥. 3푥 = 9푥 2 푎푛푑 10푦. 10푦 = 100푦2 
Perfect Square and Factoring 
Numbers such as 1,4,9 and 16 are called perfect squares. Since they can be expressed as 
the square of an integer. Products of the form (푎 + 푏)2 푎푛푑 (푎 − 푏)2 are called perfect 
squares and the expansions of these products are called perfect square trinomials. 
(푎 + 푏)2 = 푎2 + 2푎푏 + 푏2 
(푎 − 푏)2 = 푎2 − 2푎푏 + 푏2 
Finding a Product Factoring 
(푦 + 8)2 = 푦2 + 2(푦)(8) + 82 푦2 + 16푦 + 64 = (푦)2 + 2(푦)(8) + 
(8)2 
= 푦2 + 16푦 + 64 = (푦 + 8)2
Examples: 
Determine whether 16푎2 + 81 − 72푎 is a perfect square trinomial. 
1) 16푎2 + 81 − 72푎 = 16푎2 − 72푎 + 81 
= (4푎)2 − 2(4푎)(푎) + (푎)2 
= (4푎 − 9)2 
2) 푥 2 + 22푥 + 121 = (푥)2 + 2(푥)(11) + (11)2 
14 
= (푥 + 11)2 
Solving Equations by Factoring Zero Product Property 
For all numbers a and b, if ab=0, then a=0, b=0 or both a and b equal 0. 
Example: 
1) Solve 16t(9-t)=0 
16t(9-t)=0, then 16t=0 or 9-t zero product property 
16t=0 or 9-t=0 solve each equation 
t=0 9=t 
check: Substitute 0and 9 for t in the original. 
16t(9-t)=0 
16(0)(9-0)=0 or 16(9)(9-9)=0 
0(9)=0 144(0)=0 
0=0 0=0 
SOLUTION SET: (0,9) 
2) (y+2)(3y+5)=0 
If (y+2)(3y+5)=0, then y+2=0 or 3y+5=0 
y+2=0 or 3y+5=0 
y=-2 3y=-5 
푦 = − 5 
3 
Check: (y+2)(3y+5)=0 
(-2+2)[(3)(-2)+5]=0 or (− 5 
3 
+ 2) [(3) (− 5 
3 
) + 5] = 0 
0(-1)=0 
1 
3 
(0) = 0 
0=0 0=0 
SOLUTION SET: (-2, − 5 
3 
) 
1.5 Rational Expressions 
A fraction where the numerator and denominator are polynomials, and is defined 
for all values of the variable that do not make the denominator zero. 
Reducing Rational Expression to Lowest Terms 
We need to lowest term the fraction, if the numerator and denominator have no 
common factor.
푥2−2푥푦+4푦2 
3푥(푥+푦)−2푦 (푥+푦)+2푥2 −푦2 
15 
Examples: 
1) 
4푎2 푏푐3 
6푎푏3 푐4 = 2.2.푎.푎 .푏.푐.푐.푐 
2.3.푎.푏.푏.푏.푐.푐.푐 .푐 
= 2푎 
3푏2푐 
2) 
푥2+2푥푦 +푦2 
푥2 −푦2 = (푥+푦)(푥+푦 ) 
= 
(푥+푦)−(푥−푦 ) 
푥+ 푦 
푥− 푦 
3) 
푥3+8푦 3 
4푥+8푦 
= 푥+2푦 (푥2−2푥푦 +4푦2 
4(푥+2푦 ) 
= 
4 
Multiplying and Dividing Rational Expressions 
In multiplication if 
푝 
푞 
푎푛푑 푟 
푠 
are rational expressions and q and s are real numbers 
not equal to 0, then 
푝 
푞 
. 푟 
푠 
= 푝푟 
푞푠 
. 
Examples: 
1) 
4 
3 
. 1 
5 
= 4 
15 
2) 
푐 
. (푎 + 2푏)(푎 − 푏) 
푎2−푏2 푐 
= 
(푎 + 푏)(푎 − 푏) 
. (푎 + 2푏)(푎 − 푏) 
푐(푎+2푏) 
= 
푎+푏 
In dividing algebraic fractions, multiply the dividend by the reciprocal of the divisor. The 
reciprocal of a fraction is its multiplicative inverse. 
Examples: 
1) 
4 
3 
÷ 
6 
5 
= 4 
3 
. 5 
6 
= 20 
18 
표푟 10 
9 
2) 
8 
7 
÷ 3 = 8 
7 
. 1 
2 
= 8 
14 
표푟 4 
7 
3) 
푦2 −16 
푦 −5 
÷ 2푦 −8 
푥푦−5푥 
(푦−4)(푦+4) 
= 
푦 −5 
. 푥(푦−5) 
2(푦−4) 
= 푥푦 +4푥 
2 
Adding and Subtracting Rational Expressions. 
To add and subtract rational expressions, it is the important that the least common 
denominator is accurately determined. 
Examples: 
1) 
5 
6 
− 2 
3 
+ 1 
8 
= 20−16+3 
24 
= 7 
24 
2) 
4 
5 
+ 3 
5 
+ 2 
5 
= 4+3+2 
5 
= 9 
5 
3) 3푥 − 2푦 + 2푥2 −푦2 
푥 +푦 
= 
푥+푦 
= 
3푥2 +3푥푦 −2푥푦 +2푦2 +2푥2 −푦2 
푥 +푦 
= 
5푥2 +푥푦− 3푦2 
푥+푦
Simplifying Complex Rational Expressions 
A factor which contains one or more fractions either in the numerator or 
The 푛푡ℎ root of a real number 
If n is a positive integer greater than 
1 푎푛푑 푎 푎푛푑 푏 are real number such that 
푏푛 = 푎, then b is an 푛푡ℎ root of a. 
The principal 푛푡ℎ root of a real number. If n is a 
positive integer greater than 1, a is a real number, 
and √푎 푛 denotes the princial 푛푡ℎ root of a, then 
If a>0, √푎 푛 is the positive 푛푡ℎ root of a. 
If a<0, and n is odd, √푎 푛 is the negative 푛푡ℎ 
root of a. 
√0 푛 = 0 
16 
denominator or in both. 
Examples: 
1) 
4 
31 
3 
= 4 
3 
. 3 
1 
= 12 
3 
표푟 4 
2) 
3 
2+ 
1 
3 
= 3 
6+1 
3 
= 3 
7 
3 
= 3. 3 
7 
= 9 
7 
1.6 Rational Exponents 
We defined 푎푛 if n is any integer (positive, negative or zero). To define a 
power of a where the exponent is any rational number, not specifically an integer. 
1 
That is, we wish to attach a meaning to 푎 
⁄푛 푎푛푑 푎 
푚 
⁄푛, where the exponents are 
fractions. Before discussing fractional exponents, we give the following 
definition. 
Definition 
Examples 1: 
1) 2 is a square root of 4 because 22 = 4 
2) 3 is a fourth root of 81 because 34 = 81 
3) 4 is a cube root of 64 because 43 = 64 
Definition 
. 
The symbol √ is called a radical sign. The entire expression √푎 푛 is called a 
radical, where the number a is the radicand and the number n is the index that indicates 
the order of the radical.
17 
Examples 2: 
1) √4 = 2 
2) √81 4 = 3 
3) √64 3 = 4 
Definition 
Examples 3: 
1) 25 
1 
⁄2 = √25 = 5 
2) −8 
1 
⁄3= √−8 3 = −2 
3) ( 1 
81 
)1/4=√ 1 
81 
4 
= 1 
3 
Definition 
Examples 4: 
1) 9 
3 
⁄2=(√9)3=33=27 
2) 8 
2 
⁄3 = (√8 3 )2=22 = 4 
3) −27 
4 
⁄3 = (√−27 3 )4=(-3)4=81 
It can be shown that the commutative law holds for rational 
exponents, and therefore 
(푎푚)1/n=(푎 
1 
⁄푛)m 
From which it follows that 푛√푎푚 = ( √푎 푛 )m 
The next theorem follows from this equality and the definition of 푎 
푚 
⁄푛 
If n is a positive integer greater than 1, and a is 
a real number, then if √푎 푛 is a real number 
푎 
1 
⁄푛 = √푎 푛 
If m and n are positive integers that are 
relatively prime, and a is a real number, 
then if √푎 푛 is a real number 
푎 
푚 
⁄푛 = (√푎 푛 )m ⇔ 푎 
푚 
⁄푛 = (푎 
1 
⁄푛)m
If m and n are positive integrers that are 
relatively prime, and a is a real number, 
then if √푎 푛 is a real number 
푚 
⁄푛 = (푎푚)1/n 
18 
Theorem 1 
Examples 5: 
푚 
⁄푛 = 푛√푚 Theorem 1 푎is applied ⇔ 푎 
in the following: 
1) 9 
3 
⁄2=√93=729 =27 
2) 8 
2 
⁄3 = √8 3 2=√64 3 = 4 
3) −27 
4 
⁄3 = (√−27 3 )4=√531441 3 =81 
푎 
Observe that 푎 
푚 
⁄푛 can be evaluated by finding either ( √푎 푛 )m or 푛√푎푚. Compare 
example 4 and 5 and you will see the computation of ( √푎 푛 )m in example 4 is simpler than 
that for 푛√푎푚 in example 5. 
The laws of positive-integer exponents are satisfied by positive-rational exponents 
with one exception: For certain values of p and q, (ap)q≠apq for a<0. This situation arises 
in the following example. 
Examples 6: 
1) [(-9)2]1/2=811/2=9 and (-9)2(1/2)=(-9)1=-9 
Therefore [(-9)2]1/2≠(-9)2(1/2). 
2) [(-9)2]1/4=811/4=3 and (-9)2(1/4)=(-9)1/2 (not a real number) 
Therefore [(-9)2]1/4≠(-9)2(1/4). 
The problems that arise in example 6 are avoided by adopting the following rule: If m 
and n are positive even integers and a is a real number, then (푎푚 )1/n=│a│m/n 
A particular case of this equality occurs when m=n. We then have (푎푛)1/n=│a│ 
(if n is a positive even integer) or, equivalently, 푛√푎푛 = │a│ (if n is even) 
If n is 2, we have √푎2 = │a│ 
Examples 7: 
1) [(-9)2]1/2=│-9│=9 
2) [(-9)2]1/4=│-9│2/4=91/2=3
Definition 
If m and n positiv e integer that are 
relatively prime and a is a real number and 
1 
⁄3)2=(1 
19 
Example:8 
1) 8 
−2 
⁄3 = 1 
8 
2 
⁄3= 
1 
( 3√8)2 = 
1 
22= 
1 
4 
2) 8 
−2 
⁄3 = (8 
−1 
⁄3)2=( 1 
8 
2 
1 
4 
)2= 
3) 
1 
푥 
⁄3 
푥 
1 
⁄4 =푥 
1 
⁄3. 
1 
푥 
1 
⁄4=푥 
1 
⁄3.푥 
−1 
⁄4 = 푥(1 
⁄3)−1 
⁄4 = 푥 
1 
⁄12 
a≠0, then if √푎 푛 is a real number. 
푎 
−푚 
⁄푛 = 
1 
푎 
푚 
⁄푛
ASSESSMENT 
20 
TEST I 
Name the property that justifies each step. 
1. Simple 6a+(8b+2a) 
a. 6a+(8b+2a)=6a+(2a+8b) 
b. =(6a+2a)+8b 
c. =(6+2)a+8b 
d. =8a+8b 
2. Simplify 6푎2 + (6푎 + 푎2 ) + 9푎 
a. 6푎2 + (6푎 + 푎2 ) + 9푎 = 6푎2 + (푎2 + 6푎)+9a 
b. = (6푎2 + 푎2 ) + (6푎+9a) 
c. = (6푎2 + 1푎2) + (6푎+9a) 
d. =6 + 1(푎2) + (6 + 9)푎 
e. = 7푎2 + 15푎 
Simplify and express the following. 
1. 
푎푚+5 
푎푚−2 
2. [ 
(푥+푦)0+푎0+ 푏0 
푎+푏+푐 
]-2 
3. (푎−2 + 푦)-2 
4. (37푥+5 )(34푥−4) 
5. (9푥푦2)(4푥 3푦) 
Rational Expression (simplify) 
1. 
9푛 
63푛 
÷ 9푛 
2. 
−15푚3 푛2 푝2 
−35푚2 푛5 푝 
3. 
푥+푦 
푥2−푦2 
4. 
3푚− 1 
9(푚−1)2−4 
5. 
4푚푛+6 
10푚 +8푛 
6. 
2푥2 +3푥 −5 
10푥 +25 
7. 
푥2−5푥 −24 
4푥2 −27푥 −40 
8. 
25푎2 +70푎 +49 
25푎 2 −49 
Factor each polynomial into two binomials 
1. a2+ 12a+ 27 
2. y2+ 21y+ 110 
3. n2-4n+ 4 
4.x2-12x + 20 
5. x2+ 11x -12
Answer the following word problemsand multiple choice questions 
1. The area of a rectangle is (x2–12x + 35). If the length is (x-5),find the 
width.(hint:“x5” times“something” will give you “x2–12x + 35.”). 
2. The area of a rectangle is 3a2+ 5a–28. If the length is (a+ 4), find the width. 
3. A rectangle has an area of 3x2+ 5x –12. What factors are the length and width of 
21 
the rectangle? 
a. (3x + 4)(x –3) 
b. (3x –4)(x + 3) 
c. (3x + 3)(x –4) 
d. (3x –3)(x + 4) 
4. The area of a certain rectangle is 5n2–6n–27. Which factors are the width and 
length of the rectangle? 
a. (5n + 3)(n –9) 
b. (5n –3)(n + 9) 
c. (5n + 9)(n –3) 
d. (5n –9)(n + 3) 
5. If the area of a certain rectangle is 6m2–2a –28, and the length is (2m + 4), what is 
the width? 
a. (3m + 7) 
b. (4m –7) 
c. (4m + 7) 
d. (3m –7) 
TEST II 
A. Determine whether each statement is true or false. 
1. Every integer is also a real number. 
2. Every irrational number is also an irrational number. 
3. Every natural number is also a whole number. 
4. Every real number is also a rational number. 
B. State whether each decimal represents a rational o irrational number. 
5. √4 
6. √5 
7. 0 
8. 3 
9. 0.63586358 
10. √866 
C.Determine which real number property is shown by each of the following 
1. − 1 
4 
+ 1 
4 
= 0 
2. 2(1)=2 
3. 
1 
4 
(4)=1 
4. -7+(-4)=-4+(-7) 
5. 0.3(0)=0.3
22 
6. 5[3+(-1)]=5(3-1) 
7. (8+ 
9 
8 
9 
8 
)+0.45=8+( 
+0.45) 
8. 5(8+8)=5(8)+5(8) 
9. 6x+(8x+10)=(6x+8x)+10 
10. 5a+2b=2b+5a 
D.Simplify and express the following expressions with positive and negative integrals 
only. 
1. 50 
2. 
10푚4 
30m 
3. 
16푏4 푐 
−4푏푐3 
4. 푦3. 푦4 
5. (5푥푦)6 
6. (푎푏)3 
7. (푥 3푦2)3 
8. [(−5)2]2 
9. 
푥5푦6 
푥푦 
= 
10. 
푎7 
푎3 
E.Simplify the following radicals 
1.(5√2)(3√6) 
2.(3푎3√4푥2)(4 √3푥푦 3 ) 
3.4√ 9 
16 
4.√2(3+√3) 
5.5√2+3√2 
6.√18 − 2√27 + 3√3 − 6√8 
7.√16푏 + √4푏 
8. 
−12√24 
3√2 
9.√8 + √50 
10. 4√푥 7푦10 
F.Simplify the following polynomials. 
1. (5푥 − 1) + (10푥 2 + 7푥) 
2.(20푥 2 + 2) + (15푥2 − 8) + (3푥 2 − 4) 
3.(푥 2 + 푦2 + 8) + (4푥 2 − 2푦2 − 9) 
4.(−3푥 2 + 5푦 − 4푥푦 + 푦2) 푓푟표푚(2푥2 − 4푦 + 7푥푦 − 6푦2) 
5.2푥 2 + 6푥 + 5 푎푛푑 3푥2 − 2푥 − 1
23 
6.(푥 + 2)(푥2 − 2푥 + 3) 
7.푎푏(2푎 + 1) 
푥2−3푥 −10 
8. 
푥+ 2 
푥6+2푥4 +6푥−9 
9. 
푥3+3 
10. (3푥 3 − 11푥 2푦 + 11푥푦2 − 2푦3) ÷ (푥 − 2푦) 
G.Find the factors and its GCF 
1. 21 
2. 4, 12 
3. 63 
4. 304 
5. 18, 35 
6. 12푎푛2, 40푎4 
7. 7푦2, 14푦2 
8. 15, 10 
9. 6푎2, 18푏2, 9푏3 
10. 18푥 2푦2, 6푦2, 42푥 2푦3 
H.Factor each trinomials. If possible. If the trinomial cannot be factored using integers. 
Write prime. 
1. 6푚푥 − 4푚 + 3푥 − 2푟 
2. 3푚푦 − 푎푏 + 푎푚 − 30푦 
3. 푎2 − 2푎푏 + 푎 − 20 
4. 3푚2 − 5푚2 푝 + 3푝2 − 5푝3 
5. 4푎푥 − 14푏푥 + 35푏푦 − 10푎푦 
6. 푦2 + 12푦 + 27 
7. 푐2 + 2푐 − 3 
8. 푥 2 − 5푥 − 24 
9. 7푎2 + 22푎 + 3 
10. 6푦2 − 11푦 + 4 
I.Factor out each polynomials 
1. 푥 2 − 49 
2. 푥 2 − 36푦2 
3. 16푎2 − 9푏2 
4. 2푎2 − 25 
5. 2푧2 − 98 
6. 푛2 − 8푛 + 16 
7. 4푘2 − 4푘 + 1 
8. 푥 2 + 6푥 − 9 
9. 1 − 10푧 + 25푧2 
10. 50푥 240푥 + 8
24 
J. Solve and simplify each polynomials 
1. 
푎+1 
푎3 − 푎 +2 
푎2 + 푎 +3 
푎 
2. 
5푥3 
7푦4 . 21푦 2 
10푥2 
3. 
9푥5 
36푥2 
4. 
5−푎 
푎2− 25 
5. 
10푎2 −29푎 +10 
6푎2 −29푎 +10 
÷ 10푎2 −19푎 +6 
12푎 2 −28푎 +15 
6. 
1 
푥+ℎ 
− 
1 
푥 
ℎ 
7. 
푥6−7푥3 − 8 
4푥2−4푥−8 ÷ (2푥 2 + 4푥 + 8) 
8. 
푎 
푏 
− 
푏 
푎 
푎 
푏 
+ 
푏 
푎 
9. 
푡2 −2푡 −15 
푡2 −9 
. 푡2− 6푡 +9 
12−4푡 
10. 
푎−1+푏−1 
푎−2−푏−2
Chapter II 
Equations and 
Inequalities 
25
26 
2.Equations 
To sove an equation means to isolate the variable having a coefficient of 1 
on one side of the equation. By using Addition Property of Equality. 
Examples: 
1) solve r+16=-7 
r+16=-7 
r+16+(-16)=-7+(-16) add -16 to each side 
r=-23 the sum of -16 and 16 is 0 
check: 
r+16=-7 
-23+16=-7 
-7=-7 
2) x+(3.28)=-17.56 
x+(3.28)=-17.56 
x+(3.28)+(3.28)=-17.56+3.28 
x=-14.28 
check: x+(3.28)=-17.56 
-14.28+(-3.28)=-17.56 
-17.56=-17.56 
3) y+21=-7 
y+21+(-21)=-7+(-21) 
y=-28 
check: y+21=-7 
-28+21=-7 
-7=-7 
Equations by Using Subtraction 
The property that used to subtract the same number from each side of an 
equation is called the subtraction property of equality. 
Examples: 
1) x+15=-6 
x+15-15=-6-15 
x=-21 
check: 
x+15=-6 
-21+15=-6 
-6=-6
27 
2) b-(-8)=23 
b+8=23 
b+8-8=23-8 
b=15 
check: 
b-(-8)=23 
15-(-8)=23 
23=23 
Equations by Using Multiplication and Division 
To solve the equation by using multiplication, we use the multiplication property of 
equality. 
For any numbers a,b, and c, if a=b, then ac=bc 
Eamples: 
1) 
5 
12 
= 푟 
24 
24( 5 
12 
) = ( 푟 
24 
)24 multiply each side by 24 
10 = 푟 
Check: 
5 
= 푟 
12 
24 
replace r with 10 
5 
12 
= 
10 
24 
5 
12 
= 
5 
12 
2) 24=-2a 
24=-2a 
− 
1 
2 
(24) = − 
1 
2 
(2푎) 
−12=a 
푐ℎ푒푐푘: 
24 =−2a 
24 =−2a(-12) 
24 = 24 
To solve the equation by using division, we use the division property of equality. 
For any numbers a,b,c with c≠ 0, 
If a=b, then 
푎 
푐 
= 푏 
푐 
. 
Examples: 
1. -6x=11 
−6푥 
−6 
= 11 
−6 
divide each side by -6 
푥 = − 
11 
6
28 
Check: -6x=11 
−6(− 11 
6 
)=11 
11 = 11 
2. 4x=24 
4푥 
4 
= 
24 
4 
X=6 
Check: 4x=24 
4(6)=24 
24=24 
2.2 Appplication of Linear Equations 
In many applications of algebra, the problems are stated in words. They are called 
word problems, and they give relatiomships between known numbers and unknown 
numbers to be determined. In this section we solve word problems by using linear 
equations. There is no specific method to use. However, here are some steps that give a 
possible procedurefor you to follow. As you read through the examples, refer to these 
steps to see how they are applied. 
1. Read the problem carefully so that you understand it. To gain 
understanding, it is often helpful to make a specific axample that involves 
a similar situation in which all the quatities are known. 
2. Determine the quantities that are known and those that are unknown. Use 
a variable to represent one of the unknown quantities inthe equation you 
will obtain. When employing only one equation, as we are in this section, 
any other unknown quantities should be expressed in terms of this one 
variable. Because the variable is a number, its definition should indicate 
this fact. For instance, if the unknown quantity is a length and lengths are 
mesured in feet, then if x is a variable, x should be defined as the number 
of feet in the length or, equivalently, x feet is the length. If the unknown 
quuantity is time, and time is measured in seconds, then if t is the variable, 
t should be defined as the number of seconds in the time or, equivalently, t 
seconds is the time. 
3. Write down any numerical facts known about the variable. 
4. From the information in step 3, determined two algebraic expressions for 
the same number and form an equation from them. The use of a table as 
suggested in step 3 will help you to discover equal algebraic expressions. 
5. Solve the equation you obtained in step 4. From the solution set, write a 
conclusion that answers the questions of the problem. 
6. It is important to keep in mind that the variable represents a number and 
the equation involves numbers. The units of measurement do not appear in 
the equation or its solution set.
7. Check your results by determining whether the condition of the word 
problem are satisfied. This check is to verify the accuracy of the equation 
obtained in step 4 as well as the accuracy of its solution set. 
Example 1 
If a rectangle has a length that is 3cm less than four times its width and its 
perimeter is 9cm, what arethe dimension? 
Solution 
w: the number of centimeters in the width of the rectangle 
4w-3: the number of centimeters in the length of the rectangle 
(4w-3)cm 
w cm w cm 
(4w-3)cm 
w+(4w-3)+ w+(4w-3)=19 
10w-6=19 
10w=25 
29 
w= 
5 
2 
4w-3=4(5 
)-3 
2 
=7 
Example 2 
A man invested part of $15,000 at 12 percent and the remainder at 8 percent. If 
his annual income from the two investments is $1456, how much does he have invested 
at each rate? 
Solution 
x: the number of dollars invested at 12 percent 
15,000-x: the number of dollars invested at 8 percent
Number of Dollar × Rate = Number of Dollars 
invested in Interest 
12 percent investment x 0.12 0.12x 
8 percent investment 15,000-x 0.08 0.08(15,000-x) 
0.12x+0.08(15,000-x)=1456 
30 
0.12x+1200-0.08x=1456 
0.04x=256 
x=6400 15,000-x=15,000-6400 
=8600 
Thus the man has $6400 invested at 12 percent and $8600 at 8 percent. 
Example 3. 
A father and daughter leave home at the same time in separate automobiles. The father 
drives to his office, a distance of 24 km, and the daughter drives to school, a distance of 
28 km. They arrive at their destinations at the same time. What are their average rates, if 
the father’s average rate is 12km/hr less than his daughter’s? 
Solution: 
r: the number of kilometers per hour in the daughter’s average rate 
r-12: the number of kilometers per hour in the father’s average rate 
Number of Kilometers ÷ Number of Kilometers = number of hours 
In Distance per hour in rate in time 
Daughter 28 r 
28 
푟 
Father 24 r-12 
24 
푟−12 
Equation: 
28 
= 
푟 
24 
푟 − 12 
Solve the equation by first multiplying on both sides by the LCD: 
푟(푟 − 12) 
28 
푟 
= 푟(푟 − 12) 
24 
푟 − 12 
(r-12)28=r(24) 
(r-12)7=r(6) 
7r-84=6r 
7r-6r=84 
r=84 r-12=84-12 
=72 
Therefore, the daughter’s average rate is 84km/hr and the father’s average rate is 
72km/hr.
2.3 Quadratic Equation in One Variable 
ax2 + bx + c =0, a≠0 
where a, b, and c are real number constants and a≠0, is called a second degree 
polynomial equation, or quadratic equation , in the variable x. the word “ quadratic “ 
comes from quadrate, meaning square or rectangular when a quadratic equation is written 
in the above manner, it is said to be in standard form. 
The following are examples of quadratic equations in x, written in the form, with 
31 
the indicated values of a, b and c. 
8x2 + 16x -5 = 0 ( a = 8, b = 16, c = -5 ) 
2x 2 - 10 = 0 ( a =2, b = 0, c = -10 ) 
5x2 – 7x = 0 ( a = 5, b = -7, c = 0 ) 
Theorem 1 
If r and s are real numbers, then rs = 0 if and only if r =0 or s=0 
This theorem can be extended to a product of more than two factors. For 
instances, if r, s , t, u єR, then rstu = 0 if and only if at least one of the numbers r, s, t or u 
is 0. 
To find the solution set of the equation 
X2 +3x – 10 = 0 
We factor the left side and obtain 
( x+5 )( x-2 ) 
By applying Theorem 1, it follows that the equations gives a true statement if and only if 
X+5 = 0 or x-2 = 0 
The solution of the first of these equation is -5 and the solution of the second is 2. 
Therefore the solution set of the given equation. 
Example 1: 
1+5x/6 = 2x2 /3 
(6) (1) + (6) 5x/6 = (6) 2x2 /3 
6+ 5x =4x2 
-4x2 +5x +6 = 0 
4x 2-5x -6 = 0 
( 4x+3 ) (x-2 ) = 0 
4x=3 = 0 x-2 =0 
4x =-3 x =2 
X =-3/4 
The solution set is {-3/4, 2 } 
Suppose we have a quadratic equation of the form 
X 2 = d 
That is, there is no first degree term. Then an equivalent equation is 
X2 –d = 0
And, factoring the left member, we obtain 
( x- d )( x + d ) 
We set each factor equal to zero and solve the equations 
X - √ d = 0 x + √d = 0 
x = √d x = -√d 
therefore the solution set of the equation x2 = d is { √d , -√d }. We can abbreviate 
this solution set as {± √d }. Thus 
x2 = d if and only if x= ±√d 
32 
Example 2: 
a. X2 = 25 b. x2 =13 c. x2 =16 
X = ±√ 25 x =√13 x = ± √-16 
X = ± √5 x = ± 4ἰ 
The solution set of the equation 
x2 + 6x-1 =0 
we first add 1 to each side and obtain 
x2+ 6x =1 
we now add to each side the square of one half of the coefficient of x, or 32. We obtain 
x2 +6x +9 = 1+9 
The left side is now the square of x + 3. Thus we have 
(x +3 )2 =10 
Taking the square root of both sides of the equation, we have 
X = 3 = ±√10 
X = -3 ±√10 
This method is called completing the square 
To complete the square of x2 + kx, add the square of one half the coefficient of x; that is, 
add (k/2)2. 
Quadratic Formula 
If a≠0, the solutions of the equation ax2+ bx + c =0 are given by 
푥 = −푏±√푏2−4푎푐 
2푎 
Example 3: Use quadratic formula to find the solution set of the equation 
6x2= 10 + 11x 
×= 
−푏 ± √푏2 − 4푎푐 
2푎 
= 
−(−11) ± √(−11)2 − 4(6)(−10) 
2(6) 
= 
11 ± √121 + 240 
12
33 
= 
11 ± √361 
12 
= 
11 ± 19 
12 
= 
11 + 19 
12 
= 
11 − 19 
12 
= 30 
12 
= −8 
12 
= 5 
2 
= − 2 
3 
The solution set is {− 2 
3 
, 5 
2 
}. 
2.4 INEQUALITIES 
Trichotomy Property of Order 
If a and b are real numbers, exactly one of the following three statements is true. 
A<B B<A A = B 
Transitive Property or order 
If a, b and c are real numbers, and if a<b and b<c then a<c. 
The domain of a variable in an inequality is the set of real numbers for which the 
members of the inequalities are defined. 
Examples: 
4x – 9 < 10 x – 9/10 ≤ x 3 < 5x + 7 ≤ 15 
An example of a quadratic inequality having domain R is 
X2 + 5x > 13 
The inequality 
4x/x + 3 < 6 is rational. 
Any number in the domain for which the inequality is true is a solution of the 
inequality, and the set of all solution is called the solution set. An absolute inequality is 
one that is true for every number in the domain. For instance, if x is a real numbers x + 1 
< x + 2 and x2 ≥ 0 are absolute inequalities. A conditional inequality is one for which 
there is at least one number in the domain that is not in the solution set. To find the 
solution set of a conditional inequality we proceed in a manner similar to that used to 
solve an equation that is we obtain equivalent inequalities until we have one whose 
solution set is apparent. 
PROPERTIES OF INEQUALITIES 
If a, b and c are real numbers 
a. Addition Property 
If a < b, then a + c < b + c 
b. Subtraction Property
34 
If a < b, then a – c < b – c 
c. Multiplication Property 
If a < b and c > 0, then ac < bc 
If a < b and c < 0, then ac > bc 
Example 1 
Find and show on the real number line the solution set of the inequality. 
3x – 8 < 7 
3x – 8 + 8 < 7 + 8 
3x < 15 
1/3 (3x) < 1/3 (15) 
X < 5 
Example 2 
Find and show on the real number line the solution set of the inequality 
X – 7/4 ≤ x 
(4)x – 7/4 ≤ (4)x 
X – 7 ≤ 4x 
X – 7 – 4x + 7 ≤ 4x – 4x + 7 
-3x ≤ 7 
-1/3 (-3x) ≥ (-1/3)7 
X ≥ -7/3 
2.5 POLYNOMIAL AND RATIONAL INEQUALITIES 
A quadratic and inequalities is one of the form ax2 + bx + c < 0 
A critical number of the inequality above is a real root of the quadratic equation 
ax2 + bx + c = 0. 
To solve the inequality 
X2 – 8 < 2x 
We first write an equivalent inequality having all the non-zero terms on one side 
of inequality sign. Thus we have x2 – 2x – 8 < 0 
` (x + 2) (x – 4) < 0 
Example I 
Find and show on the real number line the solution set of the inequality. 
x 2 + 5x ≥ - 6 
x 2 +5x + 6 ≥ 0 
(x+3) (x+2) ≥ 0 
Figure 1 
-4 -3 -2 -1 0 1 2 
The critical numbers are -3 and -2. The points corresponding to these numbers are 
plotted in figure 1 and the following intervals are determined. 
(-∞ , -3) (-3 , -2) (-2 , +∞)
35 
Figure 2 
] [ 
-4 -3 -2 -1 0 1 2 
Thus, the solution set of the given inequality is (-∞ , -3] U [-2 , +∞) , appearing in 
Figure 2. 
Example 2 
Find the solution set of each of the following inequalities 
x2- 6x + 9 < 0 
(x-3) 2< 0 
Because there is no value of x for which (x-3) 2 is negative, there is no solution. 
Therefore, the solution set is 
9x2 + 12x + 4 ≤ 0 
(3x + 2)2 ≥ 0 
Because (3x + 2)2 is non-negative for all values of x , the solution set is the set of all 
real numbers. 
2.7 Equations and Inequalities Involving Absolute Value 
The absolute value of areal number a denoted by /a/ , is given by 
/ a / = { a, if a ≥ 0 } 
/a / = { -a, if a < 0 } 
Example 1 
Find the solution set of the equation 
/ 3x-2 / = / 8-4x / 
3x – 2 = 8 – 4x 
3x + 4x = 8 + 2 
7x = 10 
x = 10/7 
3x – 2 = - ( 8 – 4x ) 
3x – 2 = - 8 + 4x 
3x – 4x = - 8 + 2 
-x = -6 
x=6 
The solution set is {10/7 , 6} 
Example 2 
Find the solution set of the equation 
/ 2x – 7 / <9 
-9 <2x – 7 <9 
-9 + 7 <2x <9 + 7 
-2 <2x <16 
-1 <x <8 
Therefore the solution set is the open interval (-1, 8)
36 
Theorem 1 
If a and b are real numbers, then 
/ ab / = / a / / b / 
Example 3 
/ ( 3x + 2 – 8 / <1 
/ 3x -6 / <1 
/ 3 (x-2) / <1 
/ 3 / / x-2 / <1 
3 / x-2 / <1 
/ x-2 / <1/3 
The Triangle Inequality 
If a and b are real numbers, then 
/ a + b / ≤ / a / + / b / 
If a = 2 and b = 7 , then 
/ a + b / = / 2 + 7 / / a / + / b / = / 2 / + / 7 / 
= / 9 / = 2 + 7 
= 9 = 9 
If a = -2 and b = -7 , then 
/ a + b / = / -2 + -7 / / a / + / b / = / 2 / + / -7 / 
= / -9 / = 2 +7 
= 9 = 9
ASSESSMENT 
37 
TEST I 
A. Find the solution set of the equation. 
1. 3 (6x – 5) = 11 – ( 4 + 8x ) 
2. 49x2 – 64 = 0 
3. 2 + 3 = 0 
6 – y y – 2 
4. (x – 4) ( x + 2) = 7 
5. 2p2 – 4p – 5 = 0 
6. √2x + 5 + x = 5 
7. √y + 2 + √y + 5 - √8 – y = 0 
8. / r + 1 / = 6 
r – 1 
9. / 8x - 2 / = x + 7 
10. / 9x + 7 / = 11 
B.Find the solution set of the inequality and write it with interval notation. Show the 
solution set on the real number line. 
11. 3x – 2 ≤ 20 
12. x2 < 64 
13. 2x + 1 > 1 
x – 5 
14. 3x – 4 ≤ 12 
2x – 3 
15. 2x2 – 3 x ≥ 5 
16. 5x + 8 ≥ 2x – 2 
17. -7 ≤ 8 – 4x ≤ 10 
-5 
18. / x – 10 / ≤ 17 
19. x2 + 3x – 10 ≥ 1 
20. / 4x – 5 / < 15 
C.Solve for x in terms of the other symbols 
21. d/10x – d/5 = 1/x 
22. rsx2 + s2 x + rtx + st = 0 
23. x2 + b2 = 2 bx + a2 
24. 5x2 – 4xy – x + y -1 = 0 
25. x2 + xy + 4x - 1 = 0 
Show that the two inequalities are equivalent.
26. A woman leaves home at 10 A.M and walks to her office at he rate of 7 
km/hr. At 10. 15 A.M. the womans daughter leaves home and rides her bicycles at a rate 
15 km/hr along the same route to school. At what time does she overtake her mother? 
27. How many liters of a solution that is 85 percent glycerine should be added to 
30 liters of a solution that is 40 percent glycerine to give a solution that is 60 percent 
glycerine. 
28. In a long distance race around a 600 m track the winner finished one lap ahead 
of the loser. If the average rate of the winner was 8 m/sec and the average rate of the loser 
was 7. 25 m/s how soon after the start did the winner complete the race? 
29. An automobile radiator contains 9 liters of a solution that is 10 percent 
antifreeze and 95 percent water. How much of the solution should be drained and 
replaced with pure antifreeze to obtain a solution that is 45 percent antifreeze? 
30. The perimeter of a rectangle must not be greater than 50 cm and the length 
must be 10 cm. What is the range of values for the width? 
38 
TEST II 
A. Find the solution set of the equation. 
1. X2 = 64 
2. 6t2 – 11 = 0 
3. 9x2 = x 
4. 3t/3t + 4 + 2/5 – 1/3t – 4 
5. 64y2 – 80y + 25 = 0 
B. Find the solution set of the equation by completing the square. 
6. X2 + 6x + 8 = 0 
7. 4x2 + 4x – 3 = 0 
8. 3y2 + 4y + 2 = 0 
C. Find the solution set of the equation by using the quadratic form. 
9. X2 + 1 = 6x 
10. 5y2 – 4y – 2 = 0 
D.Find the solution of the inequality and write it with interval notation. 
1. 2x – 1 < 6 
2. -3 > 4x + ¾ 
3. 11 ≥ 5x – 4 > 1 
4. 2x – 4 < 6 
5. 1 < 4x – 1/3 < 5 
6. 5x – 2 < 5x – 2/4 
7. 10 – 3x > 4x – 5/-3 
8. 5 ≤ 3x – 4 < 14 
9. -1 < 7 – 2x/5 ≤ 5 
10. 6 ≤ 2 – x ≤ 8 
E.Find the solution set of the equation. 
11. / x – 8 / = 9 
12. / 4y – 10 / = 5
39 
13. / 3t -2 / = t2 
14. / x – 1 / = x2 
15. / 5t – 15 / = 20 
CHAPTER III 
Points and 
Equations
40 
3.1 Points in a Plane 
Ordered pairs of real numbers are important in our discussions. Any two real 
numbers form a pair. When the order of appearance of the numbers is a significant, we 
call it an ordered pair. If x is the first real number and y is the second, this ordered pair is 
denoted by writing them in parenthesis with a comma separating them as (x,y). 
The set of all ordered pairs of real numbers is called the number plane, denoted by 
R2 , 
And each ordered pair (x,y) is a point in the number plane. The intersection of these two 
perpendicular line segments is the point P, associate with the ordered pair (x, y). Refer to 
figure 1, the first number x of the pair is called the abscissa or x-coordinate of P, and the 
second number y is called the ordinate or the y-coordinate of P. 
The x and y axes are called the coordinate axes. They divide the plane into four 
parts called quadrants. The first quadrant is the one in which the abscissa and the ordinate 
are both positive, that is , the upper right quadrant. The other quadrant are numbered in 
the counterclockwise direction, with the fourth being the lowered right quadrant. 
Pythagorean Theorem 
In a right triangle, if a and b are the lengths of the perpendicular sides and c is the 
length of the hypotenuse then, a2 + b2 = c2. 
Distance Formula 
The distance between two points P1 (x1 , y1) and P2 ( x2 , y2) is given by 
P1P2 = √(x2 – x1)2 + (y2 –y1)2 
Converse of Pythagorean Theorem 
If a ,b and c are the length of the side of a triangle and a2 + b2 = c2, then the 
triangle is a right triangle, and c is the length of the hypotenuse. 
Midpoint Formulas 
If M (x,y) is the midpoint of the line segment from P1 (x1 , y1) to P2 ( x2 , y2), then 
x = x1 + x2 y = y1 + y2 
2 2 
Solution 
From the midpoint formulas, if M is the point (x,.y), then 
x = 5 – 1 y = -3 + 6 
2 2 
= 2 = 3/2 
3.2 Graphs of Equations 
The graph of an equation in R2 is the set of all points in R2 whose coordinate are 
numbers satisfying the equation. 
Example 1 
Draw a sketch of the graph of the equation y2 – 4x = 0 
Solution
y2 = 4x 
y = ± 2√x 
y = 2 √x and y = -2 √x 
41 
Symmetry of two points 
Two points P and Q are said to be a symmetric with respect to a line if and only if 
the line is the perpendicular bisector of the line segment PQ. Two points P and Q are said 
to be symmetric with respect to a third point if and only if the third point is the midpoint 
of the line segment PQ. 
Symmetry of a graph 
The graph of an equation is symmetric with respect to a line l if and only if for 
every point P on the graph there is a point Q, also on the graph , such that P and K are 
symmetric with respect to l. The graph of an equation is symmetric with respect to a point 
R if and only if for every point P on the graph there is a point S, also on the graph such 
that P and S are symmetric with respect to R. 
Example 2 
Draw a sketch of the graph of the equation. 
2y = x3 
2(-y) = (-x)3 
-2y = -x3 
Circle 
A circle is the set of all points in a plane equidistant from a fixed point. The fixed 
point is called the center of the circle and the constant equal distance is called the radius 
of a circle. 
Equation of a Circle 
An equation of the circle with center at the point (h,k) and radius r is 
(x – h)2 + (y – k)2 = r2 
Example 3 
Find an equation of the circle having the diameter with endpoints at A (-2,3) and 
B (4,5). 
Solution 
ℎ = 
−2 + 4 
2 
= 1 
푘 = 3+5 
2 
=4 
3.3 Equation of a line 
Slope 
If P1 (x1 , y1) and P2 (x2 , y2) are any two distinct points on line l, denoted by m, is given 
by 
m = y2 – y1 
x2 – x1 
If the slope of a line is positive, then as the abscissa of a point on the line 
increases, the ordinate increases. A line whose slope is negative, for this line as the 
abscissa of the point on the line increases, the ordinate decreases. If a line is parallel to 
the x axis, then y2 = y1 ; so the slope of the line is zero. If a line is parallel to the y axis, x2
= x1 ; thus the fraction y2 – y1 / x2 – x1is meaningless because we cannot divide by zero. 
Thus the slope of a vertical line is not defined. 
42 
Example 1 
Determine the slope of a line. 
A (3,7) and B (-2,-4) 
m = -4 – 7 /-2 -3 
= -11 / -5 
= 11 / 5 
A (-3,4) and B (5,4) 
m = 4 – 4 / 5 – (-3) 
= 0/8 
= 0 
The point slope form we choose the particular point (0,b) (that is , the point 
where the line intersects the y axis ) for the point ( x1,y1) , we have 
y - b = m ( x – 0 ) 
y = mx + b 
The number b, the ordinate of the point where the line intersects the y axis, is the 
y intercept of the line. Consequently, the preceding equation is called the slope-intercept 
form. 
Theorem 1 
The graph of the equation 
Ax + By + C = 0 
Where A, B, and C are constants and where not both A and B are zero is a line. 
Theorem 2 
If l1 and l2 are two distinct nonvertical lines having slopes m1 and m2, 
respectively, then l1 and l2 are parallel if and only if m1 = m2 
Theorem 3 
Two nonvertical lines l1 and l2, having slopes m1 and m2 , respectively, are 
perpendicular if and only if m1m2 = - 1
ASSESSMENT 
TEST I 
A.Find the center and radius of the circle. 
43 
1. x2 + y2 + 4x – 6y -3 = 0 
2. 3x2 + 3y2 + 4x -4 = 0 
B. Determine the slope of the line and find an equation of the line. 
3. (1,-3) and (4,5) 
4. (2,5) and (6,9) 
5. (9,8) and (-2,4) 
6. (6,2) and (-9,7) 
7. (4,2) and (-7,8) 
8. (-1,3) and (6,1) 
9. (4-6) and (5,6) 
10. (9,7) and (6,-4) 
C.Find the slope and y-intercept of the line having the given equation . 
11. 2x – 5y - 10 = 0 
12. 2x + 3y + 12 = 0 
13. Prove that the quadrilateral having vertices at (2,1) , (6, -2) , (9,6) and 
(7, 10) is a rectangle. 
14. Find an equation of the line through the point ( -1,6) and perpendicular 
to the line whose equation is 4x + 2y – 5 = 0. 
15. Prove that the lines having the equations 2x + 5y + 20 = 0 and 5x – 2y - 
10 = 0 are perpendicular. 
D. For the given points A and B , find the directed distances : (a) AB : (b) BA 
16. A (2,4) and B (7,8) 
17. A (4, - 8) and B (5, - 10) 
18. A (7, - 10) and B (- 8, 7 ) 
19. A (- 4, 5 ) and B (10,3) 
20. Given that A is the point (-2, 3) and B is the point ( -x , 3) , find x such 
that (a) AB = -8 ; (b) BA = -8 
21. Given that A is the point (-4, y) and B is the point (-4, 3) , find y such 
that (a) AB = -3 ; (b) BA = -3 
22. A (10,-5) , B (-4, 6) , C (-2, 9) 
23. A (4,10) , B (7, -3) , C (-1, -1) 
E. Find the center and radius of the circle. 
1. 2x2 + 2y2 – 2x + 2y + 7 = 0 
2. x2 + y2 – 10x – 10y + 25 = 0 
3. 3x2 + 3y2 + 4y – 7 = 0 
4. x2 + y2 - 6x - 8y + 9 = 0
5. x2 + y2 + 2x + 10y + 18 = 0 
F. Find an equation of the line satisfying the given condition. 
6.(a) the slope is 4 and through the point ( -3 , 2) 
(b) through the two points ( -2, -6) and ( 4,5 ) 
7 (a) the slope is -7 and through the point (-5,4) 
44 
(b) through two points (5,7) and (-6,9) 
8. (a) through the point ( 2,-8) and parallel to the x-axis 
(b) through the point (3,5) and parallel to the y-axis 
G. Find the slope and y-intercept of the line having the given equation. 
9. (a) x + 3y -6 = 0 ; (b) 4y – 9 = 0 
10. (a) x – 4y -2 ; (b) 4x = 3y
Chapter IV 
Functions and their 
Plane 
45
46 
4.1 Functions 
A function can be thought of as a correspondence from a set x of real numbers x 
to a set y of real numbers y, where the number y is unique for a specific value of x. 
A function is asset of ordered pairs of real numbers (x , y) in which no two 
distinct ordered pairs have the same first number. The set of all admissible values of x is 
called the domain of the function , and the set of all resulting values of y is called the 
range of the function. 
Graph of a function 
If f is a function , then the graph of f is the set of all points (x , y) in R2 for which 
(x , y) is an ordered pair in f. 
The graph of a function can be intersected in a vertical line at most one point. 
Example 1 
The function h is defined by 
h{ (x , y) y = / x/ } 
The required formula of h, x can be any real number. Therefore the domain is ( - ∞ 
, + ∞ ). Because we observe from figure 1 that y can be any non negative number the 
range is [ 0 , + ∞ ] 
Quadratic Function 
The general quadratic function is defined by 
F ( x ) = ax2 + bx + c 
Where a, b and c are constants representing real numbers and a≠0. The graph of f is the 
same as the graph of an equation 
Y = ax2 + bx + c 
If the function f is the defined by 
F ( x ) = -2 x2 + 8x – 5 
The graph of f is the same as the graph of an equation 
Y = 2x2 + 8x – 5 
This equation is equivalent to 
2(x2 – 4x) = -y – 5 
To complete the square of the binomial with the parentheses, we add 2(4) to both sides of 
the equation and we have 
2(x2 – 4x +4) = -y – 5 + 8 
(x – 2 )2 = 4p (y – k) 
Where (h,k) is (2,3) and = -1/8. Therefore the vertex of the parabola is at 92,3) and the 
axis is the line x = 2. Because p<0 , the parabola opens downward we find a few points 
on the parabola. 
The zeros of the function are the values of x for which f(x) = 0. 
When the graph of a quadratic function opens upward the function has a minimum 
value, which occurs at the vertex of the parabola. There is no maximum value for such a
function. When the parabola opens downward. The function has a maximum value 
occurring at the vertex ;it has no minimum value 
We now apply the method used in the solution to the general quadratic function defined 
by 
47 
f (x) = ax2 + bx + c 
-1 = ax2 + bx + c 
ax2 + bx = y – c 
a (x2 +b/a x) = y – c 
a (x2 + b/a x + b2/4a2 0 = y – c +b2/4a 
(x = b/2a )2 = 1/a ( y+b2 – 4ac/4a) 
The graph of this equation is a parabola having its vertex at the point where x = -b/2a. if 
a>0, the parabola opens upward and so f has a minimum value at the point where x + - 
b/2a 
Theorem 1 
The quadratic function defined by f (x) = ax2 +bx +c, where x = -b/2a. if a>o, the extreme 
value is a minimum value, and if a,a0, the extreme value is a maximum value 
Example use theorem 1 to find either a maximum or minimum value of the function g if 
g (x) = -5/2 x2 + 8x -10 
For the given quadratic function a = -5/2 and b =8. Because a<0, g has a maximumu 
value at the point 
x = -b/2a 
= -8/2(-5/2) 
= 8/5 
The maximum value is 
G (8/5) = -5/2(8/5)2 + 8(8/5) – 10 
=-5/2 (64/25) + 64/5 – 10 
= -18/5 
Rational Functions 
A rational function is of the form 푓(푥) = 푝(푥) 
, where p(x) and q(x)are polynomial 
푞(푥) 
functions and 푞(푥) ≠ 0. A graphing calculator is a good tool for exploring graphs of 
rational functions. 
Graphs of rational functions may have breaks in continuity. This means that, 
unlike polynomials functions which can be traced with a pencil that never leaves the 
paper, a rational function may not be traceable. Breaks in continuity can occur where 
there is a vertical asymptote or point discontinuity. Point of discontinuity is like a hole 
in the graph. Vertical asymptote and point of discontinuity occur for the values of x that 
make the denominator of the rational function zero. 
Graphing Rational Functions
48 
Connection: 
Mathematical History 
Mathematician Maria Gaetana Agnesi was one of the greatest women scholars of 
all time. In the analytic geometry section of her book Analytical Institutions, Agnesi 
discussed the characteristics of the equation 푥 2푦 = 푎2 (푎 − 푦), called the “curve of 
Agnesi”. The equation can be expressed as 푦 = 푎2 
푥2+푎2. 
Because the function described above is the ratio of two polynomial expression a3 and 
푥 2 + 푎2 is called a rational function. A rational function is function of the form 푓 (푥) = 
푝(푥) 
, where p(x) and q(x) are polynomial functions and q(x)≠ 0 
푞(푥) 
Examples of Rational Function: 
풇(풙) = 푥 
푥− 1 
푔(푥) = 3 
푥−3 
ℎ(푥) = 푥 +1 
(푥+2) (푥−5) 
The lines that graph of the rational function approaches is called 
Asymptote. If the function is not define when 푥 = 푎, then either there is 
a “hole” in the graph 푥 = 푎. 
POLYNOMIALS 
The expression x2+2xy+y2 is called a polynomial. A polynomial is a monomial or 
a sum of monomials. The monomials that make up the polynomial are called the terms of 
the polynomial. The two monomials xy and xy ca be combined because they are like 
terms. Like terms are two monomials that are the same, or differ only by their numerical 
coefficient. An expression like m2+7mb+12cd with three unliked terms is called 
trinomial. An expression like xy+b3 with two unliked terms is called binomials. The 
degree of a polynomial is the degree of the monomial with the greatest degree. Thus, the 
degree of x2+2xy+y2 is 2.
Remember: 
If a polynomial contains only one term, it called monomial; if two terms, it is called 
binomial; if it is contains three terms it is called trinomial. If a polynomial has more than 
three terms, it is called multinomial. 
The following table shows examples of polynomials. 
Polynomial No. of terms Class by terms 
12푥푦2 
5푥푦 + 3푧2 푣 
49 
4푥 2푦 − 3푥푦2 − 푥푦 
3푥 − 5푦 + 푎 + 10푏 
5푥 4 + 2푥 3 − 푥 2 − 6푥 + 8 
1 
2 
3 
4 
5 
Monomial 
Binomial 
Trinomial 
Multinomial 
Multinomial 
Example: 
Determine whether or not each expression is a polynomial. Then state the degree of each 
polynomial. 
a. 
2 
7 
x4y3 – x3 
This expression is a polynomial. The degree of the first term is 4 + 3 or 7, and the 
degree of the second term is 3. The degree of the polynomial is 7. 
b. 9 + √푥 − 3 
This expression is not polynomials because √푥 is not a monomial. 
The FOIL Method is an application of the distributive property that make the 
multiplication easier. 
FOIL Method of 
Multiplying 
Polynomial 
The product of two binomial is he sum of the products of: 
F the first terms 
O the outer terms 
I the inner terms 
L the last terms
50 
Example: 
Find (k2 +3k +9) (k +3) 
(k2 +3k +9) (k +3) 
= 푘2 (k+3) + 3k (k +3) +9(k +3) distributive property 
=k2∙k+k2∙ 3 + 3푘 ∙ +9 ∙ 푘 + 3 ∙ 9 distributive property 
= 푘2∙ 푘 + 푘2∙ 3푘2+9푘 + 9푘 + 27 
=k2+6푘2+18푘 + 27 combined like terms 
Dividing Polynomials 
You can use a process similar to long division of a whole numbers to divide a 
polynomial by a polynomial when doing the division, remember that you can only add ad 
subtract like terms. 
Example: 
Simplify: c2 –c –30 
c –6 
c 
푐 − 6√푐2 − 푐 − 30 
푐2− 6 
5푐 −30 
−푐 − (−6푐) = −푐 + 6푐 표푟 5푐 푐 + 5 
푐 − 6√푐2 − 푐 − 30 
푐2−6 
5푐 −30 
5푐 −30 
0 
Polynomial functions 
A polynomial function is a function that can be defined by evaluating a 
polynomial. A function f of one argument is called a polynomial function if it satisfies 
푓(푥) = 푎푛푥 푛 + 푎푛−1푥 푛− 1 + ⋯ + 푎2푥 2 + 푎1푥 + 푎0 
For all arguments x, where n is a non-negative integer and a0, a1, a2, ..., an are constant 
coefficients. 
For example, the function f, taking real numbers to real numbers, defined by 
푓(푥) = 푥 3 
is a polynomial function of one variable. Polynomial functions of multiple variables can 
also be defined, using polynomials in multiple indeterminates, as in 
푓(푥, 푦) = 2푥 3 + 4푥 2푦 + 푥푦5 + 푦2 − 7 
An example is also the function 푓(푥) = cos(2푎푟푐표푠(푥)) which, although it doesn't look 
like a polynomial, is a polynomial function on [−1,1] since for every from [−1,1] it is 
true that 푓(푥) = 2푥 2 −1 
Polynomial functions are a class of functions having many important properties. They are 
all continuous, smooth, entire, computable, etc
Graphs of Polynomial Function 
A polynomial function in one real variable can be represented by a graph. 
51 
 The graph of the zero polynomial 
f(x) = 0 
is the x-axis. 
 The graph of a degree 0 polynomial 
f(x) = a0, where a0 ≠ 0, 
is a horizontal line with y-intercept a0 
 The graph of a degree 1 polynomial (or linear function) 
f(x) = a0 + a1x , where a1 ≠ 0, 
is an oblique line with y-intercept a0 and slope a1. 
 The graph of a degree 2 polynomial 
f(x) = a0 + a1x + a2x2, where a2 ≠ 0 
is a parabola. 
 The graph of a degree 3 polynomial 
f(x) = a0 + a1x + a2x2, + a3x3, where a3 ≠ 0 
is a cubic curve. 
 The graph of any polynomial with degree 2 or greater 
f(x) = ao + a1x + a2x2 + ... + anxn , where an ≠ 0 and n ≥ 2 
is a continuous non-linear curve. 
 The graph of a non-constant (univariate) polynomial always tends to 
infinity when the variable increases indefinitely (in absolute value) 
Polynomial graphs are analyzed in calculus using intercepts, slopes, concavity, and end 
behavior. 
Polynomial of degree 2: 
f(x) = x2 − x − 2 
= (x + 1)(x − 2) 
Polynomial of degree 3: 
f(x) = x3/4 + 3x2/4 − 3x/2 − 2 
= 1/4 (x + 4)(x + 1)(x − 2)
52 
Polynomial of degree 4: 
f(x) = 1/14 (x + 4)(x + 1)(x − 1)(x − 3) + 
0.5 
Polynomial of degree 5: 
f(x) = 1/20 (x + 4)(x + 2)(x + 1 )(x − 
1)(x − 3) 
+ 2 
Polynomial of degree 2: 
f(x) = x2 − x − 2 
= (x + 1)(x − 2) 
Polynomial of degree 3: 
f(x) = x3/4 + 3x2/4 − 3x/2 − 2 
= 1/4 (x + 4)(x + 1)(x − 2)
53 
Polynomial of degree 4: 
f(x) = 1/14 (x + 4)(x + 1)(x − 1)(x − 3) + 
0.5 
Polynomial of degree 5: 
f(x) = 1/20 (x + 4)(x + 2)(x + 1 )(x − 
1)(x − 3) 
+ 2 
Inverse function 
Definition of Inverse 
Function 
Two function f and g are inverse function if and only if both of 
their compositions are the identity function. That is, 
(푓 ∘ 푔) = 푥 and (푔 ∘ 푓)(푥) = 푥 
An inverse function is a function that "reverses" another function: if the 
function f applied to an input x gives a result of y, then applying its inverse 
function g to y gives the result x, and vice versa. i.e., f(x) = y if and only if g(y) = x. 
A function f that has an inverse is said to be invertible. When it exists, the inverse 
function is uniquely determined by f and is denoted by f −1, read f 
inverse. Superscripted "−1" does not, in general, refer to numerical exponentiation. 
In some situations, for instance when f is an invertible real-valued function of a real 
variable, the relationship between f andf−1 can be written more compactly, in this 
case, f−1(f(x)) = x = f(f−1(x)), meaning f−1 composed with f, in either order, is the identity 
function on R.
54 
Property of Inverse 
Function 
Suppose 푓 and푓−1 are inverse function. Then 푓(푎) = 푏 and 
only if 푓−1(푏) = 푎 
Definition of inverse 
Relationship 
Two relationships are inverse relationship if and only if 
whenever one relation contains the element ( a, b ), the other 
relation contains the element ( b, a )
ASSESSMENT 
TEST I. 
A. Draw a sketch of the graph of the function and determine its domain and range. 
55 
1. f = {(x,y) / y = 3x – 1 } 
2. F = { ( x, y / y = 2x2 } 
3. F = { ( x, y) / y = / 3x + 2 / } 
4. G = { ( x, y} = x2 – 4 } 
x-2 
5. g = { ( x, y /y = (x2 – 4 ) ( x – 3 ) } 
x2 – x – 6 
6. f : y = { -2 if x ≤ 3 } 
{ 2 if 3 < x } 
7. g : y = { -4 if x< -2 } 
{-1 if -2 ≤ x ≤ 2 } 
{ 3 if 2 < x } 
8. f : y = { 3x +2 if x ≠ 1 } 
{ 8 if x = 1 } 
9. G : y { 9 – x2 if x ≠ -3 } 
{ 4 if x = -3} 
10. G : y = { x2 – 4 if x < 3 } 
{2x – 1 if 3 ≤ x } 
B. Find the zeros of the function. 
1. f(x) = x2 – 2x – 3 
2. f(x) = 2x2 – 2x – 1 
3. f(x) = x2 – 3x + 1 
4. f(x) = 6x2 – 7x – 5 
C. Use theorem 1 to find either maximum or minimum value of the function. 
5. f(x) = -1/2 ( x2 + 8x + 8) 
6. G (x = 1/8 ( 4x2 + 12x -9 ) 
7. g (x) = 3x2 + 6x + 9 
8. f(x) = 2 + 4x - 3x2 
9. Find two numbers whose sum difference is 10 and whose product is a 
minimum. 
10. Find two numbers whose sum is 20 and whose product is a maximum.
Chapter V 
Exponential and 
Logarithmic 
Function 
56 
c
57 
EXPONENTIAL FUNCTION 
An equation of the form 푦 = 푎 ∙ 푏푥 , 푤ℎ푒푟푒 푎 ≠ 0, 푏 > 0, and 푏 ≠ 1, ia called 
exponential function with base b. 
The logarithm of a number is the exponent to which another fixed value, the base, must 
be raised to produce that number. Logarithms are exponents. They were once used t 
simplifies calculations, but the advent of calculators and computers caused calculation 
with logarithms to be used less and less. 
Definition of Logarithm Suppose 푏 > 0 and 푏 ≠ 1. for 푛 > 0, there is a number p such 
that 퐿표푔푏 푛 = 푝 if and only if 푏푝 = 푛. 
The chart below shows some equivalent exponential and logarithmic equations. 
Exponential Equation Logarithmic Equation 
52 = 25 
105 = 100,000 
80 = 1 
2−4 = 
1 
16 
9 
1 
2 = 3 
log5 25 = 2 
log10 100,000 = 5 
log8 1 = 0 
log2 
1 
16 
= −4 
log9 3 = 
1 
2 
Integral Exponents 
Basic Laws of Exponents 
1. 풃풙 ∙ 풃풚 = 풃풙+풚 
2. 
풃풙 
풃풚 = 풃풙−풚 (풃 ≠ ퟎ) 
3. 풊풇 풃 ≠ ퟎ, ퟏ, −ퟏ 풕풉풆풏 풃풙 = 풃풚 풂풏풅 풐풏풍풚 풊풇 풙 = 풚 
4. (풂풃)풙 퐜퐱 퐛퐲 
5. (풂 
풃 
) 
풙 
= 풂풙 
풃풙 (풃 ≠ ퟎ) 
6. 풊풇 풙 ≠ ퟎ, 풂 > ퟎ, 풃 > ퟎ, 풕풉풆풏 풂풙 = 풃풙 풊풇 풂풏풅 풐풏풍풚 풊풇 풂 = 풃 
7. (풃풙)풚 = 푩풙풚 
8. 풃ퟎ = ퟏ (풃 ≠ ퟎ) 
9. 풃−풙 = ퟏ 
풃풙 
Each of the above rules should be familiar to you from algebra I. 
Here are some sample problems with their solutions. 
1) Watch the difference between these two: 
a) (-3)-2 b) -(3)-2 
The first one is squaring a negative number and the second is squaring a positive 
number and then making the whole result negative. 
a) = 1/(-3)2 = 1/9 b) 1/-(3)2 = -1/9 
c) 7 . 2-3 = d) (7 . 2)-3 = 
The first one raises the power then multiplies, while the second one multiplies first
58 
then raises the power. 
c) = 7/8 d) = 14-3 = 1/143 = 1/2744 
e) (3−2 + 3−3)−1 = 
= ( 1 
9 
+ 1 
27 
) 
−1 
= ( 3 
27 
+ 1 
27 
) 
−1 
= ( 4 
27 
) 
1 
= 27 
4 
In the above example our first step is to work inside the grouping symbols and get a 
common denominator. Then add the two fractions. Only when you have a single 
fraction, is it permitted to invert the fraction. 
f) (푎−2 − 푏−2)−1 
= ( 1 
푎 −2 − 1 
푏2) 
−1 
= ( 푏2 
푎2 푏2 − 푎2 
푎2푏2) 
−1 
= (푏2−푎2 
푎2 푏2 ) 
−1 
= 푎2 푏2 
(푏−푎)(푏+ 푎) 
In the above example we again simplify inside the grouping symbols and get a common 
denominator. Once we have a single fraction in step 3 we can invert the fraction. Notice 
the factoring in the last step! 
g) 
(3푎−1) 
2 
(3푎−1)−2 
= (3푎−1 )4 
= 81푎−4 = 81 
푎4 
In this example, the numerator and denominator have the same base. We can apply the 
division rule by subtracting the exponents. Then simplify. Remember, no negative 
exponents should be left in the answer. 
An important type of rule can now be stated using exponents. It is a growth or decay 
problem. We can mathematically model this function by using the following: 
A(t) = Ao(1 + r)t 
Where Ao is the initial amount at time t = 0 
r is the rate (as a decimal) 
t is the time 
A(t) is the amount after the time t. 
If r > 0, then it is an exponential growth. 
If -1 < r < 0, then it decays exponentially.
1) Suppose a bike costs $100 now and it increases at a rate of 5% per year. What will be 
the cost of the bike in 4 years? 
59 
Solution: Ao = 100, r = 5% = .05, t = 4 
A(4) = 100(1 + .05)4 = 100(1.05)4 = 121.55 
The bike will cost $121.55 in 4 years. (Rounded to the nearest penny) 
2) Suppose a car is worth $15,000 new. What will it be worth in 3 years if it decreases at 
a rate of 12% per year? 
Solution: This is a decrease problem with 
Ao = 15000, r = -.12, t = 3 
A(3) = 15000(1-.12)3 = 15000(.88)3 = 10222.08 
The car will be worth $10222.08 in 3 years. 
1). 
4−4 
4−2 + 4−3 
= 4−4 
4−2+4−3 ∙ 44 
44 
= 1 
42 + 41 
= 1 
16 +4 
= 1 
20 
One of the easier ways to do this problem is to multiply both numerator and denominator 
by the positive power of the biggest negative exponent. In this case we multiplied by 44. 
This greatly simplifies the problem. 
Rational Exponents 
All rules presented in the previous section were defined for integers only. All of 
the properties in the last section can also be extended to include rational exponents 
according to the following definitions: 
1) 푏 
1 
푛 = √푏 푛 
2)푏 
푚 
푛 = ( 푛√푏) = 푛√푏푚 
Example: 
1) 8 
1 
3 
=3√8=2 
2) 2) 8− 
1 
3 = 1 
= 1 
3√8 2 
3 
= 23 + 8 
3) 163/4 = ( √16 4 ) 
3 
4 = 1 
4) 16− 
(4√16 ) 
3 = 1 
23 = 1 
8 
5) ( 8 
27 
) 
1 
3 = √ 8 
27 
3 
= 2 
3
60 
6) ( 8 
27 
) 
− 
2 
3 = (27 
8 
) 
2 
3 = ( √27 
8 
3 
2 
= (3 
) 
2 
) 
2 
= 9 
4 
7) (1001/2 - 361/2)2 = (10 - 6)2 = 42 = 16 
8) x1/2(x3/2 + 2x1/2) = x2 + 2x 
9) 
( √4푎2) 3 2 
6√4푎2 
= 
(4푎2 ) 
2훽 
(4푎2) 
1 
6 
1 
6 = (4푎2) 
= (4푎2)2훽− 
1 
2 = 2푎 
We can use these rules to solve for x when x is the exponent. This method will only 
work if the bases are the same. Check back in section 5-1 for the appropriate rule!! 
Example: 
1) 16x = 25 We can write both sides in base two. 
24x = 25 Now use the fact that the bases are the same, the exponents are = 
4x = 5 And solve for x!! 
x = 5/4 To check it, take the 5/4 root of 16 = 25!! 
2) 271-x = (1/9)3-x You need to make both bases the same. How about 3!! 
33(1-x) = 3-2(3-x) Notice the power on the right side is negative. 
3(1 - x) = -2(3 - x) Because the bases are =, the exponents must be = 
3 - 3x = -6 + 2x Solve for x. 
3 = -6 + 5x 
9 = 5x 
9/5 = 
The growth and decay formula can also be used with rational numbers. Consider the 
following: 
1) The cost of a computer has been increasing at 7% per year. If it costs $1500 now, 
find the cost: 
a) 2 years and 6 months from now 
b) 3 years and 3 months ago. 
Solutions: 
a) Ao = 1500, r = .07 and t = 2.5 
A(2.5) = 1500(1 + .07)2.5 = 1500(1.07)2.5 = 1776.44 
b) Ao = 1500, r = .07, and t = -3.25 
A(-3.25) = 1500(1.07)-3.25 = 1203.91
Exponential Functions 
Any function in the form f(x) = abx, where a > 0, b > 0 and b not equal to 1 is called 
an exponential function with base b. Let's take a look at a couple of simple exponential 
graphs. 
f(x) = 2x 
X f(x) 
3 8 
2 4 
1 2 
0 1 
-1 1/2 
-2 1/4 
-3 1/8 
Notice the domain is all real numbers and the range is y > 0. As x gets larger (right), y 
gets very large. As x gets smaller(left), y approaches zero asymptotically. Notice also 
that the graph crosses the y-axis at (0, 1). The above is the general shape of an 
exponential with b > 1. This is an example of exponential growth. 
61 
Now let's look at the graph of 
f(x) = (1/2)x 
X f(x) 
3 1/8 
2 1/4 
1 1/2 
0 1 
-1 2 
-2 4 
-3 8
Observe that this graph is the reflection about the y-axis of the first graph. The domain 
is still all real numbers and the range is y > 0. The y-intercept is (0, 1). This is the 
general form of an exponential graph if 0 < b < 1. It is an example of anexponential 
decay. 
Look at the following graphs that illustrate the general properties of exponentials. 
62
63 
Do you see the similarities of each graph? 
How about this one? 
Many of the functions associated with exponential growth or decay are functions of time. 
We have already had one form: 
A(t) = Ao(1 + r)t 
A second form looks like: 
A(t) = Aobt /k 
where k = time needed to multiply Ao by b 
Rule of 72 
If a quantity is growing at r% per year then the doubling time is approximately 72/r 
years. 
For example, if a quantity grows at 10% per year, then it will take 72/10 or 7.2 years to 
double in value. In other words, it will take you 7.2 years to double your money if you 
put it into an account that pays 10% interest. At the current bank rate or 2%, it will take 
you 72/2 or 36 years to double your money!! Boy, jump all over that investment!! 
Sample Problems 
1) Suppose you invest money so that it grows at A(t) = 1000(2)t /8 
a) How much money did you invest? 
b) How long will it take to double your money? 
Solutions: 
a) The original amount in the formula is $1000. 
b) This means what time will it take to get $2000. 
2000 = 1000(2)t /8 
2 = 2t /8 
1 = t/8 
8 = t 
It will take 8 years to double your money!!
2) Suppose that t hours from now the population of a bacteria colony is given by: P(t) = 
150(100)t /10 
a) What is the initial population? 
b) How long does it take for the population to be multiplied by 100)20/10 = 
64 
150(100)2 = 1,500,000 
c) What is the population at t = 20? 
Solutions: 
a) It is 150 from the original equation. 
b) It takes 10 hours. That's the definition of the exponential 
function. 
c) P(20) = 150() The half life of a substance is 5 days. We have 4 kg 
present now. 
a) Write a formula for this decay problem. 
b) How much is left after 10 days? 15 days? 20 days? 
Solutions: 
a) A(t) = 4(1/2)t /5 
b) A(10) = 4(1/2)10/5 = 4(1/2)2 = 1 kg. 
A(15) = 4(1/2)15/5 = 4(1/2)3 = 1/2 kg. 
A(20) = 4(1/2)20/5 = 4(1/2)4 = 1/4 kg. 
3) The value of a car is given by the equation V(t) = 6000(.82)t 
a) What is the annual rate of depreciation? 
b) What is the current value? 
c) What will be the value in three years? 
Solutions: 
a) It is 1 - .82 or .18 = 18% 
b) The current value is given in the formula, $6000. 
c) V(3) = 6000(.82)3 = 3308.21 Which is $3308.21 
The number e and the function ex 
Definition of the irrational number e
Without getting into a discussion of limits right now, we can get an idea of what's 
happening by taking increasingly larger values of n. We will talk about limits later on in 
the year. Study the following table of values and use your calculator to double check the 
results: 
65 
N (1 + 
1/n)n 
10 2.593742 
100 2.704814 
1000 2.716924 
10,000 2.718146 
100,000 2.718268 
1,000,000 2.718280 
If you study this chart, you see that the number e approaches a value of 2.718 . . . The 
function ex is called the natural exponential function. The graph of ex and e-x are graphed 
below: 
Notice, they fit the pattern of the previous section. The number e appears in many 
applications of physics and statistics. We will take a close look at the number e and how 
it relates to compound interest. 
Compound Interest Formula 
퐴(푡) = 퐴0 (1 + 푟 
푛 
) 
푥푡 
Where: 
A(t) = amount after time t. 
Ao = Initial amount 
r = rate in decimal 
n = number of times compounded in a year. 
t = time in years 
Thus, if the interest was paid semiannually, n = 2. Paid quarterly would make n = 4, 
Paid monthly, n = 12, etc.
Sample Problems 
1) Find the value of a $1 if it is invested for 1 year at 10% interest compounded 
quarterly. 
Solution: Initial amount is $1 with r = .10, n = 4 and t = 1. 
A(1) = 1(1 + .10/4)4 = 1.1038 This means that at the end of a year, each 
dollar invested in worth 1.1038 or slightly more than $1.10. The effect of compounding 
adds another .0038 % to the interest rate. Thus the effective annual yield is 10.38%. 
2) You invest $5000 in an account paying 6% compounded quarterly for three years. 
How much will be in the account at the end of the time period? 
Solution: Initial amount is $5000, with r = .06, n = 4 and t = 3 
A(3) = 5000(1 + .06/4)12 = $5978.09. This account pays $978.09 in 
66 
interest over the three years. 
3) What is the effective annual yield on $1 invested for one year at 15% interest 
compounded monthly? 
Solution: Initial amount $1, with r = .15, n = 12 and t = 1 
A(1) = 1(1 + .15/12)12 = 1.1608. The effective annual yield is 16.08%. 
This is a relatively big increase because of the number of times compounded in the year. 
The above problems all had one thing in common. The number of times compounded 
was a finite number. We can also havecontinuous compounding. That is, compounding 
basically every second on the second. This would be rather cumbersome to calculate 
because the compounding is extremely large. We can use a similar formula if the 
compounding is continuous. 
P(t) = Poert 
Notice the appearance of the number e. If you look closely at the compound interest 
formula, you will see imbedded the definition of the number e. Only use this formula if 
you are sure the compounding is continuous. 
Problems 
1) Php. 500 is invested in an account paying 8% interest compounded continuously. 
They leave it in the account for 3 years. How much interest is accumulated? 
Solution: Initial amount Php 500, with r = .08 and t = 3. 
P(3) = 500(e.08(3)) = 635.62. This means the interest is Php.135.62. 
2) A population of insects rapidly increases so that the population after t days from now 
is given by A(t) = 5000e.02t . Answer the following questions: 
a) What is the initial population? 
b) How many will there be after a week? 
c) How many will there be after a month? (30 days) 
Solutions: 
a) The initial population is 5000 from the formula.
67 
b) A(7) = 5000e.14 = 5751 
c) A(30) = 5000e.6 = 9111 
Logarithmic Functions 
Common Logarithm 
Demo: Log Funtion Applet 
log x = a if and only if 10a = x 
The important thing to remember is the log represents the exponent. In the case of 
common logs, the base is always base 10. Study the following examples. 
1) log 100 = 2 because 102 = 100. 
2) log 1000 = 3 because 103 = 1000. 
3) log 1 = 0 because 100 = 1. 
4) log .1 = -1 because 10-1 = .1 
5) log .01 = -2 because 10-2 = .01 
The log function is the inverse function of the exponential function and as such their 
graphs are reflections about the y = x line. Here is the graph of the common log and the 
inverse. 
Some important facts you need to understand from the log graph. The domain of the log 
is x > 0. The range is all real numbers. The zero is at x = 1. You can only find the log of 
positive numbers. Logs of numbers less than one are negative and logs of numbers 
greater than one are positive. 
We can find the log of other bases by using the following formula similar to the common 
log definition. 
logb x = n if and only if x = bn. 
Here are some examples: 
1) log2 8 = 3 because 23 = 8 
2) log3 81 = 4 because 34 = 81. 
3) log4 1/16 = -2 because 4-2 = 1/16 
4) log8 1 = 0 because 80 = 1
One of the most important log function is called the natural log which has the number e 
as the base. When e is used as a base we use the following notation: 
ln x = a if and only if ea = x 
Most natural logs need to be calculated on your calculator. The graph of the natural log 
is shown below: 
68 
Solving Simple Log Equations 
1) Log x = 3 
Solution: To solve an equation of this type, rewrite the equation in exponential 
form. x = 103 
= 1000 
2) Log |x| = 2 
Solution: To solve an equation of this type, again rewrite the equation in 
exponential form and solve for x. 
|x| = 102 
= 100 
x = 100 or -100 
3) Log (x2 + 19) = 2 
Solution: Again, rewrite as an exponential equation and solve for x. 
x2 + 19 = 102 
x2 + 19 = 100 
x2 = 81 
x = 9 or -9 
4) Log x = .3 
Again, rewrite exponentially. 
x = 10.3 Use your calculator and round to hundredths. 
x = 2.00
69 
5) Ln x = -1.2 
Solution: Same as above. 
x = e-1.2 
x = .3 
Laws of Logarithms 
1) Logb MN = Logb M + Logb N 
2) Logb M/N = Logb M - Logb N 
3) Logb M = Logb N if and only if M = N 
4) Logb Mk = k Logb M 
5) Logb b = 1 
6) Logb 1 = 0 
7) Logb bk = k 
8) bLogb x = x 
Sample problems 
Write each log in expanded form. 
1) Log5 xy2 = 
Solution: Log5 x + Log5 y2 = Log5 x + 2 Log5 y 
2) Log7 (xy/z2) = 
Solution: Log7 x + Log7 y - 2 Log7 z 
3) 퐿표푔8√푥푦 = 퐿표푔8(푥푦) 
1 
2 = 1 
2 
(퐿표푔8푥 + 퐿표푔8푦) 
Express each as a single log. 
1) Log x + Log y - Log z = 
Solution: Log (xy)/z 
2) 2 Ln x + 3 Ln y = 
Solution: Ln x2y3 
3) (1/2) Ln x - (1/3) Ln y = 
Solution: 퐿푛 푥1/2 
푦1/3 = 퐿푛 √푥 
√푦 3 
Writing logs as single logs can be helpful in solving many log equations. 
1) Log2 (x + 1) + Log2 3 = 4 
Solution: First combine the logs as a single log. 
Log2 3(x + 1) = 4 
Now rewrite as an exponential equation. 
3(x + 1) = 24 
Now solve for x. 
3x + 3 = 16 
3x = 13 
x = 13/3 
Since this doesn't make the number inside the log zero or negative, the answer is 
acceptable.
70 
2) Log (x + 3) + Log x = 1 
Solution: Again, combine the logs as a single log. 
Log x(x + 3) = 1 
Rewrite as an exponential. 
x(x + 3) = 10 
Solve for x. 
x2 + 3x = 10 
x2 + 3x - 10 = 0 
(x + 5)(x - 2) = 0 
x = -5 or x = 2 
We have to throw out 5. Why? Because it makes (x + 3) negative and we can't 
take the log of a negative number. So the only answer is x = 2. 
3) Ln (x - 4) + Ln x = Ln 21 
Solution: Notice, this time we have a log on both sides. If we write the left 
side as a single log, we can use the rule that if the logs are equal, the quantity inside must 
be equal. 
Ln x(x - 4) = Ln 21 
Since the logs are equal, what is inside must be equal. 
x(x - 4) = 21 
Solve for x. 
x2 - 4x = 21 
x2 - 4x - 21 = 0 
(x - 7)(x + 3) = 0 
x = 7 or x = -3 
Again, we need to throw out one of the answers because it makes both quantities 
negative. Throw out -3 and keep 7. Thus, the answer is x = 7. 
Simplify each log . 
1) ln e5 
Solution: This is rule number 7. The answer is 5! 
2) Log 10-3 
Solution: This is again rule #7. The answer: -3 (This answers the question: 
what power do you raise 10 to get 10 to the third? 
3) eln 7 
Solution: This is rule #8. The answer is 7. 
4) e2ln 5 
Solution: We can use rule #8 as soon as we simplify the problem. Rewrite as: 
eln 25 = 25 The 25 came from 52. 
5) 10Log 6 
Solution: Rule #8 again. Answer: 6
71 
6) 102 + log 5 
Solution: We need to simplify before we can apply one of the rules. Rewrite 
as: (102)(10log 5) Adding exponents means you are multiplying the bases. 
= 100(5) Use rule #8 again. 
= 50 
Change of Base Formula 
An exponential equation is an equation that contains a variable in the exponent. We 
solved problems of this type in a previous chapter by putting the problem into the same 
base. Unfortunately, it is not always possible to do this. Take for example, the equation 
2x = 17. We cannot put this equation in the same base. So, how do we solve the 
problem? We use thechange of base formula!! We can change any base to a different 
base any time we want. The most used bases are obviously base 10 and base e because 
they are the only bases that appear on your calculator!! 
Change of base formula 
Logb x = Loga x/Loga b 
Pick a new base and the formula says it is equal to the log of the number in the new base 
divided by the log of the old base in the new base. 
Examples 
1) Log2 37 = 
Solution: Change to base 10 and use your calculator. 
= Log 37/log 2 
Now use your calculator and round to hundredths. 
= 5.21 
This seems reasonable, as the log2 32 = 5 and log2 64 = 6. 
2) Log7 99 = 
Solution: Change to either base 10 or base e. Both will give you the same 
answer. Try it both ways and see. 
= Log 99/Log 7 or Ln 99/ Ln 7 
Use your calculator on both of the above and prove to yourself that you 
get the same answer. Both ways give you 2.36. 
Solving Exponential Equations using change of base 
Now, let's go back up and try the original equation: 
2x = 17 
To put these in the same base, take the log of both sides. Either in base 10 or base e. 
Hint. Use base e only if the problem contains e. 
Log 2x = Log 17 
Using the log rules, we can write as: 
x Log 2 = Log 17 
Now isolate for x and use your calculator. 
x = Log 17/Log 2 
x = 4.09
To check your answer, type in 24.09 and see what you get! The answer will come out 
slightly larger than 17 do to rounding. 
72 
Sample Problems 
1) e3x = 23 
Solution: Use natural log this time. 
Ln e3x = Ln 23 
3x Ln e = Ln 23 
3x = Ln 23 ( Ln e = 1) 
x = (Ln 23)/3 
x = 1.05 
2) How long does it take $100 to become $1000 if invested at 10% compounded 
quarterly? 
Solution: Ao = 100, A(t) = 1000, r = .1, n = 4 
1000 = 100(1 + .1/4)4t 
10 = 1.0254t 
Use the change of base formula 
Log 10 = Log 1.0254t 
1 = 4t Log 1.025 (Log 10 = 1) 
1/(4Log 1.025) = t 
t = 23.31 
It will take 23.3 years to have $1000 from the $100 
investment.
ASSESSMENT 
73 
1) Simplify each: 
a) 4 . 2-4 = b) (4 . 2)-4 = c) (a-1 + b-1)-2 = 
2) Simplify each: 
3) Solve the equation: 93x = 81x + 1 
4) The half life of a radioactive isotope is 7 days. If 5.6 kg are present now, how 
much will be present in t days? In 13 days? 
5) A bacteria colony triples every 6 days. The population currently is 5000 
bacteria. What will be the population in 21 days? 
6) $1000 is invested at 8% compounded quarterly for 3 years. How much interest 
will you receive at the end of the time period? 
7) You invest $3000 at 7% compounded continuously for 2 years. How much 
money will be in the account at the end of the time period? 
8) Solve each log equation: 
a) Log x = 21 
b) Log |x| = 15 
c) Ln (x2 - 1) = 3 
d) Log x = 1.6 (Use calculator and round to hundredths.) 
9) Write each log as a single log: 
a) Log x + log y + 2Log z 
b) Ln x + Ln y - 3Ln z 
10) Simplify each log:
74 
a) Ln e5 = 
b) 102log4 = 
c) 101 + log 5 = 
11) Solve the equation: Log (x + 2) + Log 5 = 4 
12) Graph y = 3x and y = log3 x on the same axis. 
13) Use a calculator to find Log7 58 and round to hundredths place. 
14) solve the equation: 15x + 1 = 29 and round to hundredths place. 
15) How long will it take to double $1000 if invested at 6% compounded monthly?
Chapter VI 
System of Equations 
And Inequalities 
75 
.
SYSTEMS OF LINEAR EQUATIONS IN TWO VARIABLES 
Many applications of mathematics lead to more than one equation in several variables. 
The resulting equations are called a system of equations. The solution set of a system of 
equations consists of all solutions that are common to the equation in the system. 
2푥 + 푦 = 3 
5푥 + 3푦 = 10 
76 
ax + by = c 
We proved that the graph of an equation of the form is a line and that all ordered pairs (x 
, y) satisfying the equation are coordinates of points in the line. A system of two linear 
equations in two variables x and y can be written as. 
{a1x + b1y = c1} 
{a2x + b2y = c2} 
Where a1,b1, c1, a2, b2,and c2 are real numbers. The left brace is used to indicate that 
the two equations form a system. If an ordered pair (x , y) is to satisfy the system of two 
linear equations, the corresponding points (x , y) must lie on the two lines that are the 
graph of the equations. 
ILLUSTRATION 1 
A particular system of two linear equations is. 
{ 
The solution set of each of the equations in the system is an infinite set of ordered pairs of 
real numbers, and the graphs of these sets are lines. Recall that to draw a sketch of a line 
we need to find two points on the line; usually we plot the points where the line intersects 
the coordinate axis. On the line 2x + y = 3 we have the points (3/2 , 0) and (0,3) while on 
the line 5x + 3y = 10 we have the points (2,0) and(0, 10/3). Shows on the same 
coordinate system sketches of two lines. It is apparent that two lines intersect at exactly 
one point. This point, (-1 , 5) can be verified by substituting into the equations as follows: 
2(-1) + 5 = 3 
5(-1) + 3(5) = 10 
The only ordered pair that is common to the solution sets of the two equations is (-1,5). 
Hence the solution set of the system is {(-1,5)}. 
ILLUSTRATION 2 
Consider the system 
6푥 − 3푦 = 5 
2푥 − 푦 = 4 
{
The lines having these equations appear to be parallel. It can easily to be proved that the 
lines are indeed parallel by writing each of the equation, we have 
6x - 3y = 5 2x – y = 4 
-3y = -6x + 5 -y = -2x + 4 
y = 2x – 5/3 y = 2x – 4 
2푥 + 푦 = 3 
5푥 + 3푦 = 10 
푦 = 3 − 2푥 
5푥 + 3푦 = 10 
77 
Example 1 
Use the substitution method to find the solution set of the system. 
Illustration 1: { 
Solution: 
We solve the first equation for y and get the equivalent system 
{ 
We replace y in the second equation by its equal, 3 – 2 x, from the first equation. We then 
have the equivalent system 
{ 
푦 = 3 − 2푥 
5푥 + 3(3 − 2푥 ) = 10 
Simplifying the second equation, we have 
{ 
푦 = 3 − 2푥 
−푥 + 9 = 10 
Solving the second equation for x, we get 
{ 
푦 = 3 − 2푥 
푥 = −1 
Finally, we substitute the value of x from the second equation into the first equation and 
we have 
{ 
푦 = 5 
푥 = −1 
This system is equivalent to the given one. Hence the solution set is ( -1 , 5)
78 
Example 2 
Use the elimination method to find the solution set of the system of equations in Example 
1. 
2푥 + 푦 = 3 
5푥 + 3푦 = 10 
{ 
Remember that our goal is to eliminate one of the variables. Observe that the coefficient 
of y is 1 in the first equation and 3 in the second equation. To obtain an equation not 
involving y, we therefore replace the second equation by the sum of the second equation 
and -3 times the first. We begin by multiplying the first equation by -3 and writing the 
equivalent system. 
{ 
−6 − 3푦 = −9 
5푥 + 3푦 = 10 
Adding the equations given the following computations: 
−6−3푦 = −9 
5푥 +3푦 =10 
−푥 =1 
With this equation and the first equations in the given system, we can write the following 
equivalent system 
{ 
2푥 + 푦 = 3 
−푥 = 1 
If we now multiply both sides of the second equation by -1, we have the equivalent 
system 
{ 
2푥 + 푦 = 3 
푥 = −1 
We next substitute -1 for x in the first equation to obtain 
{ 
2(−1) + 푦 = 3 
푥 = −1 
{ 
푦 = 5 
푥 = −1 
SYSTEMS OF LINEAR EQUATIONS IN THREE VARIABLES 
So far the linear (first degree) equations we have discussed have contained at most two 
variables. In this section we introduce systems of linear equations in three variables. 
Consider the equation 
2푥 − 푦 + 4푧 = 10 
For which the replacement set of each of the three variables x, y and z is the set R of real 
numbers. This equation is linear in the three variables. A solution of a linear equation in 
the three variables x, y and z is the ordered triple of real numbers (r,s,z) such that if x is 
replaced by r, y by s, and z by t, the resulting statement is true. The set of all solutions is 
the solution set of the equation
79 
Illustration 1 
For the equation 
2푥 − 푦 + 4푧 = 10 
The ordered triple pair (3,4,2) is a solution because 
2(3) – 4 + 4 (2) = 10 
Some other ordered triples that satisfy this equation are (-1, 8, 5) , (2, -6, 0), (1,0,2), 
(5,0,0) , (0,-6,1) , (8, 2 1) ,and (7,2 - ½). It appears that the solution set is infinite. 
The graph of an equation in three variables is a set of points represented by ordered 
triples of real numbers. Such points appear in a three dimensional coordinate system, 
which we do not discuss. You should, however, be aware that the graph of a linear 
equation in a three variables is a plane. 
Suppose that we have the following system of linear equations in the variables x, y and z. 
{ 
푎1푥 + 푏1푦 + 푐1푧 = 푑1 
푎2푥 + 푏2푦 + 푐2푧 = 푑2 
푎3푥 + 푏3푦 + 푐3푧 = 푑3 
The solution set of this system is the intersection of the solution sets of the three 
equations. Because the graph of each equation is a plane, the solution set can be 
interpreted geometrically as the intersection of three planes. When this intersection 
consist and independent. 
Algebraic methods for finding the solution set of a system of three linear equations in 
three variables are analogous to those used to solve linear systems in two variables. The 
following examples shows the substitution method. 
Example Find the solution set of the system 
푥 − 푦 − 4푧 = 3 
2푥 − 3푦 + 2푧 = 0 
2푥 − 푦 + 2푧 = 2 
{ 
Solution, we solve the first set of the system 
푥 = 푦 + 4푧 + 3 
2푥 − 3푦 + 2푧 = 0 
2푥 − 푦 + 2푧 = 2 
{
We now substitute the value of x from the first equation into the other two equations , and 
we obtain the equivalent system 
푥 = 푦 + 4푧 + 3 
−푦 + 10푧 = −6 
푦 + 10푧 = −4 
푥 = 푦 + 4푧 + 3 
푦 = 10푧 + 6 
푥 = 푦 + 4푧 + 3 
80 
{ 
푥 = 푦 + 4푧 + 3 
2(푦 + 4푧 + 3) − 3푦 + 2푧 = 0 
2(푦 + 4푧 + 3) − 푦 + 2푧 = 2 
{ 
We next solve the second equation for y and get 
푥 = 푦 + 4푧 + 3 
푦 = 10푧 + 6 
푦 + 10푧 = −4 
Substituting the value of y from the second equation into the third gives the equivalent 
system. 
{ 
푥 = 푦 + 4푧 + 3 
푦 = 10푧 + 6 
(10푧 + 6 +) + 10 = −4 
푥 = 푦 + 4푧 + 3 
푦 = 10푧 + 6 
20푧 = −10 
{ 
{ 
푧 = −1/2 
Substituting the value of z from the third equation into the second equation, we obtain 
{ 
푦 = 1 
푧 = −1/2 
Substituting the values of y and z from the second and third equations into the first 
equation, we get 
푥 = 2 
푦 = 1 
푧 = 1/2 
{
The latter system is equivalent to the given system. Hence the solution set of the given 
system is (2,1 ,1/2).The solution can be checked by substituting into each of the given 
equations. Doing this we have 
81 
2 − 1 + 2 = 3 
4 − 3 − 1 = 0 
4 − 1 − 1 = 2 
{ 
The equations of the given system are consistent and independent. 
Exercise 
1.) { 
4푥 + 3푦 + 푧 = 15 
푥 − 푦 − 2푧 = 2 
2푥 − 2푦 + 푧 = 4 
2.) { 
2푥 + 3푦 + 푧 = 8 
5푥 + 2푦 + 3푧 = −13 
푥 − 2푦 + 5푧 = 15 
3.) { 
푥 − 푦 + 3푧 = 2 
2푥 + 2푦 − 푧 = 5 
5푥 + 2푧 = 7 
3푥 + 2푦 − 푧 = 4 
3푥 + 푦 + 3푧 = −2 
6푥 − 3푦 − 2푧 = −6 
4.) { 
5.) { 
2푥 − 3푦 − 5푧 = 4 
푥 + 7푦 + 6푧 = −7 
7푥 + 2푦 − 9푧 = 6 
3푥 − 2푦 + 4푧 = 4 
7푥 − 5푦 − 푧 = 9 
푥 + 9푦 − 9푧 = 1 
6.) { 
7.) { 
3푥 − 5푦 + 2푧 = −2 
2푥 + 3푧 = −3 
4푦 − 3푧 = 8 
푥 − 푦 = 2 
3푦 + 푧 = 1 
푥 − 2푧 = 7 
8.) { 
9.) { 
3푥 − 2푦 = 1 
푧 − 푦 = 5 
푧 − 2푥 = 5 
푥 − 푦 + 5푧 = 2 
4푥 − 3푦 + 5푧 = 3 
3푥 − 2푦 + 4푧 = 1 
10.) { 
SYSTEMS INVOLVING QUADRATIC EQUATIONS 
In sections 6.1 and 6.2 our discussions of systems of equations was confined to linear 
systems. However, a number of applications lead to nonlinear systems as illustrated in 
exercises 25 through 36. The word problems in these exercises use concepts presented 
previously, but the resulting systems involve at least one quadratic equation. In this 
section we discussed methods of solving such systems of two equations in two variables. 
We consider first a system that contains a linear equation and quadratic equation. In this 
case the system can be solved for one variable in terms of the other, and the resulting 
expression can be substituted into the quadratic equation, as shown in the following 
example.
푦2 = 4(3 − 푦) 
푥 = 3 − 푦 
푦2 + 4푦 − 12 = 0 
푦 = −6 
푥 = 3 − 푦 
82 
Example 1 
Find the solution set of the system. 
푦2 = 4푥 
푥 + 푦 = 3 
{ 
Solution 
We solve the second equation for x and obtain the equivalent system. 
푦2 = 4푥 
푥 = 3 − 푦 
{ 
Replacing x in the first equation by its equal from the second, we have the equivalent 
system 
{ 
{ 
푥 = 3 − 푦 
We now solve the first equation. 
(푦 − 2)(푦 + 6) = 0 
푦 − 2 = 0 푦 + 6 = 0 
y= 2 푦 = −6 
Because the first equation of system (II) is equivalent to the equations 푦 = 2 and 푦 = 
−6, system (II) is equivalent to the systems 
푦 = 2 
푥 = 3 − 푦 
{ 
and { 
In each of the latter two systems we have substitute into the second equation the value of 
y from the first , and we have 
{ 
푦 = 2 
푥 = 1 
and { 
푦 = −6 
푥 = 9 
These two systems are equivalent to system (I). Thus, the solution set of (I) is (1,2) , (9,- 
6).
SYSTEMS OF LINEAR INEQUALITIES AND INTRODUCTION TO LINEAR 
PROGRAMMING 
Systems of linear inequalities are important in economics, business, statistics, science, 
engineering, and other fields. With electronic computers performing most of the 
computation, large numbers of inequalities with many unknowns are usually involved. In 
this section we briefly discuss how to solve system of linear inequalities. We then give an 
introduction to linear programming, a related approach to decision making problems. 
83 
Statement of the form 
퐴푥 + 퐵푦 + 퐶 > 0 퐴푥 + 퐵푦 + 퐶 < 0 
퐴푥 + 퐵푦 + 퐶 ≥ 0 퐴푥 + 퐵푦 + 퐶 ≤ 0 
Where A,B and C are constants, A and B are not both zero, are inequalities of first degree 
in two variables. By the graph of such an inequality, we mean the (x, y) in the rectangular 
Cartesian coordinate system for which (x, y) is an ordered pair satisfying the inequality. 
Every line in a plane divides the plane into two regions, one on each side of the line. Each 
of these regions is called a half plane. The graphs of inequalities of the forms. 
퐴푥 + 퐵푦 + 퐶 > 0 and 퐴푥 + 퐵푦 + 퐶 < 0 
Are half planes. We shall show this for the particular inequalities 
2푥 − 푦 − 4 > 0 푎푛푑 퐴푥 + 퐵푦 + 퐶 < 0 
Let L be the line having the equation 2푥 − 푦 − 4 = 0. If we solve this equation for y, we 
obtain 푦 = 2푥 − 4. If (x, y) is any point in the plane, exactly one of the following 
statements holds: 
푦 = 2푥 − 4 푦 > 2푥 − 4 푦 < 
2푥 − 4 
Now, 푦 > 2푥 − 4 if and only if the point (x, y) is any point (x, 2x - 4) on line L; 
Furthermore, 푦 < 2푥 − 4 if and only if the point (x, y) is below the point (x, 2x - 4) on L; 
therefore the line L divides the plane into two regions. One region is the half plane above 
L, which is the graph of inequality 푦 > 2푥 − 4, and the other region is the half plane 
above L, which is the graph of the inequalities 푦 > 2푥 − 4, and the region is the half 
plane below L, which is the graph of inequality 푦 < 2푥 − 4. A similar discussion holds 
for any line L having an equation of the form 퐴푥 + 퐵푦 + 퐶 = 0 푤ℎ푒푟푒 퐵 ≠ 0.
If B= 0, an equation of line L is 퐴푥 + 퐶 = 0, and L is a vertical line whose 
equation 푥 = 4. Then if (x, y) is any point in the plane, exactly one of the following 
statements is true: 
The point (x, y) is to the right of the point (4, y) if and only if x >4. Showing the graph of 
inequality x >4 as the half plane lying to the right of the line x = 4. Similarly, the graph 
of x < 4 if, and only if the point (x, y) is to the left of the point (4, y). The discussion can 
be extended to any line having an equation of the form Ax +퐶 = 0. 
By generalizing the above arguments to any line, we can prove this theorem. 
84 
THEOREM 
(I) the graph of y> 푚푥 + 푏 is the half plane lying above the line y= 푚푥 + 푏. 
(II) the graph of y< 푚푥 + 푏 is the half plane lying below the line y= 푚푥 + 푏. 
(III) the graph of (y< 푚푥 + 푏) x> 푎 is the half plane lying to the right of line x= 푎. 
(IV) The graph of x< 푎 is the half plane lying to the left of the line x= 푎. 
Example 1 
Draw a sketch of the graph of the inequality 
2푥 − 4푦 + 5 > 0 
Solution 
The given inequality is equivalent to 
−4푦 > −2푥 − 5 
푦 > 1 
2 
+ 5/4 
The graph of inequality is the half plane below the line having the equation 푦 = 1/2푥 + 
5/4. A sketch of this graph is the shaded half plane. 
A closed half plain is a half plane together with the line bounding it is the graph of an 
inequality of the form. 
퐴푥 + 퐵푦 + 퐶 = 0 표푟 퐴푥 + 퐵푦 + 퐶 ≤ 0
85 
Illustration 
The inequality 
4푥 + 5푦 − 20 ≥ 0 
Is equivalent to 
5푦 ≥ −4푥 + 20 
푦 ≥ −4/5푥 + 4 
Therefore the graph of this inequality is the closed half plane consisting of the line 푦 = 
−3/5푥 + 4 and the half plane above it. A sketch of the graph. 
Two intersecting lines divide the points of the plane into four regions. Each of these 
regions is the intersection of two half planes and is defined by a system of two linear 
inequalities.
Chapter 1
Chapter 1
Chapter 1
Chapter 1
Chapter 1
Chapter 1
Chapter 1
Chapter 1
Chapter 1
Chapter 1
Chapter 1
Chapter 1
Chapter 1
Chapter 1

Mais conteúdo relacionado

Mais procurados

5.1 anti derivatives
5.1 anti derivatives5.1 anti derivatives
5.1 anti derivativesmath265
 
Differential equations of first order
Differential equations of first orderDifferential equations of first order
Differential equations of first orderUzair Saiyed
 
Linear inequalities in two variables
Linear inequalities in two variablesLinear inequalities in two variables
Linear inequalities in two variableschristianjustine
 
Complex Numbers 1 - Math Academy - JC H2 maths A levels
Complex Numbers 1 - Math Academy - JC H2 maths A levelsComplex Numbers 1 - Math Academy - JC H2 maths A levels
Complex Numbers 1 - Math Academy - JC H2 maths A levelsMath Academy Singapore
 
Integrating factors found by inspection
Integrating factors found by inspectionIntegrating factors found by inspection
Integrating factors found by inspectionShin Kaname
 
Parallel and Perpendicular Slopes lines
Parallel and Perpendicular Slopes linesParallel and Perpendicular Slopes lines
Parallel and Perpendicular Slopes linesswartzje
 
First Order Differential Equations
First Order Differential EquationsFirst Order Differential Equations
First Order Differential EquationsItishree Dash
 
Limit of Function And Its Types
Limit of Function And Its TypesLimit of Function And Its Types
Limit of Function And Its TypesAdeel Rasheed
 
Lesson 25: Evaluating Definite Integrals (slides)
Lesson 25: Evaluating Definite Integrals (slides)Lesson 25: Evaluating Definite Integrals (slides)
Lesson 25: Evaluating Definite Integrals (slides)Matthew Leingang
 
Graphs of Log functions
Graphs of Log functionsGraphs of Log functions
Graphs of Log functionslesurhommemega
 
Analytic geometry lecture1
Analytic geometry lecture1Analytic geometry lecture1
Analytic geometry lecture1admercano101
 
5 3 Direct Variation
5 3 Direct Variation5 3 Direct Variation
5 3 Direct VariationBitsy Griffin
 
Introduction to Polynomial Functions
Introduction to Polynomial FunctionsIntroduction to Polynomial Functions
Introduction to Polynomial Functionskshoskey
 
The binomial expansion
The binomial expansionThe binomial expansion
The binomial expansionJJkedst
 
Engineering Numerical Analysis Lecture-1
Engineering Numerical Analysis Lecture-1Engineering Numerical Analysis Lecture-1
Engineering Numerical Analysis Lecture-1Muhammad Waqas
 

Mais procurados (20)

5.1 anti derivatives
5.1 anti derivatives5.1 anti derivatives
5.1 anti derivatives
 
Alternating Group presentation
Alternating Group presentationAlternating Group presentation
Alternating Group presentation
 
Differential equations of first order
Differential equations of first orderDifferential equations of first order
Differential equations of first order
 
Linear inequalities in two variables
Linear inequalities in two variablesLinear inequalities in two variables
Linear inequalities in two variables
 
Complex Numbers 1 - Math Academy - JC H2 maths A levels
Complex Numbers 1 - Math Academy - JC H2 maths A levelsComplex Numbers 1 - Math Academy - JC H2 maths A levels
Complex Numbers 1 - Math Academy - JC H2 maths A levels
 
Integrating factors found by inspection
Integrating factors found by inspectionIntegrating factors found by inspection
Integrating factors found by inspection
 
Parallel and Perpendicular Slopes lines
Parallel and Perpendicular Slopes linesParallel and Perpendicular Slopes lines
Parallel and Perpendicular Slopes lines
 
First Order Differential Equations
First Order Differential EquationsFirst Order Differential Equations
First Order Differential Equations
 
Math1.2
Math1.2Math1.2
Math1.2
 
Limit of Function And Its Types
Limit of Function And Its TypesLimit of Function And Its Types
Limit of Function And Its Types
 
Linear equations
Linear equationsLinear equations
Linear equations
 
Lesson 25: Evaluating Definite Integrals (slides)
Lesson 25: Evaluating Definite Integrals (slides)Lesson 25: Evaluating Definite Integrals (slides)
Lesson 25: Evaluating Definite Integrals (slides)
 
Composite functions
Composite functionsComposite functions
Composite functions
 
Graphs of Log functions
Graphs of Log functionsGraphs of Log functions
Graphs of Log functions
 
Analytic geometry lecture1
Analytic geometry lecture1Analytic geometry lecture1
Analytic geometry lecture1
 
5 3 Direct Variation
5 3 Direct Variation5 3 Direct Variation
5 3 Direct Variation
 
Introduction to Polynomial Functions
Introduction to Polynomial FunctionsIntroduction to Polynomial Functions
Introduction to Polynomial Functions
 
Interpolation Methods
Interpolation MethodsInterpolation Methods
Interpolation Methods
 
The binomial expansion
The binomial expansionThe binomial expansion
The binomial expansion
 
Engineering Numerical Analysis Lecture-1
Engineering Numerical Analysis Lecture-1Engineering Numerical Analysis Lecture-1
Engineering Numerical Analysis Lecture-1
 

Semelhante a Chapter 1

Chapter 1 review topic in algebra 1
Chapter 1 review topic in algebra 1Chapter 1 review topic in algebra 1
Chapter 1 review topic in algebra 1jennytuazon01630
 
Expansion and Factorisation of Algebraic Expressions 2.pptx
Expansion and Factorisation of Algebraic Expressions  2.pptxExpansion and Factorisation of Algebraic Expressions  2.pptx
Expansion and Factorisation of Algebraic Expressions 2.pptxMitaDurenSawit
 
Factoring polynomials
Factoring polynomialsFactoring polynomials
Factoring polynomialsMark Ryder
 
ملزمة الرياضيات للصف السادس الاحيائي الفصل الاول
ملزمة الرياضيات للصف السادس الاحيائي الفصل الاولملزمة الرياضيات للصف السادس الاحيائي الفصل الاول
ملزمة الرياضيات للصف السادس الاحيائي الفصل الاولanasKhalaf4
 
ملزمة الرياضيات للصف السادس التطبيقي الفصل الاول الاعداد المركبة 2022
 ملزمة الرياضيات للصف السادس التطبيقي الفصل الاول الاعداد المركبة 2022 ملزمة الرياضيات للصف السادس التطبيقي الفصل الاول الاعداد المركبة 2022
ملزمة الرياضيات للصف السادس التطبيقي الفصل الاول الاعداد المركبة 2022anasKhalaf4
 
MATHS - Linear equation in two variable (Class - X) Maharashtra Board
MATHS - Linear equation in two variable (Class - X) Maharashtra BoardMATHS - Linear equation in two variable (Class - X) Maharashtra Board
MATHS - Linear equation in two variable (Class - X) Maharashtra BoardPooja M
 
Mathnasium Presentation (1)
Mathnasium Presentation (1)Mathnasium Presentation (1)
Mathnasium Presentation (1)Muhammad Arslan
 
maths_formula_sheet.pdf
maths_formula_sheet.pdfmaths_formula_sheet.pdf
maths_formula_sheet.pdfVanhoaTran2
 
Algebra 2 Section 4-6
Algebra 2 Section 4-6Algebra 2 Section 4-6
Algebra 2 Section 4-6Jimbo Lamb
 
101 math short cuts [www.onlinebcs.com]
101 math short cuts [www.onlinebcs.com]101 math short cuts [www.onlinebcs.com]
101 math short cuts [www.onlinebcs.com]Itmona
 
Module 2 exponential functions
Module 2   exponential functionsModule 2   exponential functions
Module 2 exponential functionsdionesioable
 
Advanced algebra
Advanced algebraAdvanced algebra
Advanced algebraspark21
 
6 polynomial expressions and operations
6 polynomial expressions and operations6 polynomial expressions and operations
6 polynomial expressions and operationselem-alg-sample
 
46polynomial expressions
46polynomial expressions46polynomial expressions
46polynomial expressionsalg1testreview
 
Unit 5 powerpoint[1] algebra (1)
Unit 5 powerpoint[1] algebra (1)Unit 5 powerpoint[1] algebra (1)
Unit 5 powerpoint[1] algebra (1)John O'Driscoll
 

Semelhante a Chapter 1 (20)

Chapter 1 review topic in algebra 1
Chapter 1 review topic in algebra 1Chapter 1 review topic in algebra 1
Chapter 1 review topic in algebra 1
 
Annie
AnnieAnnie
Annie
 
Em01 ba
Em01 baEm01 ba
Em01 ba
 
Expansion and Factorisation of Algebraic Expressions 2.pptx
Expansion and Factorisation of Algebraic Expressions  2.pptxExpansion and Factorisation of Algebraic Expressions  2.pptx
Expansion and Factorisation of Algebraic Expressions 2.pptx
 
Factoring polynomials
Factoring polynomialsFactoring polynomials
Factoring polynomials
 
ملزمة الرياضيات للصف السادس الاحيائي الفصل الاول
ملزمة الرياضيات للصف السادس الاحيائي الفصل الاولملزمة الرياضيات للصف السادس الاحيائي الفصل الاول
ملزمة الرياضيات للصف السادس الاحيائي الفصل الاول
 
ملزمة الرياضيات للصف السادس التطبيقي الفصل الاول الاعداد المركبة 2022
 ملزمة الرياضيات للصف السادس التطبيقي الفصل الاول الاعداد المركبة 2022 ملزمة الرياضيات للصف السادس التطبيقي الفصل الاول الاعداد المركبة 2022
ملزمة الرياضيات للصف السادس التطبيقي الفصل الاول الاعداد المركبة 2022
 
MATHS - Linear equation in two variable (Class - X) Maharashtra Board
MATHS - Linear equation in two variable (Class - X) Maharashtra BoardMATHS - Linear equation in two variable (Class - X) Maharashtra Board
MATHS - Linear equation in two variable (Class - X) Maharashtra Board
 
Appt and reasoning
Appt and reasoningAppt and reasoning
Appt and reasoning
 
MATRICES-MATHED204.pptx
MATRICES-MATHED204.pptxMATRICES-MATHED204.pptx
MATRICES-MATHED204.pptx
 
Mathnasium Presentation (1)
Mathnasium Presentation (1)Mathnasium Presentation (1)
Mathnasium Presentation (1)
 
maths_formula_sheet.pdf
maths_formula_sheet.pdfmaths_formula_sheet.pdf
maths_formula_sheet.pdf
 
Algebra 2 Section 4-6
Algebra 2 Section 4-6Algebra 2 Section 4-6
Algebra 2 Section 4-6
 
101 math short cuts [www.onlinebcs.com]
101 math short cuts [www.onlinebcs.com]101 math short cuts [www.onlinebcs.com]
101 math short cuts [www.onlinebcs.com]
 
Module 2 exponential functions
Module 2   exponential functionsModule 2   exponential functions
Module 2 exponential functions
 
Advanced algebra
Advanced algebraAdvanced algebra
Advanced algebra
 
Algebra 6
Algebra 6Algebra 6
Algebra 6
 
6 polynomial expressions and operations
6 polynomial expressions and operations6 polynomial expressions and operations
6 polynomial expressions and operations
 
46polynomial expressions
46polynomial expressions46polynomial expressions
46polynomial expressions
 
Unit 5 powerpoint[1] algebra (1)
Unit 5 powerpoint[1] algebra (1)Unit 5 powerpoint[1] algebra (1)
Unit 5 powerpoint[1] algebra (1)
 

Mais de jennytuazon01630

Mais de jennytuazon01630 (20)

COMBINED VARIATION.pptx
COMBINED VARIATION.pptxCOMBINED VARIATION.pptx
COMBINED VARIATION.pptx
 
Grade 7 Mathematics Week 4 2nd Quarter
Grade 7 Mathematics Week 4 2nd QuarterGrade 7 Mathematics Week 4 2nd Quarter
Grade 7 Mathematics Week 4 2nd Quarter
 
Adding similar fractions
Adding similar fractionsAdding similar fractions
Adding similar fractions
 
Oragnization and management
Oragnization and management   Oragnization and management
Oragnization and management
 
INTRODUCTION TO ALGEBRA
INTRODUCTION TO ALGEBRAINTRODUCTION TO ALGEBRA
INTRODUCTION TO ALGEBRA
 
Integers
IntegersIntegers
Integers
 
Cell organelles
Cell organellesCell organelles
Cell organelles
 
measurements
measurementsmeasurements
measurements
 
Properties of real numbers
Properties of real numbersProperties of real numbers
Properties of real numbers
 
Learning the principle of subsidiarity
Learning the principle of subsidiarityLearning the principle of subsidiarity
Learning the principle of subsidiarity
 
Order of operations
Order of operationsOrder of operations
Order of operations
 
Divisibility rules 2 to 12
Divisibility rules 2 to 12Divisibility rules 2 to 12
Divisibility rules 2 to 12
 
Addition and Subtraction of whole numbers
Addition and Subtraction of whole numbersAddition and Subtraction of whole numbers
Addition and Subtraction of whole numbers
 
Terms used in statistics july 7
Terms used in statistics  july 7Terms used in statistics  july 7
Terms used in statistics july 7
 
Introduction to statistics
Introduction to statisticsIntroduction to statistics
Introduction to statistics
 
Graphical presentation of data
Graphical presentation of dataGraphical presentation of data
Graphical presentation of data
 
Measures of variability
Measures of variabilityMeasures of variability
Measures of variability
 
Experimental prob grand_demo_ppt2 (2)
Experimental prob grand_demo_ppt2 (2)Experimental prob grand_demo_ppt2 (2)
Experimental prob grand_demo_ppt2 (2)
 
Parts of a circle
Parts of a circleParts of a circle
Parts of a circle
 
Symbolic logic worksheet 3
Symbolic logic worksheet 3Symbolic logic worksheet 3
Symbolic logic worksheet 3
 

Último

4.11.24 Mass Incarceration and the New Jim Crow.pptx
4.11.24 Mass Incarceration and the New Jim Crow.pptx4.11.24 Mass Incarceration and the New Jim Crow.pptx
4.11.24 Mass Incarceration and the New Jim Crow.pptxmary850239
 
Blowin' in the Wind of Caste_ Bob Dylan's Song as a Catalyst for Social Justi...
Blowin' in the Wind of Caste_ Bob Dylan's Song as a Catalyst for Social Justi...Blowin' in the Wind of Caste_ Bob Dylan's Song as a Catalyst for Social Justi...
Blowin' in the Wind of Caste_ Bob Dylan's Song as a Catalyst for Social Justi...DhatriParmar
 
4.16.24 Poverty and Precarity--Desmond.pptx
4.16.24 Poverty and Precarity--Desmond.pptx4.16.24 Poverty and Precarity--Desmond.pptx
4.16.24 Poverty and Precarity--Desmond.pptxmary850239
 
Active Learning Strategies (in short ALS).pdf
Active Learning Strategies (in short ALS).pdfActive Learning Strategies (in short ALS).pdf
Active Learning Strategies (in short ALS).pdfPatidar M
 
How to Fix XML SyntaxError in Odoo the 17
How to Fix XML SyntaxError in Odoo the 17How to Fix XML SyntaxError in Odoo the 17
How to Fix XML SyntaxError in Odoo the 17Celine George
 
4.9.24 School Desegregation in Boston.pptx
4.9.24 School Desegregation in Boston.pptx4.9.24 School Desegregation in Boston.pptx
4.9.24 School Desegregation in Boston.pptxmary850239
 
CHEST Proprioceptive neuromuscular facilitation.pptx
CHEST Proprioceptive neuromuscular facilitation.pptxCHEST Proprioceptive neuromuscular facilitation.pptx
CHEST Proprioceptive neuromuscular facilitation.pptxAneriPatwari
 
Daily Lesson Plan in Mathematics Quarter 4
Daily Lesson Plan in Mathematics Quarter 4Daily Lesson Plan in Mathematics Quarter 4
Daily Lesson Plan in Mathematics Quarter 4JOYLYNSAMANIEGO
 
31 ĐỀ THI THỬ VÀO LỚP 10 - TIẾNG ANH - FORM MỚI 2025 - 40 CÂU HỎI - BÙI VĂN V...
31 ĐỀ THI THỬ VÀO LỚP 10 - TIẾNG ANH - FORM MỚI 2025 - 40 CÂU HỎI - BÙI VĂN V...31 ĐỀ THI THỬ VÀO LỚP 10 - TIẾNG ANH - FORM MỚI 2025 - 40 CÂU HỎI - BÙI VĂN V...
31 ĐỀ THI THỬ VÀO LỚP 10 - TIẾNG ANH - FORM MỚI 2025 - 40 CÂU HỎI - BÙI VĂN V...Nguyen Thanh Tu Collection
 
Expanded definition: technical and operational
Expanded definition: technical and operationalExpanded definition: technical and operational
Expanded definition: technical and operationalssuser3e220a
 
Congestive Cardiac Failure..presentation
Congestive Cardiac Failure..presentationCongestive Cardiac Failure..presentation
Congestive Cardiac Failure..presentationdeepaannamalai16
 
MS4 level being good citizen -imperative- (1) (1).pdf
MS4 level   being good citizen -imperative- (1) (1).pdfMS4 level   being good citizen -imperative- (1) (1).pdf
MS4 level being good citizen -imperative- (1) (1).pdfMr Bounab Samir
 
Concurrency Control in Database Management system
Concurrency Control in Database Management systemConcurrency Control in Database Management system
Concurrency Control in Database Management systemChristalin Nelson
 
How to Manage Buy 3 Get 1 Free in Odoo 17
How to Manage Buy 3 Get 1 Free in Odoo 17How to Manage Buy 3 Get 1 Free in Odoo 17
How to Manage Buy 3 Get 1 Free in Odoo 17Celine George
 
CLASSIFICATION OF ANTI - CANCER DRUGS.pptx
CLASSIFICATION OF ANTI - CANCER DRUGS.pptxCLASSIFICATION OF ANTI - CANCER DRUGS.pptx
CLASSIFICATION OF ANTI - CANCER DRUGS.pptxAnupam32727
 
ARTERIAL BLOOD GAS ANALYSIS........pptx
ARTERIAL BLOOD  GAS ANALYSIS........pptxARTERIAL BLOOD  GAS ANALYSIS........pptx
ARTERIAL BLOOD GAS ANALYSIS........pptxAneriPatwari
 
Using Grammatical Signals Suitable to Patterns of Idea Development
Using Grammatical Signals Suitable to Patterns of Idea DevelopmentUsing Grammatical Signals Suitable to Patterns of Idea Development
Using Grammatical Signals Suitable to Patterns of Idea Developmentchesterberbo7
 
Q4-PPT-Music9_Lesson-1-Romantic-Opera.pptx
Q4-PPT-Music9_Lesson-1-Romantic-Opera.pptxQ4-PPT-Music9_Lesson-1-Romantic-Opera.pptx
Q4-PPT-Music9_Lesson-1-Romantic-Opera.pptxlancelewisportillo
 

Último (20)

4.11.24 Mass Incarceration and the New Jim Crow.pptx
4.11.24 Mass Incarceration and the New Jim Crow.pptx4.11.24 Mass Incarceration and the New Jim Crow.pptx
4.11.24 Mass Incarceration and the New Jim Crow.pptx
 
Blowin' in the Wind of Caste_ Bob Dylan's Song as a Catalyst for Social Justi...
Blowin' in the Wind of Caste_ Bob Dylan's Song as a Catalyst for Social Justi...Blowin' in the Wind of Caste_ Bob Dylan's Song as a Catalyst for Social Justi...
Blowin' in the Wind of Caste_ Bob Dylan's Song as a Catalyst for Social Justi...
 
4.16.24 Poverty and Precarity--Desmond.pptx
4.16.24 Poverty and Precarity--Desmond.pptx4.16.24 Poverty and Precarity--Desmond.pptx
4.16.24 Poverty and Precarity--Desmond.pptx
 
Active Learning Strategies (in short ALS).pdf
Active Learning Strategies (in short ALS).pdfActive Learning Strategies (in short ALS).pdf
Active Learning Strategies (in short ALS).pdf
 
How to Fix XML SyntaxError in Odoo the 17
How to Fix XML SyntaxError in Odoo the 17How to Fix XML SyntaxError in Odoo the 17
How to Fix XML SyntaxError in Odoo the 17
 
4.9.24 School Desegregation in Boston.pptx
4.9.24 School Desegregation in Boston.pptx4.9.24 School Desegregation in Boston.pptx
4.9.24 School Desegregation in Boston.pptx
 
Mattingly "AI & Prompt Design: Large Language Models"
Mattingly "AI & Prompt Design: Large Language Models"Mattingly "AI & Prompt Design: Large Language Models"
Mattingly "AI & Prompt Design: Large Language Models"
 
CHEST Proprioceptive neuromuscular facilitation.pptx
CHEST Proprioceptive neuromuscular facilitation.pptxCHEST Proprioceptive neuromuscular facilitation.pptx
CHEST Proprioceptive neuromuscular facilitation.pptx
 
Daily Lesson Plan in Mathematics Quarter 4
Daily Lesson Plan in Mathematics Quarter 4Daily Lesson Plan in Mathematics Quarter 4
Daily Lesson Plan in Mathematics Quarter 4
 
31 ĐỀ THI THỬ VÀO LỚP 10 - TIẾNG ANH - FORM MỚI 2025 - 40 CÂU HỎI - BÙI VĂN V...
31 ĐỀ THI THỬ VÀO LỚP 10 - TIẾNG ANH - FORM MỚI 2025 - 40 CÂU HỎI - BÙI VĂN V...31 ĐỀ THI THỬ VÀO LỚP 10 - TIẾNG ANH - FORM MỚI 2025 - 40 CÂU HỎI - BÙI VĂN V...
31 ĐỀ THI THỬ VÀO LỚP 10 - TIẾNG ANH - FORM MỚI 2025 - 40 CÂU HỎI - BÙI VĂN V...
 
Expanded definition: technical and operational
Expanded definition: technical and operationalExpanded definition: technical and operational
Expanded definition: technical and operational
 
Congestive Cardiac Failure..presentation
Congestive Cardiac Failure..presentationCongestive Cardiac Failure..presentation
Congestive Cardiac Failure..presentation
 
MS4 level being good citizen -imperative- (1) (1).pdf
MS4 level   being good citizen -imperative- (1) (1).pdfMS4 level   being good citizen -imperative- (1) (1).pdf
MS4 level being good citizen -imperative- (1) (1).pdf
 
Concurrency Control in Database Management system
Concurrency Control in Database Management systemConcurrency Control in Database Management system
Concurrency Control in Database Management system
 
prashanth updated resume 2024 for Teaching Profession
prashanth updated resume 2024 for Teaching Professionprashanth updated resume 2024 for Teaching Profession
prashanth updated resume 2024 for Teaching Profession
 
How to Manage Buy 3 Get 1 Free in Odoo 17
How to Manage Buy 3 Get 1 Free in Odoo 17How to Manage Buy 3 Get 1 Free in Odoo 17
How to Manage Buy 3 Get 1 Free in Odoo 17
 
CLASSIFICATION OF ANTI - CANCER DRUGS.pptx
CLASSIFICATION OF ANTI - CANCER DRUGS.pptxCLASSIFICATION OF ANTI - CANCER DRUGS.pptx
CLASSIFICATION OF ANTI - CANCER DRUGS.pptx
 
ARTERIAL BLOOD GAS ANALYSIS........pptx
ARTERIAL BLOOD  GAS ANALYSIS........pptxARTERIAL BLOOD  GAS ANALYSIS........pptx
ARTERIAL BLOOD GAS ANALYSIS........pptx
 
Using Grammatical Signals Suitable to Patterns of Idea Development
Using Grammatical Signals Suitable to Patterns of Idea DevelopmentUsing Grammatical Signals Suitable to Patterns of Idea Development
Using Grammatical Signals Suitable to Patterns of Idea Development
 
Q4-PPT-Music9_Lesson-1-Romantic-Opera.pptx
Q4-PPT-Music9_Lesson-1-Romantic-Opera.pptxQ4-PPT-Music9_Lesson-1-Romantic-Opera.pptx
Q4-PPT-Music9_Lesson-1-Romantic-Opera.pptx
 

Chapter 1

  • 1. Chapter I Review topics in Algebra 1 1
  • 2. 2 1.Set of real numbers  The word algebra originated from the Arabic word “al-jabr” which means the science of reduction and cancellation. The algebraic symbolism used to generalize the operatiob of arithmetic was formulate in the sixteenth and seventeenth centuries. Real number  set of rational numbers and the set of irrational numbers make up. It consists of the set of real numbers and two operations called addition and multiplication. Addiotion is denoted by the symbol “+” and multiplication is denoted by the symbol “x” or “”. If a nd b are real numbers, a+b denotes the sum of a and b, and ab or (ab) denotes their products. If the numbers are repeating or terminating decimal they called rational number. The square roots of perfect squares also name rational number. Examples: 1) √0.16 2) 0.666 3) 1 3 4) 10 9 5) 9 6
  • 3. If the numbers are not repeating or terminating decimals. They called irrational number. 3 For examples: 1) π 2) √2 3) 0.61351 4) √8 5) √11 Properties of real numbers Let us denote the set of real numbers by 푅. These properties are statement derived from the basic axioms of the real numbers system. Axioms are assumptions on operation with numbers. Axioms of Equality Let a, b, c, d ∈ R 1. Reflexive Law If a=a 2. Symmetric Law If b=c then c=b 3. Transitive Law If b=c and c=d then b=d 4. Additional Law of Equality If a=b then a+c=b+c 5. Multiplication Law of Equality If a=b then a.c=b.c Axioms for Addition and Multiplication Let a, b, c, d, ∈ R 1) A. Closure property for addition a+b ∈ R Examples: 1) 3+3=6 2) 7+(-4)=3 3) -8+4=-4 B. Closure property for multiplication a.b ∈ R Examples: 1) 3(7)=21 2) -8(3)=-24 3) 0.11=0
  • 4. 2) A. Commutative prroperty for addition 4 a+b=b+a Examples: 1) 1 2 + 7 = 7 + 1 2 2) 0.3 + (− 5 6 ) = − 5 6 + 0.3 3) 1 3 + 21 = 21 + 1 3 B. Commutative prroperty for multiplication a.b=b.a Examples: 1) 4 5 (22) = 22 (4 5 ) 2) 6.3=3.6 3) 10 9 (−25) = −25 (10 9 ) 3) A. Associative property for addition (a+b)+c=a+(b+c) Examples: 1) (3+7)+0.4=3+(7+0.4) 2) (0.36+89)+ 1 2 = 0.36 + (89 + 1 2 ) 3) (3 5 + 0.8) + 3 8 = 3 5 + (0.8 + 3 8 ) B. Associative property for multiplication (a.b).c=a.(b.c) Examples: 1) (3.x).y=3.(x.y) 2) [5(7)] 1 4 = 5 [7 (1 4 )] 3) [3푥(6푥)]]5 = 3푥[6푥(5)] 4) Identity property for multiplication a.1=a Examples: 1) 1.a3=a3 2) 3 7 (1) = 3 7 3) 3.1=3 5) A. Inverse property for addition a+(-a)=0 Examples: 1) 6+(-6)=0 2) 10+(-10)=0 3) -3+3=0
  • 5. B. Inverse property for multiplication 5 푎. 1 푎 = 1 Examples: 1) -2(− 1 )=1 2 1 8 2) 8( )=1 3) -6(- 1 6 )=1 6) Distributive property of multiplication over addition a(b+c)=ab+ac Examples: 1) 3(4+6)=3(4)+3(6) 2) -6(7+1)=-6(7)+[-6(1)] 3) a(7+5)=7a+5 1.2 Exponents and Radicals In the expression 훼푛 , α is the base and 혯 is the exponent. The expression 훼푛 means that the value α is multiplied 혯 times by itself. Examples: 1) 63= 6.6.6 =216 2) 56= 5.5.5.5.5 =15625 3) 42= 4.4 =16 Integral and zero exponents Laws of Integral and Zero Exponents Theorem 1: For any real number α, (α≠ 0) 푎0 = 1 Examples: 1) (6푎0 + 3)0=1 2) 6α0+70=6(1)+1=7 3) 2α0+70=2(1)+1=3 Theorem 2: For any real numbers α, αm. α혯= αm+n where m and n are integers. Examples: 1) α5.α4=푎5+4 = 푎9 2) 4푥푦2(2푥2푦2) = 8푥 1+2푦2+2 = 8푥 3푦4 3) 푥 푎+3. 푥 푎+4 = 푥 2푎 +7
  • 6. 6 Theorem 3: For any real numbers a+b, (ab)n=anbn, where n is any integer. Examples: 1) (5x)2=55x2=25x2 2) (-2x)3=-23x3=-8x3 3) [x(x-3)]2=x2(x-3)2 =x2(x2-6x+9) =x4-6x3+9x2 Theorem 4: For any real numbers a (am)n=amn where m and n are integers. Examples: 1) (-x2)3=-x2(3)=-x6 2) [(3x+4)2]3=(3x+4)6 3) (-x2y3z)4=-x8y12z4 Theorem 5: For any real numbers a and b (b≠0), ( 푎 푏 )푛 = 푎푛 푏푛 where n is any integer. Examples: 1) ( 푎2 푏3 )2 = 푎4 푏6 3 4 2) ( )3 = 33 43 = 27 64 푥 푦 +2 3) ( )2= 푥2 2 = 푥(푦 +2)2 푦2 +4푦 +4 Theorem 6: For any real numbers a(a≠0), 푎푚 푎푛 = 푎푚−푛 where m and n are integers. Examples: 1) 푎7 푎5 = 푎7−5=푎2 2) 푥3푦4 푧5 푥푦푧 = 푥 3−1푦4−1 푧5−1 = 푥 2푦3푧4 3) 푥4푦4 푥4푦4 = 푥 4−4푦4− 4 = 푥 0푦0 = 1(1) = 1
  • 7. 7 Theorem 7: For any real numbers a(a≠0), 푎−푛 = 1 푎푛 Where n is any positive integer. Examples: 1) 3푥 3푦−2= 3푥3 푦2 2) (4푥 2푦)−2 = 1 (4푥2푦)2 = 1 8푥4 푦2 3) (푥 2 + 푦)−2 = 1 (푥2+푦)2 = 1 푥4+푦2 Fractional Exponents: Radicals Since not all numbers are integers, we can’t expect exponents to always whole number or zero. Exponents can be form fractional. Fractional exponents may seem unfamilliar for they are usually expressed as radicals. For expression 푥 1 2 is the same as √2 (read as square root of 2), and 푥 2 3 is the same as 3√푥2 (read as cube root of x squared). The expression 푛√푎푚 is called a radical. The symbol √ is called a radical sign, where n is the index, a is the radicand and m is the power of the radicand. 푎 푚 푛 =푛√푎푚 Laws of Radicals Theorem 1: For any real numbers a, √푎푛 = 푎 푛 Examples: 1) √42 = 4 2) 3√(푥2푦)3=푥 2푦 3 3) √33 =3 Theorem 2: For any real numbers a,and b. √푎 푛 . √푏 푛 = √푎푏 푛 Examples: 1) √3. √3 = √3.3 = √9=3 2) √4. √3 = √4.3 = √12 3) √푎. √푏 = √푎. 푏
  • 8. 8 Theorem 3: For any real numbers a,and b, (b≠0) √푎 푛 √푏 푛 = √ 푎 푏 푛 Examples: 1) 3√푎 √푏 3 = √ 푎 푏 3 2) √4 √5 = √4 5 3) 4√푥 √푦 4 = √ 푥 푦 4 Theorem 4: For any real numbers a , 푚 √푎 푚푛 = √ √푎 푛 푛 = √ √푎 푚 Examples: 3 1) √64 6 = √√64 = √8 3 = 2 2 2) √16 4 = √√16 = √4 2 =2 3) √100 3 = √100 3 =√100 = 10 Theorem 5: For any real numbers a k √푎푘푚 푛 = 푛√푎푚 Examples: 1) √24 6 = √22.2 2.3 = 3√22 = √4 3 2) √93 6 = 3.2√93.1 = √9 2 =3 3) Addition and Sutraction of Radicals To add and subtract radicals, first we need to combine the like terms with similar radicals. Examples: 1) √2 + 3√2 − 2√2 = 2√2 2) √8 + √18 + √32 = √4.2 + √9.2+√16.2 = 2√2 + 3√2 + 4√2 = 9√2 3) 푦√푥 3푦 − √푥 3푦3 + 푥√푥푦3 = 푦√푥 2. 푥푦 − √푥 2. 푥. 푦2. 푦 + 푥√푥. 푦2. 푦 = 푥푦√푥푦 − 푥푦√푥푦 + 푥푦√푥푦 = 푥푦√푥푦
  • 9. Multiplication and Division of Radicals To multiply and divide radicals with the same index, multiply, or divide the radicals and copy the common index. Examples: 1) √3.√3 = √32 = 3 2) 3√푥푦. √푥 2푦 3 . √푥푧 3 =√푥푦. 푥 2푦. 푥푧 3 = √푥 4푦2푧 3 = 푥 √푥푦2푧 3 3) √16 3 ÷ √−2 3 =√16 ÷ (−2 3 )= √−8 3 = −2 9 1.3 Polynomials Polynomials was used to describe any algebraic expression. The algebraic expression, 5x+4 and x3+x2+1 are examples of polynomials in variable x. A polynomial with just one term 2x is called a monomial. If the polynomial is the sum or difference of two terms as in -9x+7, then it is called a binomial. If it has three terms like x2+2x+1, then it is called a trinomial. In general a polynomial consisting of a sum of any numbers of terms is called a multinomial. In the binomial, 5x+4 the number 5 is called the numerical coefficient of x while x is the literal coefficient and the numbers 4 is the constant term. Addition and Sutraction of Polynomials To determined the sums and differences of polynomials, only the coefficients are combined. By similar terms are refer to the terms with the same coefficients. Those with different literal coefficient are called dissimilar or unlike terms. Examples: 1. Find the sum of 2x-3y+5 and x+2y-1, =(2x-3y+5)+( x+2y-1) =2x+x-3y+2y+5-1 =3x-y++4 2. Find the differences between 2x-3y+5 and x+2y-1 =(2x-3y+5)-( x+2y-1) =2x-3y+5+(-x-2y+1) =2x-x-3y-2y+5+1 =x-5y+6 3. Subtract 2(4x+2y+3) from 5(2x-3y+1) =5(2x-3y+1)- 2(4x+2y+3) =10x-15y+5-8x+4y+6 =2x-11y+11
  • 10. 10 Multiplication of Polynomials Examples: 1) 푥 푚.푥 푛 = 푥 푚+푛 2) 푥 −2.푥 2=푥 0 = 1 3) Multiply a+2b+3c by 5m. = a+2b+3c(5m) in multiplication, we apply the =5am+10bm+15cm distributive property Division of Polynomials To divide a polynomial by a monomial, divide each term of the polynomial by the monomial. 푥 푚 푥 푛 = 푥 푚−푛 푎푛푑 푥 −푛 = 1 푥 푛 Examples: 1) 푥5 푥2 = 푥 3 1 푥5 2) 푥 −5= 3) Divide 7푥 2 − 5푥 푏푦 푥 푥 is the divisor and 7푥 2 − 5푥 as the dividend, we have 7푥2 −5푥 푥 = 7푥2 푥 - 5푥 푥 =7푥 − 5 1.4 Factoring Factors and Greatest Common Denominator If the two of more numbers are multiplied, each number is a factor of the product. In the example above, 18 is expressed as the product of different pair of whole numbers. 18=2.9 18=3.6 18=18.1 A prime number is a whole number, greater than1, whose only factors are 1 and itself. A composite number is a whole number greater than 1, that is not prime. Examples: 1) Find the prime factorization of 84. 84=2.42 the least prime factor of 84 is 2 =2.2.24 the least prime factor of 42 is 2 =2.2.3.7 the least prime factor of 21 is 3 All of the factors in tha last row are prime. Thus, the prime factorization of 84 is 2.2.3.7 or 22.3.7. 2) Factor 20a2b 20a2b=2.10.a.a.b =2.2.5.a.a.b
  • 11. The greatest common factor of two or more integer is the product of the prime 11 factors common to the integers. Examples: 1) Find the GCF of 54, 63, and 180. 54=2.③.③.3 factor each number 63③.③7 180=2.2.③.③.5 then circle the common factors The GCF of 54, 63, and 180 is 3.3 or 9. 2) 8푎2푏 푎푛푑 18푎2푏2푐 8푎2푏=②. 2.2. 푎 ⃝ . 푎 ⃝ . 푏 ⃝ 18푎2푏2 푐=②.3.3. 푎 ⃝ . 푎 ⃝ . 푏 ⃝. 푏. 푐 = 2푎2푏 The GCF of 8푎2 푏 푎푛푑 18푎2푏2푐 is 2푎2푏. Factoring Using the Distributive Property To multiplied a polynomial by a monomial by using the distributive property. Multiplying Polynomials Factoring Polynomials 3(a+b)=3a+3b 3a+3b=3(a+b) x(y-z)=xy-xz xy-xz= x(y-z) 3y(4x+2)=3y(4x)+3y(2) 12xy+6y=3y(4x)+3y(2) =12xy+6y =3y(4x+2) Examples: 1) Use the distributive property to factor 10푦2 + 15푦 10푦2=2.⑤. 푦 ⃝. 푦 15푦 =3.⑤.푦 The GCF is 5y 10푦2 + 15푦=5y(2y)+5y(3) =5y(2y+3) distributive property 2) Factor 21푎푏2 − 33푎2푏푐 21푎푏2=③.7. a ⃝ . 푏 ⃝. 푏 33푎2푏푐=③.11. a ⃝ . 푎. 푏 .⃝c the GCF is 3ab 21푎푏2 − 33푎2푏푐= 3ab(7b)-3ab(11ac) =3ab(7b-11ac) distributive property Factoring by Grouping Polynomial with four or more terms, like 3xy-21y+5x-35, can sometimes be factored by grouping terms of the polynomials. One method is to group the terms into binomials that can each be factored using the distributive property. Then use the the distributive property again with a binomial as the common factor.
  • 12. 12 Examples: 1) Factor 3xy-21y+5x-35 3xy-21y+5x-35= (3xy-21y)+(5x-35) =3y(x-7)+5(x-7) =3y+5(x-7) Check by using FOIL ; (3y+5)(x-7)=3y(x)+3y(-7)+5(x)-5(7) =3xy-21y+5x-35 2) Factor 8푚2 푛 − 5푚 − 24푚푛 + 15 8푚2 푛 − 5푚 − 24푚푛 + 15=(8푚2푛 − 5푚) + (−24푚푛 + 15 = 푚(8푚푛 − 5) + (−3)(8푚푛 − 5) =푚 − 3(8푚푛 − 5) Check: 푚 − 3(8푚푛 − 5) = 푚(8푚푛) + 푚(−5) + (−3)(8푚푛) + (−3)(-5) = 8푚2 푛 − 5푚 − 24푚푛 + 15 Factoring Trinomials When two numbers are multiplied each number is a factor of the product. Similarly if two binomials are multiplied, each binomials is factor of the product. Consider the binomials 5x+2 and 3x+7. You can use the FOIL method to find their product. (5x+2)( 3x+7)=(5x)(3x)+(5x)(7)+(2)(3x)+(2)(7) =15x2+35x+6x+14 =15x2+(35+6)x+14 =15x2+41x+14 You can be use this pattern to factor quadratic trinomials, such as 2푦2 + 7푦 + 6 Factors of 12 Sum of Factors 1.12 1+12=13 no 2.6 2+6=8 no 3.4 3+4=7 yes 2푦2 + (3 + 4)푦 + 6 Select the factors 3 and 4. 2푦2 + 3푦 + 4푦 + 6 (2푦2 + 3푦) + (4푦 + 6) Group terms that have a 푦(2푦 + 3) + 2(2푦 + 3) common monomials factor (푦 + 2)(2푦 + 3) Factor (use the distirbutive property) Therefore 2푦2 + 7푦 + 6= (푦 + 2)(2푦 + 3)
  • 13. 13 Example: Factor 5푥 − 6 + 푥 2 The trinomials 5푥 − 6 + 푥 2 can be written as 푥 2 + 5푥 − 6. For this trinomials, the constant terms is -6 and the coefficient of x is 5. Thus, we need ti find two factors two factors of -6 whose sum is 5. Factors of -6 Sum of factors 1, -6 1+(-6)=-5 no -1, 6 -1+6=5 yes Select the factors -1 and 6 Therefore, 푥 2 + 5푥 − 6 = (푥 − 1)(푥 + 6) Factoring Differences of Square The product of the sum and ifference of two expressions is called the differences of squares. The process for finding this product can be reversed in order to factor the differenceof squres. Factoring the difference of square can also be modeled geometrically. 푎2 − 푏2 = (푎 − 푏)(푎 + 푏) Examples: 1) 푓푎푐푡표푟 푎2 − 64 푎2 − 64 = (푎)2 − (8)2 = (푎 − 8)(푎 + 8) 푎. 푎 = 푎2 푎푛푑 8.8 = 64 use the difference of square 2) 푓푎푐푡표푟 푎푥 2 − 100푦2 푎푥 2 − 100푦2 = (3푥)2 − 10푦2 = (3푥 − 10푦)(3푥 + 10푦) 3푥. 3푥 = 9푥 2 푎푛푑 10푦. 10푦 = 100푦2 Perfect Square and Factoring Numbers such as 1,4,9 and 16 are called perfect squares. Since they can be expressed as the square of an integer. Products of the form (푎 + 푏)2 푎푛푑 (푎 − 푏)2 are called perfect squares and the expansions of these products are called perfect square trinomials. (푎 + 푏)2 = 푎2 + 2푎푏 + 푏2 (푎 − 푏)2 = 푎2 − 2푎푏 + 푏2 Finding a Product Factoring (푦 + 8)2 = 푦2 + 2(푦)(8) + 82 푦2 + 16푦 + 64 = (푦)2 + 2(푦)(8) + (8)2 = 푦2 + 16푦 + 64 = (푦 + 8)2
  • 14. Examples: Determine whether 16푎2 + 81 − 72푎 is a perfect square trinomial. 1) 16푎2 + 81 − 72푎 = 16푎2 − 72푎 + 81 = (4푎)2 − 2(4푎)(푎) + (푎)2 = (4푎 − 9)2 2) 푥 2 + 22푥 + 121 = (푥)2 + 2(푥)(11) + (11)2 14 = (푥 + 11)2 Solving Equations by Factoring Zero Product Property For all numbers a and b, if ab=0, then a=0, b=0 or both a and b equal 0. Example: 1) Solve 16t(9-t)=0 16t(9-t)=0, then 16t=0 or 9-t zero product property 16t=0 or 9-t=0 solve each equation t=0 9=t check: Substitute 0and 9 for t in the original. 16t(9-t)=0 16(0)(9-0)=0 or 16(9)(9-9)=0 0(9)=0 144(0)=0 0=0 0=0 SOLUTION SET: (0,9) 2) (y+2)(3y+5)=0 If (y+2)(3y+5)=0, then y+2=0 or 3y+5=0 y+2=0 or 3y+5=0 y=-2 3y=-5 푦 = − 5 3 Check: (y+2)(3y+5)=0 (-2+2)[(3)(-2)+5]=0 or (− 5 3 + 2) [(3) (− 5 3 ) + 5] = 0 0(-1)=0 1 3 (0) = 0 0=0 0=0 SOLUTION SET: (-2, − 5 3 ) 1.5 Rational Expressions A fraction where the numerator and denominator are polynomials, and is defined for all values of the variable that do not make the denominator zero. Reducing Rational Expression to Lowest Terms We need to lowest term the fraction, if the numerator and denominator have no common factor.
  • 15. 푥2−2푥푦+4푦2 3푥(푥+푦)−2푦 (푥+푦)+2푥2 −푦2 15 Examples: 1) 4푎2 푏푐3 6푎푏3 푐4 = 2.2.푎.푎 .푏.푐.푐.푐 2.3.푎.푏.푏.푏.푐.푐.푐 .푐 = 2푎 3푏2푐 2) 푥2+2푥푦 +푦2 푥2 −푦2 = (푥+푦)(푥+푦 ) = (푥+푦)−(푥−푦 ) 푥+ 푦 푥− 푦 3) 푥3+8푦 3 4푥+8푦 = 푥+2푦 (푥2−2푥푦 +4푦2 4(푥+2푦 ) = 4 Multiplying and Dividing Rational Expressions In multiplication if 푝 푞 푎푛푑 푟 푠 are rational expressions and q and s are real numbers not equal to 0, then 푝 푞 . 푟 푠 = 푝푟 푞푠 . Examples: 1) 4 3 . 1 5 = 4 15 2) 푐 . (푎 + 2푏)(푎 − 푏) 푎2−푏2 푐 = (푎 + 푏)(푎 − 푏) . (푎 + 2푏)(푎 − 푏) 푐(푎+2푏) = 푎+푏 In dividing algebraic fractions, multiply the dividend by the reciprocal of the divisor. The reciprocal of a fraction is its multiplicative inverse. Examples: 1) 4 3 ÷ 6 5 = 4 3 . 5 6 = 20 18 표푟 10 9 2) 8 7 ÷ 3 = 8 7 . 1 2 = 8 14 표푟 4 7 3) 푦2 −16 푦 −5 ÷ 2푦 −8 푥푦−5푥 (푦−4)(푦+4) = 푦 −5 . 푥(푦−5) 2(푦−4) = 푥푦 +4푥 2 Adding and Subtracting Rational Expressions. To add and subtract rational expressions, it is the important that the least common denominator is accurately determined. Examples: 1) 5 6 − 2 3 + 1 8 = 20−16+3 24 = 7 24 2) 4 5 + 3 5 + 2 5 = 4+3+2 5 = 9 5 3) 3푥 − 2푦 + 2푥2 −푦2 푥 +푦 = 푥+푦 = 3푥2 +3푥푦 −2푥푦 +2푦2 +2푥2 −푦2 푥 +푦 = 5푥2 +푥푦− 3푦2 푥+푦
  • 16. Simplifying Complex Rational Expressions A factor which contains one or more fractions either in the numerator or The 푛푡ℎ root of a real number If n is a positive integer greater than 1 푎푛푑 푎 푎푛푑 푏 are real number such that 푏푛 = 푎, then b is an 푛푡ℎ root of a. The principal 푛푡ℎ root of a real number. If n is a positive integer greater than 1, a is a real number, and √푎 푛 denotes the princial 푛푡ℎ root of a, then If a>0, √푎 푛 is the positive 푛푡ℎ root of a. If a<0, and n is odd, √푎 푛 is the negative 푛푡ℎ root of a. √0 푛 = 0 16 denominator or in both. Examples: 1) 4 31 3 = 4 3 . 3 1 = 12 3 표푟 4 2) 3 2+ 1 3 = 3 6+1 3 = 3 7 3 = 3. 3 7 = 9 7 1.6 Rational Exponents We defined 푎푛 if n is any integer (positive, negative or zero). To define a power of a where the exponent is any rational number, not specifically an integer. 1 That is, we wish to attach a meaning to 푎 ⁄푛 푎푛푑 푎 푚 ⁄푛, where the exponents are fractions. Before discussing fractional exponents, we give the following definition. Definition Examples 1: 1) 2 is a square root of 4 because 22 = 4 2) 3 is a fourth root of 81 because 34 = 81 3) 4 is a cube root of 64 because 43 = 64 Definition . The symbol √ is called a radical sign. The entire expression √푎 푛 is called a radical, where the number a is the radicand and the number n is the index that indicates the order of the radical.
  • 17. 17 Examples 2: 1) √4 = 2 2) √81 4 = 3 3) √64 3 = 4 Definition Examples 3: 1) 25 1 ⁄2 = √25 = 5 2) −8 1 ⁄3= √−8 3 = −2 3) ( 1 81 )1/4=√ 1 81 4 = 1 3 Definition Examples 4: 1) 9 3 ⁄2=(√9)3=33=27 2) 8 2 ⁄3 = (√8 3 )2=22 = 4 3) −27 4 ⁄3 = (√−27 3 )4=(-3)4=81 It can be shown that the commutative law holds for rational exponents, and therefore (푎푚)1/n=(푎 1 ⁄푛)m From which it follows that 푛√푎푚 = ( √푎 푛 )m The next theorem follows from this equality and the definition of 푎 푚 ⁄푛 If n is a positive integer greater than 1, and a is a real number, then if √푎 푛 is a real number 푎 1 ⁄푛 = √푎 푛 If m and n are positive integers that are relatively prime, and a is a real number, then if √푎 푛 is a real number 푎 푚 ⁄푛 = (√푎 푛 )m ⇔ 푎 푚 ⁄푛 = (푎 1 ⁄푛)m
  • 18. If m and n are positive integrers that are relatively prime, and a is a real number, then if √푎 푛 is a real number 푚 ⁄푛 = (푎푚)1/n 18 Theorem 1 Examples 5: 푚 ⁄푛 = 푛√푚 Theorem 1 푎is applied ⇔ 푎 in the following: 1) 9 3 ⁄2=√93=729 =27 2) 8 2 ⁄3 = √8 3 2=√64 3 = 4 3) −27 4 ⁄3 = (√−27 3 )4=√531441 3 =81 푎 Observe that 푎 푚 ⁄푛 can be evaluated by finding either ( √푎 푛 )m or 푛√푎푚. Compare example 4 and 5 and you will see the computation of ( √푎 푛 )m in example 4 is simpler than that for 푛√푎푚 in example 5. The laws of positive-integer exponents are satisfied by positive-rational exponents with one exception: For certain values of p and q, (ap)q≠apq for a<0. This situation arises in the following example. Examples 6: 1) [(-9)2]1/2=811/2=9 and (-9)2(1/2)=(-9)1=-9 Therefore [(-9)2]1/2≠(-9)2(1/2). 2) [(-9)2]1/4=811/4=3 and (-9)2(1/4)=(-9)1/2 (not a real number) Therefore [(-9)2]1/4≠(-9)2(1/4). The problems that arise in example 6 are avoided by adopting the following rule: If m and n are positive even integers and a is a real number, then (푎푚 )1/n=│a│m/n A particular case of this equality occurs when m=n. We then have (푎푛)1/n=│a│ (if n is a positive even integer) or, equivalently, 푛√푎푛 = │a│ (if n is even) If n is 2, we have √푎2 = │a│ Examples 7: 1) [(-9)2]1/2=│-9│=9 2) [(-9)2]1/4=│-9│2/4=91/2=3
  • 19. Definition If m and n positiv e integer that are relatively prime and a is a real number and 1 ⁄3)2=(1 19 Example:8 1) 8 −2 ⁄3 = 1 8 2 ⁄3= 1 ( 3√8)2 = 1 22= 1 4 2) 8 −2 ⁄3 = (8 −1 ⁄3)2=( 1 8 2 1 4 )2= 3) 1 푥 ⁄3 푥 1 ⁄4 =푥 1 ⁄3. 1 푥 1 ⁄4=푥 1 ⁄3.푥 −1 ⁄4 = 푥(1 ⁄3)−1 ⁄4 = 푥 1 ⁄12 a≠0, then if √푎 푛 is a real number. 푎 −푚 ⁄푛 = 1 푎 푚 ⁄푛
  • 20. ASSESSMENT 20 TEST I Name the property that justifies each step. 1. Simple 6a+(8b+2a) a. 6a+(8b+2a)=6a+(2a+8b) b. =(6a+2a)+8b c. =(6+2)a+8b d. =8a+8b 2. Simplify 6푎2 + (6푎 + 푎2 ) + 9푎 a. 6푎2 + (6푎 + 푎2 ) + 9푎 = 6푎2 + (푎2 + 6푎)+9a b. = (6푎2 + 푎2 ) + (6푎+9a) c. = (6푎2 + 1푎2) + (6푎+9a) d. =6 + 1(푎2) + (6 + 9)푎 e. = 7푎2 + 15푎 Simplify and express the following. 1. 푎푚+5 푎푚−2 2. [ (푥+푦)0+푎0+ 푏0 푎+푏+푐 ]-2 3. (푎−2 + 푦)-2 4. (37푥+5 )(34푥−4) 5. (9푥푦2)(4푥 3푦) Rational Expression (simplify) 1. 9푛 63푛 ÷ 9푛 2. −15푚3 푛2 푝2 −35푚2 푛5 푝 3. 푥+푦 푥2−푦2 4. 3푚− 1 9(푚−1)2−4 5. 4푚푛+6 10푚 +8푛 6. 2푥2 +3푥 −5 10푥 +25 7. 푥2−5푥 −24 4푥2 −27푥 −40 8. 25푎2 +70푎 +49 25푎 2 −49 Factor each polynomial into two binomials 1. a2+ 12a+ 27 2. y2+ 21y+ 110 3. n2-4n+ 4 4.x2-12x + 20 5. x2+ 11x -12
  • 21. Answer the following word problemsand multiple choice questions 1. The area of a rectangle is (x2–12x + 35). If the length is (x-5),find the width.(hint:“x5” times“something” will give you “x2–12x + 35.”). 2. The area of a rectangle is 3a2+ 5a–28. If the length is (a+ 4), find the width. 3. A rectangle has an area of 3x2+ 5x –12. What factors are the length and width of 21 the rectangle? a. (3x + 4)(x –3) b. (3x –4)(x + 3) c. (3x + 3)(x –4) d. (3x –3)(x + 4) 4. The area of a certain rectangle is 5n2–6n–27. Which factors are the width and length of the rectangle? a. (5n + 3)(n –9) b. (5n –3)(n + 9) c. (5n + 9)(n –3) d. (5n –9)(n + 3) 5. If the area of a certain rectangle is 6m2–2a –28, and the length is (2m + 4), what is the width? a. (3m + 7) b. (4m –7) c. (4m + 7) d. (3m –7) TEST II A. Determine whether each statement is true or false. 1. Every integer is also a real number. 2. Every irrational number is also an irrational number. 3. Every natural number is also a whole number. 4. Every real number is also a rational number. B. State whether each decimal represents a rational o irrational number. 5. √4 6. √5 7. 0 8. 3 9. 0.63586358 10. √866 C.Determine which real number property is shown by each of the following 1. − 1 4 + 1 4 = 0 2. 2(1)=2 3. 1 4 (4)=1 4. -7+(-4)=-4+(-7) 5. 0.3(0)=0.3
  • 22. 22 6. 5[3+(-1)]=5(3-1) 7. (8+ 9 8 9 8 )+0.45=8+( +0.45) 8. 5(8+8)=5(8)+5(8) 9. 6x+(8x+10)=(6x+8x)+10 10. 5a+2b=2b+5a D.Simplify and express the following expressions with positive and negative integrals only. 1. 50 2. 10푚4 30m 3. 16푏4 푐 −4푏푐3 4. 푦3. 푦4 5. (5푥푦)6 6. (푎푏)3 7. (푥 3푦2)3 8. [(−5)2]2 9. 푥5푦6 푥푦 = 10. 푎7 푎3 E.Simplify the following radicals 1.(5√2)(3√6) 2.(3푎3√4푥2)(4 √3푥푦 3 ) 3.4√ 9 16 4.√2(3+√3) 5.5√2+3√2 6.√18 − 2√27 + 3√3 − 6√8 7.√16푏 + √4푏 8. −12√24 3√2 9.√8 + √50 10. 4√푥 7푦10 F.Simplify the following polynomials. 1. (5푥 − 1) + (10푥 2 + 7푥) 2.(20푥 2 + 2) + (15푥2 − 8) + (3푥 2 − 4) 3.(푥 2 + 푦2 + 8) + (4푥 2 − 2푦2 − 9) 4.(−3푥 2 + 5푦 − 4푥푦 + 푦2) 푓푟표푚(2푥2 − 4푦 + 7푥푦 − 6푦2) 5.2푥 2 + 6푥 + 5 푎푛푑 3푥2 − 2푥 − 1
  • 23. 23 6.(푥 + 2)(푥2 − 2푥 + 3) 7.푎푏(2푎 + 1) 푥2−3푥 −10 8. 푥+ 2 푥6+2푥4 +6푥−9 9. 푥3+3 10. (3푥 3 − 11푥 2푦 + 11푥푦2 − 2푦3) ÷ (푥 − 2푦) G.Find the factors and its GCF 1. 21 2. 4, 12 3. 63 4. 304 5. 18, 35 6. 12푎푛2, 40푎4 7. 7푦2, 14푦2 8. 15, 10 9. 6푎2, 18푏2, 9푏3 10. 18푥 2푦2, 6푦2, 42푥 2푦3 H.Factor each trinomials. If possible. If the trinomial cannot be factored using integers. Write prime. 1. 6푚푥 − 4푚 + 3푥 − 2푟 2. 3푚푦 − 푎푏 + 푎푚 − 30푦 3. 푎2 − 2푎푏 + 푎 − 20 4. 3푚2 − 5푚2 푝 + 3푝2 − 5푝3 5. 4푎푥 − 14푏푥 + 35푏푦 − 10푎푦 6. 푦2 + 12푦 + 27 7. 푐2 + 2푐 − 3 8. 푥 2 − 5푥 − 24 9. 7푎2 + 22푎 + 3 10. 6푦2 − 11푦 + 4 I.Factor out each polynomials 1. 푥 2 − 49 2. 푥 2 − 36푦2 3. 16푎2 − 9푏2 4. 2푎2 − 25 5. 2푧2 − 98 6. 푛2 − 8푛 + 16 7. 4푘2 − 4푘 + 1 8. 푥 2 + 6푥 − 9 9. 1 − 10푧 + 25푧2 10. 50푥 240푥 + 8
  • 24. 24 J. Solve and simplify each polynomials 1. 푎+1 푎3 − 푎 +2 푎2 + 푎 +3 푎 2. 5푥3 7푦4 . 21푦 2 10푥2 3. 9푥5 36푥2 4. 5−푎 푎2− 25 5. 10푎2 −29푎 +10 6푎2 −29푎 +10 ÷ 10푎2 −19푎 +6 12푎 2 −28푎 +15 6. 1 푥+ℎ − 1 푥 ℎ 7. 푥6−7푥3 − 8 4푥2−4푥−8 ÷ (2푥 2 + 4푥 + 8) 8. 푎 푏 − 푏 푎 푎 푏 + 푏 푎 9. 푡2 −2푡 −15 푡2 −9 . 푡2− 6푡 +9 12−4푡 10. 푎−1+푏−1 푎−2−푏−2
  • 25. Chapter II Equations and Inequalities 25
  • 26. 26 2.Equations To sove an equation means to isolate the variable having a coefficient of 1 on one side of the equation. By using Addition Property of Equality. Examples: 1) solve r+16=-7 r+16=-7 r+16+(-16)=-7+(-16) add -16 to each side r=-23 the sum of -16 and 16 is 0 check: r+16=-7 -23+16=-7 -7=-7 2) x+(3.28)=-17.56 x+(3.28)=-17.56 x+(3.28)+(3.28)=-17.56+3.28 x=-14.28 check: x+(3.28)=-17.56 -14.28+(-3.28)=-17.56 -17.56=-17.56 3) y+21=-7 y+21+(-21)=-7+(-21) y=-28 check: y+21=-7 -28+21=-7 -7=-7 Equations by Using Subtraction The property that used to subtract the same number from each side of an equation is called the subtraction property of equality. Examples: 1) x+15=-6 x+15-15=-6-15 x=-21 check: x+15=-6 -21+15=-6 -6=-6
  • 27. 27 2) b-(-8)=23 b+8=23 b+8-8=23-8 b=15 check: b-(-8)=23 15-(-8)=23 23=23 Equations by Using Multiplication and Division To solve the equation by using multiplication, we use the multiplication property of equality. For any numbers a,b, and c, if a=b, then ac=bc Eamples: 1) 5 12 = 푟 24 24( 5 12 ) = ( 푟 24 )24 multiply each side by 24 10 = 푟 Check: 5 = 푟 12 24 replace r with 10 5 12 = 10 24 5 12 = 5 12 2) 24=-2a 24=-2a − 1 2 (24) = − 1 2 (2푎) −12=a 푐ℎ푒푐푘: 24 =−2a 24 =−2a(-12) 24 = 24 To solve the equation by using division, we use the division property of equality. For any numbers a,b,c with c≠ 0, If a=b, then 푎 푐 = 푏 푐 . Examples: 1. -6x=11 −6푥 −6 = 11 −6 divide each side by -6 푥 = − 11 6
  • 28. 28 Check: -6x=11 −6(− 11 6 )=11 11 = 11 2. 4x=24 4푥 4 = 24 4 X=6 Check: 4x=24 4(6)=24 24=24 2.2 Appplication of Linear Equations In many applications of algebra, the problems are stated in words. They are called word problems, and they give relatiomships between known numbers and unknown numbers to be determined. In this section we solve word problems by using linear equations. There is no specific method to use. However, here are some steps that give a possible procedurefor you to follow. As you read through the examples, refer to these steps to see how they are applied. 1. Read the problem carefully so that you understand it. To gain understanding, it is often helpful to make a specific axample that involves a similar situation in which all the quatities are known. 2. Determine the quantities that are known and those that are unknown. Use a variable to represent one of the unknown quantities inthe equation you will obtain. When employing only one equation, as we are in this section, any other unknown quantities should be expressed in terms of this one variable. Because the variable is a number, its definition should indicate this fact. For instance, if the unknown quantity is a length and lengths are mesured in feet, then if x is a variable, x should be defined as the number of feet in the length or, equivalently, x feet is the length. If the unknown quuantity is time, and time is measured in seconds, then if t is the variable, t should be defined as the number of seconds in the time or, equivalently, t seconds is the time. 3. Write down any numerical facts known about the variable. 4. From the information in step 3, determined two algebraic expressions for the same number and form an equation from them. The use of a table as suggested in step 3 will help you to discover equal algebraic expressions. 5. Solve the equation you obtained in step 4. From the solution set, write a conclusion that answers the questions of the problem. 6. It is important to keep in mind that the variable represents a number and the equation involves numbers. The units of measurement do not appear in the equation or its solution set.
  • 29. 7. Check your results by determining whether the condition of the word problem are satisfied. This check is to verify the accuracy of the equation obtained in step 4 as well as the accuracy of its solution set. Example 1 If a rectangle has a length that is 3cm less than four times its width and its perimeter is 9cm, what arethe dimension? Solution w: the number of centimeters in the width of the rectangle 4w-3: the number of centimeters in the length of the rectangle (4w-3)cm w cm w cm (4w-3)cm w+(4w-3)+ w+(4w-3)=19 10w-6=19 10w=25 29 w= 5 2 4w-3=4(5 )-3 2 =7 Example 2 A man invested part of $15,000 at 12 percent and the remainder at 8 percent. If his annual income from the two investments is $1456, how much does he have invested at each rate? Solution x: the number of dollars invested at 12 percent 15,000-x: the number of dollars invested at 8 percent
  • 30. Number of Dollar × Rate = Number of Dollars invested in Interest 12 percent investment x 0.12 0.12x 8 percent investment 15,000-x 0.08 0.08(15,000-x) 0.12x+0.08(15,000-x)=1456 30 0.12x+1200-0.08x=1456 0.04x=256 x=6400 15,000-x=15,000-6400 =8600 Thus the man has $6400 invested at 12 percent and $8600 at 8 percent. Example 3. A father and daughter leave home at the same time in separate automobiles. The father drives to his office, a distance of 24 km, and the daughter drives to school, a distance of 28 km. They arrive at their destinations at the same time. What are their average rates, if the father’s average rate is 12km/hr less than his daughter’s? Solution: r: the number of kilometers per hour in the daughter’s average rate r-12: the number of kilometers per hour in the father’s average rate Number of Kilometers ÷ Number of Kilometers = number of hours In Distance per hour in rate in time Daughter 28 r 28 푟 Father 24 r-12 24 푟−12 Equation: 28 = 푟 24 푟 − 12 Solve the equation by first multiplying on both sides by the LCD: 푟(푟 − 12) 28 푟 = 푟(푟 − 12) 24 푟 − 12 (r-12)28=r(24) (r-12)7=r(6) 7r-84=6r 7r-6r=84 r=84 r-12=84-12 =72 Therefore, the daughter’s average rate is 84km/hr and the father’s average rate is 72km/hr.
  • 31. 2.3 Quadratic Equation in One Variable ax2 + bx + c =0, a≠0 where a, b, and c are real number constants and a≠0, is called a second degree polynomial equation, or quadratic equation , in the variable x. the word “ quadratic “ comes from quadrate, meaning square or rectangular when a quadratic equation is written in the above manner, it is said to be in standard form. The following are examples of quadratic equations in x, written in the form, with 31 the indicated values of a, b and c. 8x2 + 16x -5 = 0 ( a = 8, b = 16, c = -5 ) 2x 2 - 10 = 0 ( a =2, b = 0, c = -10 ) 5x2 – 7x = 0 ( a = 5, b = -7, c = 0 ) Theorem 1 If r and s are real numbers, then rs = 0 if and only if r =0 or s=0 This theorem can be extended to a product of more than two factors. For instances, if r, s , t, u єR, then rstu = 0 if and only if at least one of the numbers r, s, t or u is 0. To find the solution set of the equation X2 +3x – 10 = 0 We factor the left side and obtain ( x+5 )( x-2 ) By applying Theorem 1, it follows that the equations gives a true statement if and only if X+5 = 0 or x-2 = 0 The solution of the first of these equation is -5 and the solution of the second is 2. Therefore the solution set of the given equation. Example 1: 1+5x/6 = 2x2 /3 (6) (1) + (6) 5x/6 = (6) 2x2 /3 6+ 5x =4x2 -4x2 +5x +6 = 0 4x 2-5x -6 = 0 ( 4x+3 ) (x-2 ) = 0 4x=3 = 0 x-2 =0 4x =-3 x =2 X =-3/4 The solution set is {-3/4, 2 } Suppose we have a quadratic equation of the form X 2 = d That is, there is no first degree term. Then an equivalent equation is X2 –d = 0
  • 32. And, factoring the left member, we obtain ( x- d )( x + d ) We set each factor equal to zero and solve the equations X - √ d = 0 x + √d = 0 x = √d x = -√d therefore the solution set of the equation x2 = d is { √d , -√d }. We can abbreviate this solution set as {± √d }. Thus x2 = d if and only if x= ±√d 32 Example 2: a. X2 = 25 b. x2 =13 c. x2 =16 X = ±√ 25 x =√13 x = ± √-16 X = ± √5 x = ± 4ἰ The solution set of the equation x2 + 6x-1 =0 we first add 1 to each side and obtain x2+ 6x =1 we now add to each side the square of one half of the coefficient of x, or 32. We obtain x2 +6x +9 = 1+9 The left side is now the square of x + 3. Thus we have (x +3 )2 =10 Taking the square root of both sides of the equation, we have X = 3 = ±√10 X = -3 ±√10 This method is called completing the square To complete the square of x2 + kx, add the square of one half the coefficient of x; that is, add (k/2)2. Quadratic Formula If a≠0, the solutions of the equation ax2+ bx + c =0 are given by 푥 = −푏±√푏2−4푎푐 2푎 Example 3: Use quadratic formula to find the solution set of the equation 6x2= 10 + 11x ×= −푏 ± √푏2 − 4푎푐 2푎 = −(−11) ± √(−11)2 − 4(6)(−10) 2(6) = 11 ± √121 + 240 12
  • 33. 33 = 11 ± √361 12 = 11 ± 19 12 = 11 + 19 12 = 11 − 19 12 = 30 12 = −8 12 = 5 2 = − 2 3 The solution set is {− 2 3 , 5 2 }. 2.4 INEQUALITIES Trichotomy Property of Order If a and b are real numbers, exactly one of the following three statements is true. A<B B<A A = B Transitive Property or order If a, b and c are real numbers, and if a<b and b<c then a<c. The domain of a variable in an inequality is the set of real numbers for which the members of the inequalities are defined. Examples: 4x – 9 < 10 x – 9/10 ≤ x 3 < 5x + 7 ≤ 15 An example of a quadratic inequality having domain R is X2 + 5x > 13 The inequality 4x/x + 3 < 6 is rational. Any number in the domain for which the inequality is true is a solution of the inequality, and the set of all solution is called the solution set. An absolute inequality is one that is true for every number in the domain. For instance, if x is a real numbers x + 1 < x + 2 and x2 ≥ 0 are absolute inequalities. A conditional inequality is one for which there is at least one number in the domain that is not in the solution set. To find the solution set of a conditional inequality we proceed in a manner similar to that used to solve an equation that is we obtain equivalent inequalities until we have one whose solution set is apparent. PROPERTIES OF INEQUALITIES If a, b and c are real numbers a. Addition Property If a < b, then a + c < b + c b. Subtraction Property
  • 34. 34 If a < b, then a – c < b – c c. Multiplication Property If a < b and c > 0, then ac < bc If a < b and c < 0, then ac > bc Example 1 Find and show on the real number line the solution set of the inequality. 3x – 8 < 7 3x – 8 + 8 < 7 + 8 3x < 15 1/3 (3x) < 1/3 (15) X < 5 Example 2 Find and show on the real number line the solution set of the inequality X – 7/4 ≤ x (4)x – 7/4 ≤ (4)x X – 7 ≤ 4x X – 7 – 4x + 7 ≤ 4x – 4x + 7 -3x ≤ 7 -1/3 (-3x) ≥ (-1/3)7 X ≥ -7/3 2.5 POLYNOMIAL AND RATIONAL INEQUALITIES A quadratic and inequalities is one of the form ax2 + bx + c < 0 A critical number of the inequality above is a real root of the quadratic equation ax2 + bx + c = 0. To solve the inequality X2 – 8 < 2x We first write an equivalent inequality having all the non-zero terms on one side of inequality sign. Thus we have x2 – 2x – 8 < 0 ` (x + 2) (x – 4) < 0 Example I Find and show on the real number line the solution set of the inequality. x 2 + 5x ≥ - 6 x 2 +5x + 6 ≥ 0 (x+3) (x+2) ≥ 0 Figure 1 -4 -3 -2 -1 0 1 2 The critical numbers are -3 and -2. The points corresponding to these numbers are plotted in figure 1 and the following intervals are determined. (-∞ , -3) (-3 , -2) (-2 , +∞)
  • 35. 35 Figure 2 ] [ -4 -3 -2 -1 0 1 2 Thus, the solution set of the given inequality is (-∞ , -3] U [-2 , +∞) , appearing in Figure 2. Example 2 Find the solution set of each of the following inequalities x2- 6x + 9 < 0 (x-3) 2< 0 Because there is no value of x for which (x-3) 2 is negative, there is no solution. Therefore, the solution set is 9x2 + 12x + 4 ≤ 0 (3x + 2)2 ≥ 0 Because (3x + 2)2 is non-negative for all values of x , the solution set is the set of all real numbers. 2.7 Equations and Inequalities Involving Absolute Value The absolute value of areal number a denoted by /a/ , is given by / a / = { a, if a ≥ 0 } /a / = { -a, if a < 0 } Example 1 Find the solution set of the equation / 3x-2 / = / 8-4x / 3x – 2 = 8 – 4x 3x + 4x = 8 + 2 7x = 10 x = 10/7 3x – 2 = - ( 8 – 4x ) 3x – 2 = - 8 + 4x 3x – 4x = - 8 + 2 -x = -6 x=6 The solution set is {10/7 , 6} Example 2 Find the solution set of the equation / 2x – 7 / <9 -9 <2x – 7 <9 -9 + 7 <2x <9 + 7 -2 <2x <16 -1 <x <8 Therefore the solution set is the open interval (-1, 8)
  • 36. 36 Theorem 1 If a and b are real numbers, then / ab / = / a / / b / Example 3 / ( 3x + 2 – 8 / <1 / 3x -6 / <1 / 3 (x-2) / <1 / 3 / / x-2 / <1 3 / x-2 / <1 / x-2 / <1/3 The Triangle Inequality If a and b are real numbers, then / a + b / ≤ / a / + / b / If a = 2 and b = 7 , then / a + b / = / 2 + 7 / / a / + / b / = / 2 / + / 7 / = / 9 / = 2 + 7 = 9 = 9 If a = -2 and b = -7 , then / a + b / = / -2 + -7 / / a / + / b / = / 2 / + / -7 / = / -9 / = 2 +7 = 9 = 9
  • 37. ASSESSMENT 37 TEST I A. Find the solution set of the equation. 1. 3 (6x – 5) = 11 – ( 4 + 8x ) 2. 49x2 – 64 = 0 3. 2 + 3 = 0 6 – y y – 2 4. (x – 4) ( x + 2) = 7 5. 2p2 – 4p – 5 = 0 6. √2x + 5 + x = 5 7. √y + 2 + √y + 5 - √8 – y = 0 8. / r + 1 / = 6 r – 1 9. / 8x - 2 / = x + 7 10. / 9x + 7 / = 11 B.Find the solution set of the inequality and write it with interval notation. Show the solution set on the real number line. 11. 3x – 2 ≤ 20 12. x2 < 64 13. 2x + 1 > 1 x – 5 14. 3x – 4 ≤ 12 2x – 3 15. 2x2 – 3 x ≥ 5 16. 5x + 8 ≥ 2x – 2 17. -7 ≤ 8 – 4x ≤ 10 -5 18. / x – 10 / ≤ 17 19. x2 + 3x – 10 ≥ 1 20. / 4x – 5 / < 15 C.Solve for x in terms of the other symbols 21. d/10x – d/5 = 1/x 22. rsx2 + s2 x + rtx + st = 0 23. x2 + b2 = 2 bx + a2 24. 5x2 – 4xy – x + y -1 = 0 25. x2 + xy + 4x - 1 = 0 Show that the two inequalities are equivalent.
  • 38. 26. A woman leaves home at 10 A.M and walks to her office at he rate of 7 km/hr. At 10. 15 A.M. the womans daughter leaves home and rides her bicycles at a rate 15 km/hr along the same route to school. At what time does she overtake her mother? 27. How many liters of a solution that is 85 percent glycerine should be added to 30 liters of a solution that is 40 percent glycerine to give a solution that is 60 percent glycerine. 28. In a long distance race around a 600 m track the winner finished one lap ahead of the loser. If the average rate of the winner was 8 m/sec and the average rate of the loser was 7. 25 m/s how soon after the start did the winner complete the race? 29. An automobile radiator contains 9 liters of a solution that is 10 percent antifreeze and 95 percent water. How much of the solution should be drained and replaced with pure antifreeze to obtain a solution that is 45 percent antifreeze? 30. The perimeter of a rectangle must not be greater than 50 cm and the length must be 10 cm. What is the range of values for the width? 38 TEST II A. Find the solution set of the equation. 1. X2 = 64 2. 6t2 – 11 = 0 3. 9x2 = x 4. 3t/3t + 4 + 2/5 – 1/3t – 4 5. 64y2 – 80y + 25 = 0 B. Find the solution set of the equation by completing the square. 6. X2 + 6x + 8 = 0 7. 4x2 + 4x – 3 = 0 8. 3y2 + 4y + 2 = 0 C. Find the solution set of the equation by using the quadratic form. 9. X2 + 1 = 6x 10. 5y2 – 4y – 2 = 0 D.Find the solution of the inequality and write it with interval notation. 1. 2x – 1 < 6 2. -3 > 4x + ¾ 3. 11 ≥ 5x – 4 > 1 4. 2x – 4 < 6 5. 1 < 4x – 1/3 < 5 6. 5x – 2 < 5x – 2/4 7. 10 – 3x > 4x – 5/-3 8. 5 ≤ 3x – 4 < 14 9. -1 < 7 – 2x/5 ≤ 5 10. 6 ≤ 2 – x ≤ 8 E.Find the solution set of the equation. 11. / x – 8 / = 9 12. / 4y – 10 / = 5
  • 39. 39 13. / 3t -2 / = t2 14. / x – 1 / = x2 15. / 5t – 15 / = 20 CHAPTER III Points and Equations
  • 40. 40 3.1 Points in a Plane Ordered pairs of real numbers are important in our discussions. Any two real numbers form a pair. When the order of appearance of the numbers is a significant, we call it an ordered pair. If x is the first real number and y is the second, this ordered pair is denoted by writing them in parenthesis with a comma separating them as (x,y). The set of all ordered pairs of real numbers is called the number plane, denoted by R2 , And each ordered pair (x,y) is a point in the number plane. The intersection of these two perpendicular line segments is the point P, associate with the ordered pair (x, y). Refer to figure 1, the first number x of the pair is called the abscissa or x-coordinate of P, and the second number y is called the ordinate or the y-coordinate of P. The x and y axes are called the coordinate axes. They divide the plane into four parts called quadrants. The first quadrant is the one in which the abscissa and the ordinate are both positive, that is , the upper right quadrant. The other quadrant are numbered in the counterclockwise direction, with the fourth being the lowered right quadrant. Pythagorean Theorem In a right triangle, if a and b are the lengths of the perpendicular sides and c is the length of the hypotenuse then, a2 + b2 = c2. Distance Formula The distance between two points P1 (x1 , y1) and P2 ( x2 , y2) is given by P1P2 = √(x2 – x1)2 + (y2 –y1)2 Converse of Pythagorean Theorem If a ,b and c are the length of the side of a triangle and a2 + b2 = c2, then the triangle is a right triangle, and c is the length of the hypotenuse. Midpoint Formulas If M (x,y) is the midpoint of the line segment from P1 (x1 , y1) to P2 ( x2 , y2), then x = x1 + x2 y = y1 + y2 2 2 Solution From the midpoint formulas, if M is the point (x,.y), then x = 5 – 1 y = -3 + 6 2 2 = 2 = 3/2 3.2 Graphs of Equations The graph of an equation in R2 is the set of all points in R2 whose coordinate are numbers satisfying the equation. Example 1 Draw a sketch of the graph of the equation y2 – 4x = 0 Solution
  • 41. y2 = 4x y = ± 2√x y = 2 √x and y = -2 √x 41 Symmetry of two points Two points P and Q are said to be a symmetric with respect to a line if and only if the line is the perpendicular bisector of the line segment PQ. Two points P and Q are said to be symmetric with respect to a third point if and only if the third point is the midpoint of the line segment PQ. Symmetry of a graph The graph of an equation is symmetric with respect to a line l if and only if for every point P on the graph there is a point Q, also on the graph , such that P and K are symmetric with respect to l. The graph of an equation is symmetric with respect to a point R if and only if for every point P on the graph there is a point S, also on the graph such that P and S are symmetric with respect to R. Example 2 Draw a sketch of the graph of the equation. 2y = x3 2(-y) = (-x)3 -2y = -x3 Circle A circle is the set of all points in a plane equidistant from a fixed point. The fixed point is called the center of the circle and the constant equal distance is called the radius of a circle. Equation of a Circle An equation of the circle with center at the point (h,k) and radius r is (x – h)2 + (y – k)2 = r2 Example 3 Find an equation of the circle having the diameter with endpoints at A (-2,3) and B (4,5). Solution ℎ = −2 + 4 2 = 1 푘 = 3+5 2 =4 3.3 Equation of a line Slope If P1 (x1 , y1) and P2 (x2 , y2) are any two distinct points on line l, denoted by m, is given by m = y2 – y1 x2 – x1 If the slope of a line is positive, then as the abscissa of a point on the line increases, the ordinate increases. A line whose slope is negative, for this line as the abscissa of the point on the line increases, the ordinate decreases. If a line is parallel to the x axis, then y2 = y1 ; so the slope of the line is zero. If a line is parallel to the y axis, x2
  • 42. = x1 ; thus the fraction y2 – y1 / x2 – x1is meaningless because we cannot divide by zero. Thus the slope of a vertical line is not defined. 42 Example 1 Determine the slope of a line. A (3,7) and B (-2,-4) m = -4 – 7 /-2 -3 = -11 / -5 = 11 / 5 A (-3,4) and B (5,4) m = 4 – 4 / 5 – (-3) = 0/8 = 0 The point slope form we choose the particular point (0,b) (that is , the point where the line intersects the y axis ) for the point ( x1,y1) , we have y - b = m ( x – 0 ) y = mx + b The number b, the ordinate of the point where the line intersects the y axis, is the y intercept of the line. Consequently, the preceding equation is called the slope-intercept form. Theorem 1 The graph of the equation Ax + By + C = 0 Where A, B, and C are constants and where not both A and B are zero is a line. Theorem 2 If l1 and l2 are two distinct nonvertical lines having slopes m1 and m2, respectively, then l1 and l2 are parallel if and only if m1 = m2 Theorem 3 Two nonvertical lines l1 and l2, having slopes m1 and m2 , respectively, are perpendicular if and only if m1m2 = - 1
  • 43. ASSESSMENT TEST I A.Find the center and radius of the circle. 43 1. x2 + y2 + 4x – 6y -3 = 0 2. 3x2 + 3y2 + 4x -4 = 0 B. Determine the slope of the line and find an equation of the line. 3. (1,-3) and (4,5) 4. (2,5) and (6,9) 5. (9,8) and (-2,4) 6. (6,2) and (-9,7) 7. (4,2) and (-7,8) 8. (-1,3) and (6,1) 9. (4-6) and (5,6) 10. (9,7) and (6,-4) C.Find the slope and y-intercept of the line having the given equation . 11. 2x – 5y - 10 = 0 12. 2x + 3y + 12 = 0 13. Prove that the quadrilateral having vertices at (2,1) , (6, -2) , (9,6) and (7, 10) is a rectangle. 14. Find an equation of the line through the point ( -1,6) and perpendicular to the line whose equation is 4x + 2y – 5 = 0. 15. Prove that the lines having the equations 2x + 5y + 20 = 0 and 5x – 2y - 10 = 0 are perpendicular. D. For the given points A and B , find the directed distances : (a) AB : (b) BA 16. A (2,4) and B (7,8) 17. A (4, - 8) and B (5, - 10) 18. A (7, - 10) and B (- 8, 7 ) 19. A (- 4, 5 ) and B (10,3) 20. Given that A is the point (-2, 3) and B is the point ( -x , 3) , find x such that (a) AB = -8 ; (b) BA = -8 21. Given that A is the point (-4, y) and B is the point (-4, 3) , find y such that (a) AB = -3 ; (b) BA = -3 22. A (10,-5) , B (-4, 6) , C (-2, 9) 23. A (4,10) , B (7, -3) , C (-1, -1) E. Find the center and radius of the circle. 1. 2x2 + 2y2 – 2x + 2y + 7 = 0 2. x2 + y2 – 10x – 10y + 25 = 0 3. 3x2 + 3y2 + 4y – 7 = 0 4. x2 + y2 - 6x - 8y + 9 = 0
  • 44. 5. x2 + y2 + 2x + 10y + 18 = 0 F. Find an equation of the line satisfying the given condition. 6.(a) the slope is 4 and through the point ( -3 , 2) (b) through the two points ( -2, -6) and ( 4,5 ) 7 (a) the slope is -7 and through the point (-5,4) 44 (b) through two points (5,7) and (-6,9) 8. (a) through the point ( 2,-8) and parallel to the x-axis (b) through the point (3,5) and parallel to the y-axis G. Find the slope and y-intercept of the line having the given equation. 9. (a) x + 3y -6 = 0 ; (b) 4y – 9 = 0 10. (a) x – 4y -2 ; (b) 4x = 3y
  • 45. Chapter IV Functions and their Plane 45
  • 46. 46 4.1 Functions A function can be thought of as a correspondence from a set x of real numbers x to a set y of real numbers y, where the number y is unique for a specific value of x. A function is asset of ordered pairs of real numbers (x , y) in which no two distinct ordered pairs have the same first number. The set of all admissible values of x is called the domain of the function , and the set of all resulting values of y is called the range of the function. Graph of a function If f is a function , then the graph of f is the set of all points (x , y) in R2 for which (x , y) is an ordered pair in f. The graph of a function can be intersected in a vertical line at most one point. Example 1 The function h is defined by h{ (x , y) y = / x/ } The required formula of h, x can be any real number. Therefore the domain is ( - ∞ , + ∞ ). Because we observe from figure 1 that y can be any non negative number the range is [ 0 , + ∞ ] Quadratic Function The general quadratic function is defined by F ( x ) = ax2 + bx + c Where a, b and c are constants representing real numbers and a≠0. The graph of f is the same as the graph of an equation Y = ax2 + bx + c If the function f is the defined by F ( x ) = -2 x2 + 8x – 5 The graph of f is the same as the graph of an equation Y = 2x2 + 8x – 5 This equation is equivalent to 2(x2 – 4x) = -y – 5 To complete the square of the binomial with the parentheses, we add 2(4) to both sides of the equation and we have 2(x2 – 4x +4) = -y – 5 + 8 (x – 2 )2 = 4p (y – k) Where (h,k) is (2,3) and = -1/8. Therefore the vertex of the parabola is at 92,3) and the axis is the line x = 2. Because p<0 , the parabola opens downward we find a few points on the parabola. The zeros of the function are the values of x for which f(x) = 0. When the graph of a quadratic function opens upward the function has a minimum value, which occurs at the vertex of the parabola. There is no maximum value for such a
  • 47. function. When the parabola opens downward. The function has a maximum value occurring at the vertex ;it has no minimum value We now apply the method used in the solution to the general quadratic function defined by 47 f (x) = ax2 + bx + c -1 = ax2 + bx + c ax2 + bx = y – c a (x2 +b/a x) = y – c a (x2 + b/a x + b2/4a2 0 = y – c +b2/4a (x = b/2a )2 = 1/a ( y+b2 – 4ac/4a) The graph of this equation is a parabola having its vertex at the point where x = -b/2a. if a>0, the parabola opens upward and so f has a minimum value at the point where x + - b/2a Theorem 1 The quadratic function defined by f (x) = ax2 +bx +c, where x = -b/2a. if a>o, the extreme value is a minimum value, and if a,a0, the extreme value is a maximum value Example use theorem 1 to find either a maximum or minimum value of the function g if g (x) = -5/2 x2 + 8x -10 For the given quadratic function a = -5/2 and b =8. Because a<0, g has a maximumu value at the point x = -b/2a = -8/2(-5/2) = 8/5 The maximum value is G (8/5) = -5/2(8/5)2 + 8(8/5) – 10 =-5/2 (64/25) + 64/5 – 10 = -18/5 Rational Functions A rational function is of the form 푓(푥) = 푝(푥) , where p(x) and q(x)are polynomial 푞(푥) functions and 푞(푥) ≠ 0. A graphing calculator is a good tool for exploring graphs of rational functions. Graphs of rational functions may have breaks in continuity. This means that, unlike polynomials functions which can be traced with a pencil that never leaves the paper, a rational function may not be traceable. Breaks in continuity can occur where there is a vertical asymptote or point discontinuity. Point of discontinuity is like a hole in the graph. Vertical asymptote and point of discontinuity occur for the values of x that make the denominator of the rational function zero. Graphing Rational Functions
  • 48. 48 Connection: Mathematical History Mathematician Maria Gaetana Agnesi was one of the greatest women scholars of all time. In the analytic geometry section of her book Analytical Institutions, Agnesi discussed the characteristics of the equation 푥 2푦 = 푎2 (푎 − 푦), called the “curve of Agnesi”. The equation can be expressed as 푦 = 푎2 푥2+푎2. Because the function described above is the ratio of two polynomial expression a3 and 푥 2 + 푎2 is called a rational function. A rational function is function of the form 푓 (푥) = 푝(푥) , where p(x) and q(x) are polynomial functions and q(x)≠ 0 푞(푥) Examples of Rational Function: 풇(풙) = 푥 푥− 1 푔(푥) = 3 푥−3 ℎ(푥) = 푥 +1 (푥+2) (푥−5) The lines that graph of the rational function approaches is called Asymptote. If the function is not define when 푥 = 푎, then either there is a “hole” in the graph 푥 = 푎. POLYNOMIALS The expression x2+2xy+y2 is called a polynomial. A polynomial is a monomial or a sum of monomials. The monomials that make up the polynomial are called the terms of the polynomial. The two monomials xy and xy ca be combined because they are like terms. Like terms are two monomials that are the same, or differ only by their numerical coefficient. An expression like m2+7mb+12cd with three unliked terms is called trinomial. An expression like xy+b3 with two unliked terms is called binomials. The degree of a polynomial is the degree of the monomial with the greatest degree. Thus, the degree of x2+2xy+y2 is 2.
  • 49. Remember: If a polynomial contains only one term, it called monomial; if two terms, it is called binomial; if it is contains three terms it is called trinomial. If a polynomial has more than three terms, it is called multinomial. The following table shows examples of polynomials. Polynomial No. of terms Class by terms 12푥푦2 5푥푦 + 3푧2 푣 49 4푥 2푦 − 3푥푦2 − 푥푦 3푥 − 5푦 + 푎 + 10푏 5푥 4 + 2푥 3 − 푥 2 − 6푥 + 8 1 2 3 4 5 Monomial Binomial Trinomial Multinomial Multinomial Example: Determine whether or not each expression is a polynomial. Then state the degree of each polynomial. a. 2 7 x4y3 – x3 This expression is a polynomial. The degree of the first term is 4 + 3 or 7, and the degree of the second term is 3. The degree of the polynomial is 7. b. 9 + √푥 − 3 This expression is not polynomials because √푥 is not a monomial. The FOIL Method is an application of the distributive property that make the multiplication easier. FOIL Method of Multiplying Polynomial The product of two binomial is he sum of the products of: F the first terms O the outer terms I the inner terms L the last terms
  • 50. 50 Example: Find (k2 +3k +9) (k +3) (k2 +3k +9) (k +3) = 푘2 (k+3) + 3k (k +3) +9(k +3) distributive property =k2∙k+k2∙ 3 + 3푘 ∙ +9 ∙ 푘 + 3 ∙ 9 distributive property = 푘2∙ 푘 + 푘2∙ 3푘2+9푘 + 9푘 + 27 =k2+6푘2+18푘 + 27 combined like terms Dividing Polynomials You can use a process similar to long division of a whole numbers to divide a polynomial by a polynomial when doing the division, remember that you can only add ad subtract like terms. Example: Simplify: c2 –c –30 c –6 c 푐 − 6√푐2 − 푐 − 30 푐2− 6 5푐 −30 −푐 − (−6푐) = −푐 + 6푐 표푟 5푐 푐 + 5 푐 − 6√푐2 − 푐 − 30 푐2−6 5푐 −30 5푐 −30 0 Polynomial functions A polynomial function is a function that can be defined by evaluating a polynomial. A function f of one argument is called a polynomial function if it satisfies 푓(푥) = 푎푛푥 푛 + 푎푛−1푥 푛− 1 + ⋯ + 푎2푥 2 + 푎1푥 + 푎0 For all arguments x, where n is a non-negative integer and a0, a1, a2, ..., an are constant coefficients. For example, the function f, taking real numbers to real numbers, defined by 푓(푥) = 푥 3 is a polynomial function of one variable. Polynomial functions of multiple variables can also be defined, using polynomials in multiple indeterminates, as in 푓(푥, 푦) = 2푥 3 + 4푥 2푦 + 푥푦5 + 푦2 − 7 An example is also the function 푓(푥) = cos(2푎푟푐표푠(푥)) which, although it doesn't look like a polynomial, is a polynomial function on [−1,1] since for every from [−1,1] it is true that 푓(푥) = 2푥 2 −1 Polynomial functions are a class of functions having many important properties. They are all continuous, smooth, entire, computable, etc
  • 51. Graphs of Polynomial Function A polynomial function in one real variable can be represented by a graph. 51  The graph of the zero polynomial f(x) = 0 is the x-axis.  The graph of a degree 0 polynomial f(x) = a0, where a0 ≠ 0, is a horizontal line with y-intercept a0  The graph of a degree 1 polynomial (or linear function) f(x) = a0 + a1x , where a1 ≠ 0, is an oblique line with y-intercept a0 and slope a1.  The graph of a degree 2 polynomial f(x) = a0 + a1x + a2x2, where a2 ≠ 0 is a parabola.  The graph of a degree 3 polynomial f(x) = a0 + a1x + a2x2, + a3x3, where a3 ≠ 0 is a cubic curve.  The graph of any polynomial with degree 2 or greater f(x) = ao + a1x + a2x2 + ... + anxn , where an ≠ 0 and n ≥ 2 is a continuous non-linear curve.  The graph of a non-constant (univariate) polynomial always tends to infinity when the variable increases indefinitely (in absolute value) Polynomial graphs are analyzed in calculus using intercepts, slopes, concavity, and end behavior. Polynomial of degree 2: f(x) = x2 − x − 2 = (x + 1)(x − 2) Polynomial of degree 3: f(x) = x3/4 + 3x2/4 − 3x/2 − 2 = 1/4 (x + 4)(x + 1)(x − 2)
  • 52. 52 Polynomial of degree 4: f(x) = 1/14 (x + 4)(x + 1)(x − 1)(x − 3) + 0.5 Polynomial of degree 5: f(x) = 1/20 (x + 4)(x + 2)(x + 1 )(x − 1)(x − 3) + 2 Polynomial of degree 2: f(x) = x2 − x − 2 = (x + 1)(x − 2) Polynomial of degree 3: f(x) = x3/4 + 3x2/4 − 3x/2 − 2 = 1/4 (x + 4)(x + 1)(x − 2)
  • 53. 53 Polynomial of degree 4: f(x) = 1/14 (x + 4)(x + 1)(x − 1)(x − 3) + 0.5 Polynomial of degree 5: f(x) = 1/20 (x + 4)(x + 2)(x + 1 )(x − 1)(x − 3) + 2 Inverse function Definition of Inverse Function Two function f and g are inverse function if and only if both of their compositions are the identity function. That is, (푓 ∘ 푔) = 푥 and (푔 ∘ 푓)(푥) = 푥 An inverse function is a function that "reverses" another function: if the function f applied to an input x gives a result of y, then applying its inverse function g to y gives the result x, and vice versa. i.e., f(x) = y if and only if g(y) = x. A function f that has an inverse is said to be invertible. When it exists, the inverse function is uniquely determined by f and is denoted by f −1, read f inverse. Superscripted "−1" does not, in general, refer to numerical exponentiation. In some situations, for instance when f is an invertible real-valued function of a real variable, the relationship between f andf−1 can be written more compactly, in this case, f−1(f(x)) = x = f(f−1(x)), meaning f−1 composed with f, in either order, is the identity function on R.
  • 54. 54 Property of Inverse Function Suppose 푓 and푓−1 are inverse function. Then 푓(푎) = 푏 and only if 푓−1(푏) = 푎 Definition of inverse Relationship Two relationships are inverse relationship if and only if whenever one relation contains the element ( a, b ), the other relation contains the element ( b, a )
  • 55. ASSESSMENT TEST I. A. Draw a sketch of the graph of the function and determine its domain and range. 55 1. f = {(x,y) / y = 3x – 1 } 2. F = { ( x, y / y = 2x2 } 3. F = { ( x, y) / y = / 3x + 2 / } 4. G = { ( x, y} = x2 – 4 } x-2 5. g = { ( x, y /y = (x2 – 4 ) ( x – 3 ) } x2 – x – 6 6. f : y = { -2 if x ≤ 3 } { 2 if 3 < x } 7. g : y = { -4 if x< -2 } {-1 if -2 ≤ x ≤ 2 } { 3 if 2 < x } 8. f : y = { 3x +2 if x ≠ 1 } { 8 if x = 1 } 9. G : y { 9 – x2 if x ≠ -3 } { 4 if x = -3} 10. G : y = { x2 – 4 if x < 3 } {2x – 1 if 3 ≤ x } B. Find the zeros of the function. 1. f(x) = x2 – 2x – 3 2. f(x) = 2x2 – 2x – 1 3. f(x) = x2 – 3x + 1 4. f(x) = 6x2 – 7x – 5 C. Use theorem 1 to find either maximum or minimum value of the function. 5. f(x) = -1/2 ( x2 + 8x + 8) 6. G (x = 1/8 ( 4x2 + 12x -9 ) 7. g (x) = 3x2 + 6x + 9 8. f(x) = 2 + 4x - 3x2 9. Find two numbers whose sum difference is 10 and whose product is a minimum. 10. Find two numbers whose sum is 20 and whose product is a maximum.
  • 56. Chapter V Exponential and Logarithmic Function 56 c
  • 57. 57 EXPONENTIAL FUNCTION An equation of the form 푦 = 푎 ∙ 푏푥 , 푤ℎ푒푟푒 푎 ≠ 0, 푏 > 0, and 푏 ≠ 1, ia called exponential function with base b. The logarithm of a number is the exponent to which another fixed value, the base, must be raised to produce that number. Logarithms are exponents. They were once used t simplifies calculations, but the advent of calculators and computers caused calculation with logarithms to be used less and less. Definition of Logarithm Suppose 푏 > 0 and 푏 ≠ 1. for 푛 > 0, there is a number p such that 퐿표푔푏 푛 = 푝 if and only if 푏푝 = 푛. The chart below shows some equivalent exponential and logarithmic equations. Exponential Equation Logarithmic Equation 52 = 25 105 = 100,000 80 = 1 2−4 = 1 16 9 1 2 = 3 log5 25 = 2 log10 100,000 = 5 log8 1 = 0 log2 1 16 = −4 log9 3 = 1 2 Integral Exponents Basic Laws of Exponents 1. 풃풙 ∙ 풃풚 = 풃풙+풚 2. 풃풙 풃풚 = 풃풙−풚 (풃 ≠ ퟎ) 3. 풊풇 풃 ≠ ퟎ, ퟏ, −ퟏ 풕풉풆풏 풃풙 = 풃풚 풂풏풅 풐풏풍풚 풊풇 풙 = 풚 4. (풂풃)풙 퐜퐱 퐛퐲 5. (풂 풃 ) 풙 = 풂풙 풃풙 (풃 ≠ ퟎ) 6. 풊풇 풙 ≠ ퟎ, 풂 > ퟎ, 풃 > ퟎ, 풕풉풆풏 풂풙 = 풃풙 풊풇 풂풏풅 풐풏풍풚 풊풇 풂 = 풃 7. (풃풙)풚 = 푩풙풚 8. 풃ퟎ = ퟏ (풃 ≠ ퟎ) 9. 풃−풙 = ퟏ 풃풙 Each of the above rules should be familiar to you from algebra I. Here are some sample problems with their solutions. 1) Watch the difference between these two: a) (-3)-2 b) -(3)-2 The first one is squaring a negative number and the second is squaring a positive number and then making the whole result negative. a) = 1/(-3)2 = 1/9 b) 1/-(3)2 = -1/9 c) 7 . 2-3 = d) (7 . 2)-3 = The first one raises the power then multiplies, while the second one multiplies first
  • 58. 58 then raises the power. c) = 7/8 d) = 14-3 = 1/143 = 1/2744 e) (3−2 + 3−3)−1 = = ( 1 9 + 1 27 ) −1 = ( 3 27 + 1 27 ) −1 = ( 4 27 ) 1 = 27 4 In the above example our first step is to work inside the grouping symbols and get a common denominator. Then add the two fractions. Only when you have a single fraction, is it permitted to invert the fraction. f) (푎−2 − 푏−2)−1 = ( 1 푎 −2 − 1 푏2) −1 = ( 푏2 푎2 푏2 − 푎2 푎2푏2) −1 = (푏2−푎2 푎2 푏2 ) −1 = 푎2 푏2 (푏−푎)(푏+ 푎) In the above example we again simplify inside the grouping symbols and get a common denominator. Once we have a single fraction in step 3 we can invert the fraction. Notice the factoring in the last step! g) (3푎−1) 2 (3푎−1)−2 = (3푎−1 )4 = 81푎−4 = 81 푎4 In this example, the numerator and denominator have the same base. We can apply the division rule by subtracting the exponents. Then simplify. Remember, no negative exponents should be left in the answer. An important type of rule can now be stated using exponents. It is a growth or decay problem. We can mathematically model this function by using the following: A(t) = Ao(1 + r)t Where Ao is the initial amount at time t = 0 r is the rate (as a decimal) t is the time A(t) is the amount after the time t. If r > 0, then it is an exponential growth. If -1 < r < 0, then it decays exponentially.
  • 59. 1) Suppose a bike costs $100 now and it increases at a rate of 5% per year. What will be the cost of the bike in 4 years? 59 Solution: Ao = 100, r = 5% = .05, t = 4 A(4) = 100(1 + .05)4 = 100(1.05)4 = 121.55 The bike will cost $121.55 in 4 years. (Rounded to the nearest penny) 2) Suppose a car is worth $15,000 new. What will it be worth in 3 years if it decreases at a rate of 12% per year? Solution: This is a decrease problem with Ao = 15000, r = -.12, t = 3 A(3) = 15000(1-.12)3 = 15000(.88)3 = 10222.08 The car will be worth $10222.08 in 3 years. 1). 4−4 4−2 + 4−3 = 4−4 4−2+4−3 ∙ 44 44 = 1 42 + 41 = 1 16 +4 = 1 20 One of the easier ways to do this problem is to multiply both numerator and denominator by the positive power of the biggest negative exponent. In this case we multiplied by 44. This greatly simplifies the problem. Rational Exponents All rules presented in the previous section were defined for integers only. All of the properties in the last section can also be extended to include rational exponents according to the following definitions: 1) 푏 1 푛 = √푏 푛 2)푏 푚 푛 = ( 푛√푏) = 푛√푏푚 Example: 1) 8 1 3 =3√8=2 2) 2) 8− 1 3 = 1 = 1 3√8 2 3 = 23 + 8 3) 163/4 = ( √16 4 ) 3 4 = 1 4) 16− (4√16 ) 3 = 1 23 = 1 8 5) ( 8 27 ) 1 3 = √ 8 27 3 = 2 3
  • 60. 60 6) ( 8 27 ) − 2 3 = (27 8 ) 2 3 = ( √27 8 3 2 = (3 ) 2 ) 2 = 9 4 7) (1001/2 - 361/2)2 = (10 - 6)2 = 42 = 16 8) x1/2(x3/2 + 2x1/2) = x2 + 2x 9) ( √4푎2) 3 2 6√4푎2 = (4푎2 ) 2훽 (4푎2) 1 6 1 6 = (4푎2) = (4푎2)2훽− 1 2 = 2푎 We can use these rules to solve for x when x is the exponent. This method will only work if the bases are the same. Check back in section 5-1 for the appropriate rule!! Example: 1) 16x = 25 We can write both sides in base two. 24x = 25 Now use the fact that the bases are the same, the exponents are = 4x = 5 And solve for x!! x = 5/4 To check it, take the 5/4 root of 16 = 25!! 2) 271-x = (1/9)3-x You need to make both bases the same. How about 3!! 33(1-x) = 3-2(3-x) Notice the power on the right side is negative. 3(1 - x) = -2(3 - x) Because the bases are =, the exponents must be = 3 - 3x = -6 + 2x Solve for x. 3 = -6 + 5x 9 = 5x 9/5 = The growth and decay formula can also be used with rational numbers. Consider the following: 1) The cost of a computer has been increasing at 7% per year. If it costs $1500 now, find the cost: a) 2 years and 6 months from now b) 3 years and 3 months ago. Solutions: a) Ao = 1500, r = .07 and t = 2.5 A(2.5) = 1500(1 + .07)2.5 = 1500(1.07)2.5 = 1776.44 b) Ao = 1500, r = .07, and t = -3.25 A(-3.25) = 1500(1.07)-3.25 = 1203.91
  • 61. Exponential Functions Any function in the form f(x) = abx, where a > 0, b > 0 and b not equal to 1 is called an exponential function with base b. Let's take a look at a couple of simple exponential graphs. f(x) = 2x X f(x) 3 8 2 4 1 2 0 1 -1 1/2 -2 1/4 -3 1/8 Notice the domain is all real numbers and the range is y > 0. As x gets larger (right), y gets very large. As x gets smaller(left), y approaches zero asymptotically. Notice also that the graph crosses the y-axis at (0, 1). The above is the general shape of an exponential with b > 1. This is an example of exponential growth. 61 Now let's look at the graph of f(x) = (1/2)x X f(x) 3 1/8 2 1/4 1 1/2 0 1 -1 2 -2 4 -3 8
  • 62. Observe that this graph is the reflection about the y-axis of the first graph. The domain is still all real numbers and the range is y > 0. The y-intercept is (0, 1). This is the general form of an exponential graph if 0 < b < 1. It is an example of anexponential decay. Look at the following graphs that illustrate the general properties of exponentials. 62
  • 63. 63 Do you see the similarities of each graph? How about this one? Many of the functions associated with exponential growth or decay are functions of time. We have already had one form: A(t) = Ao(1 + r)t A second form looks like: A(t) = Aobt /k where k = time needed to multiply Ao by b Rule of 72 If a quantity is growing at r% per year then the doubling time is approximately 72/r years. For example, if a quantity grows at 10% per year, then it will take 72/10 or 7.2 years to double in value. In other words, it will take you 7.2 years to double your money if you put it into an account that pays 10% interest. At the current bank rate or 2%, it will take you 72/2 or 36 years to double your money!! Boy, jump all over that investment!! Sample Problems 1) Suppose you invest money so that it grows at A(t) = 1000(2)t /8 a) How much money did you invest? b) How long will it take to double your money? Solutions: a) The original amount in the formula is $1000. b) This means what time will it take to get $2000. 2000 = 1000(2)t /8 2 = 2t /8 1 = t/8 8 = t It will take 8 years to double your money!!
  • 64. 2) Suppose that t hours from now the population of a bacteria colony is given by: P(t) = 150(100)t /10 a) What is the initial population? b) How long does it take for the population to be multiplied by 100)20/10 = 64 150(100)2 = 1,500,000 c) What is the population at t = 20? Solutions: a) It is 150 from the original equation. b) It takes 10 hours. That's the definition of the exponential function. c) P(20) = 150() The half life of a substance is 5 days. We have 4 kg present now. a) Write a formula for this decay problem. b) How much is left after 10 days? 15 days? 20 days? Solutions: a) A(t) = 4(1/2)t /5 b) A(10) = 4(1/2)10/5 = 4(1/2)2 = 1 kg. A(15) = 4(1/2)15/5 = 4(1/2)3 = 1/2 kg. A(20) = 4(1/2)20/5 = 4(1/2)4 = 1/4 kg. 3) The value of a car is given by the equation V(t) = 6000(.82)t a) What is the annual rate of depreciation? b) What is the current value? c) What will be the value in three years? Solutions: a) It is 1 - .82 or .18 = 18% b) The current value is given in the formula, $6000. c) V(3) = 6000(.82)3 = 3308.21 Which is $3308.21 The number e and the function ex Definition of the irrational number e
  • 65. Without getting into a discussion of limits right now, we can get an idea of what's happening by taking increasingly larger values of n. We will talk about limits later on in the year. Study the following table of values and use your calculator to double check the results: 65 N (1 + 1/n)n 10 2.593742 100 2.704814 1000 2.716924 10,000 2.718146 100,000 2.718268 1,000,000 2.718280 If you study this chart, you see that the number e approaches a value of 2.718 . . . The function ex is called the natural exponential function. The graph of ex and e-x are graphed below: Notice, they fit the pattern of the previous section. The number e appears in many applications of physics and statistics. We will take a close look at the number e and how it relates to compound interest. Compound Interest Formula 퐴(푡) = 퐴0 (1 + 푟 푛 ) 푥푡 Where: A(t) = amount after time t. Ao = Initial amount r = rate in decimal n = number of times compounded in a year. t = time in years Thus, if the interest was paid semiannually, n = 2. Paid quarterly would make n = 4, Paid monthly, n = 12, etc.
  • 66. Sample Problems 1) Find the value of a $1 if it is invested for 1 year at 10% interest compounded quarterly. Solution: Initial amount is $1 with r = .10, n = 4 and t = 1. A(1) = 1(1 + .10/4)4 = 1.1038 This means that at the end of a year, each dollar invested in worth 1.1038 or slightly more than $1.10. The effect of compounding adds another .0038 % to the interest rate. Thus the effective annual yield is 10.38%. 2) You invest $5000 in an account paying 6% compounded quarterly for three years. How much will be in the account at the end of the time period? Solution: Initial amount is $5000, with r = .06, n = 4 and t = 3 A(3) = 5000(1 + .06/4)12 = $5978.09. This account pays $978.09 in 66 interest over the three years. 3) What is the effective annual yield on $1 invested for one year at 15% interest compounded monthly? Solution: Initial amount $1, with r = .15, n = 12 and t = 1 A(1) = 1(1 + .15/12)12 = 1.1608. The effective annual yield is 16.08%. This is a relatively big increase because of the number of times compounded in the year. The above problems all had one thing in common. The number of times compounded was a finite number. We can also havecontinuous compounding. That is, compounding basically every second on the second. This would be rather cumbersome to calculate because the compounding is extremely large. We can use a similar formula if the compounding is continuous. P(t) = Poert Notice the appearance of the number e. If you look closely at the compound interest formula, you will see imbedded the definition of the number e. Only use this formula if you are sure the compounding is continuous. Problems 1) Php. 500 is invested in an account paying 8% interest compounded continuously. They leave it in the account for 3 years. How much interest is accumulated? Solution: Initial amount Php 500, with r = .08 and t = 3. P(3) = 500(e.08(3)) = 635.62. This means the interest is Php.135.62. 2) A population of insects rapidly increases so that the population after t days from now is given by A(t) = 5000e.02t . Answer the following questions: a) What is the initial population? b) How many will there be after a week? c) How many will there be after a month? (30 days) Solutions: a) The initial population is 5000 from the formula.
  • 67. 67 b) A(7) = 5000e.14 = 5751 c) A(30) = 5000e.6 = 9111 Logarithmic Functions Common Logarithm Demo: Log Funtion Applet log x = a if and only if 10a = x The important thing to remember is the log represents the exponent. In the case of common logs, the base is always base 10. Study the following examples. 1) log 100 = 2 because 102 = 100. 2) log 1000 = 3 because 103 = 1000. 3) log 1 = 0 because 100 = 1. 4) log .1 = -1 because 10-1 = .1 5) log .01 = -2 because 10-2 = .01 The log function is the inverse function of the exponential function and as such their graphs are reflections about the y = x line. Here is the graph of the common log and the inverse. Some important facts you need to understand from the log graph. The domain of the log is x > 0. The range is all real numbers. The zero is at x = 1. You can only find the log of positive numbers. Logs of numbers less than one are negative and logs of numbers greater than one are positive. We can find the log of other bases by using the following formula similar to the common log definition. logb x = n if and only if x = bn. Here are some examples: 1) log2 8 = 3 because 23 = 8 2) log3 81 = 4 because 34 = 81. 3) log4 1/16 = -2 because 4-2 = 1/16 4) log8 1 = 0 because 80 = 1
  • 68. One of the most important log function is called the natural log which has the number e as the base. When e is used as a base we use the following notation: ln x = a if and only if ea = x Most natural logs need to be calculated on your calculator. The graph of the natural log is shown below: 68 Solving Simple Log Equations 1) Log x = 3 Solution: To solve an equation of this type, rewrite the equation in exponential form. x = 103 = 1000 2) Log |x| = 2 Solution: To solve an equation of this type, again rewrite the equation in exponential form and solve for x. |x| = 102 = 100 x = 100 or -100 3) Log (x2 + 19) = 2 Solution: Again, rewrite as an exponential equation and solve for x. x2 + 19 = 102 x2 + 19 = 100 x2 = 81 x = 9 or -9 4) Log x = .3 Again, rewrite exponentially. x = 10.3 Use your calculator and round to hundredths. x = 2.00
  • 69. 69 5) Ln x = -1.2 Solution: Same as above. x = e-1.2 x = .3 Laws of Logarithms 1) Logb MN = Logb M + Logb N 2) Logb M/N = Logb M - Logb N 3) Logb M = Logb N if and only if M = N 4) Logb Mk = k Logb M 5) Logb b = 1 6) Logb 1 = 0 7) Logb bk = k 8) bLogb x = x Sample problems Write each log in expanded form. 1) Log5 xy2 = Solution: Log5 x + Log5 y2 = Log5 x + 2 Log5 y 2) Log7 (xy/z2) = Solution: Log7 x + Log7 y - 2 Log7 z 3) 퐿표푔8√푥푦 = 퐿표푔8(푥푦) 1 2 = 1 2 (퐿표푔8푥 + 퐿표푔8푦) Express each as a single log. 1) Log x + Log y - Log z = Solution: Log (xy)/z 2) 2 Ln x + 3 Ln y = Solution: Ln x2y3 3) (1/2) Ln x - (1/3) Ln y = Solution: 퐿푛 푥1/2 푦1/3 = 퐿푛 √푥 √푦 3 Writing logs as single logs can be helpful in solving many log equations. 1) Log2 (x + 1) + Log2 3 = 4 Solution: First combine the logs as a single log. Log2 3(x + 1) = 4 Now rewrite as an exponential equation. 3(x + 1) = 24 Now solve for x. 3x + 3 = 16 3x = 13 x = 13/3 Since this doesn't make the number inside the log zero or negative, the answer is acceptable.
  • 70. 70 2) Log (x + 3) + Log x = 1 Solution: Again, combine the logs as a single log. Log x(x + 3) = 1 Rewrite as an exponential. x(x + 3) = 10 Solve for x. x2 + 3x = 10 x2 + 3x - 10 = 0 (x + 5)(x - 2) = 0 x = -5 or x = 2 We have to throw out 5. Why? Because it makes (x + 3) negative and we can't take the log of a negative number. So the only answer is x = 2. 3) Ln (x - 4) + Ln x = Ln 21 Solution: Notice, this time we have a log on both sides. If we write the left side as a single log, we can use the rule that if the logs are equal, the quantity inside must be equal. Ln x(x - 4) = Ln 21 Since the logs are equal, what is inside must be equal. x(x - 4) = 21 Solve for x. x2 - 4x = 21 x2 - 4x - 21 = 0 (x - 7)(x + 3) = 0 x = 7 or x = -3 Again, we need to throw out one of the answers because it makes both quantities negative. Throw out -3 and keep 7. Thus, the answer is x = 7. Simplify each log . 1) ln e5 Solution: This is rule number 7. The answer is 5! 2) Log 10-3 Solution: This is again rule #7. The answer: -3 (This answers the question: what power do you raise 10 to get 10 to the third? 3) eln 7 Solution: This is rule #8. The answer is 7. 4) e2ln 5 Solution: We can use rule #8 as soon as we simplify the problem. Rewrite as: eln 25 = 25 The 25 came from 52. 5) 10Log 6 Solution: Rule #8 again. Answer: 6
  • 71. 71 6) 102 + log 5 Solution: We need to simplify before we can apply one of the rules. Rewrite as: (102)(10log 5) Adding exponents means you are multiplying the bases. = 100(5) Use rule #8 again. = 50 Change of Base Formula An exponential equation is an equation that contains a variable in the exponent. We solved problems of this type in a previous chapter by putting the problem into the same base. Unfortunately, it is not always possible to do this. Take for example, the equation 2x = 17. We cannot put this equation in the same base. So, how do we solve the problem? We use thechange of base formula!! We can change any base to a different base any time we want. The most used bases are obviously base 10 and base e because they are the only bases that appear on your calculator!! Change of base formula Logb x = Loga x/Loga b Pick a new base and the formula says it is equal to the log of the number in the new base divided by the log of the old base in the new base. Examples 1) Log2 37 = Solution: Change to base 10 and use your calculator. = Log 37/log 2 Now use your calculator and round to hundredths. = 5.21 This seems reasonable, as the log2 32 = 5 and log2 64 = 6. 2) Log7 99 = Solution: Change to either base 10 or base e. Both will give you the same answer. Try it both ways and see. = Log 99/Log 7 or Ln 99/ Ln 7 Use your calculator on both of the above and prove to yourself that you get the same answer. Both ways give you 2.36. Solving Exponential Equations using change of base Now, let's go back up and try the original equation: 2x = 17 To put these in the same base, take the log of both sides. Either in base 10 or base e. Hint. Use base e only if the problem contains e. Log 2x = Log 17 Using the log rules, we can write as: x Log 2 = Log 17 Now isolate for x and use your calculator. x = Log 17/Log 2 x = 4.09
  • 72. To check your answer, type in 24.09 and see what you get! The answer will come out slightly larger than 17 do to rounding. 72 Sample Problems 1) e3x = 23 Solution: Use natural log this time. Ln e3x = Ln 23 3x Ln e = Ln 23 3x = Ln 23 ( Ln e = 1) x = (Ln 23)/3 x = 1.05 2) How long does it take $100 to become $1000 if invested at 10% compounded quarterly? Solution: Ao = 100, A(t) = 1000, r = .1, n = 4 1000 = 100(1 + .1/4)4t 10 = 1.0254t Use the change of base formula Log 10 = Log 1.0254t 1 = 4t Log 1.025 (Log 10 = 1) 1/(4Log 1.025) = t t = 23.31 It will take 23.3 years to have $1000 from the $100 investment.
  • 73. ASSESSMENT 73 1) Simplify each: a) 4 . 2-4 = b) (4 . 2)-4 = c) (a-1 + b-1)-2 = 2) Simplify each: 3) Solve the equation: 93x = 81x + 1 4) The half life of a radioactive isotope is 7 days. If 5.6 kg are present now, how much will be present in t days? In 13 days? 5) A bacteria colony triples every 6 days. The population currently is 5000 bacteria. What will be the population in 21 days? 6) $1000 is invested at 8% compounded quarterly for 3 years. How much interest will you receive at the end of the time period? 7) You invest $3000 at 7% compounded continuously for 2 years. How much money will be in the account at the end of the time period? 8) Solve each log equation: a) Log x = 21 b) Log |x| = 15 c) Ln (x2 - 1) = 3 d) Log x = 1.6 (Use calculator and round to hundredths.) 9) Write each log as a single log: a) Log x + log y + 2Log z b) Ln x + Ln y - 3Ln z 10) Simplify each log:
  • 74. 74 a) Ln e5 = b) 102log4 = c) 101 + log 5 = 11) Solve the equation: Log (x + 2) + Log 5 = 4 12) Graph y = 3x and y = log3 x on the same axis. 13) Use a calculator to find Log7 58 and round to hundredths place. 14) solve the equation: 15x + 1 = 29 and round to hundredths place. 15) How long will it take to double $1000 if invested at 6% compounded monthly?
  • 75. Chapter VI System of Equations And Inequalities 75 .
  • 76. SYSTEMS OF LINEAR EQUATIONS IN TWO VARIABLES Many applications of mathematics lead to more than one equation in several variables. The resulting equations are called a system of equations. The solution set of a system of equations consists of all solutions that are common to the equation in the system. 2푥 + 푦 = 3 5푥 + 3푦 = 10 76 ax + by = c We proved that the graph of an equation of the form is a line and that all ordered pairs (x , y) satisfying the equation are coordinates of points in the line. A system of two linear equations in two variables x and y can be written as. {a1x + b1y = c1} {a2x + b2y = c2} Where a1,b1, c1, a2, b2,and c2 are real numbers. The left brace is used to indicate that the two equations form a system. If an ordered pair (x , y) is to satisfy the system of two linear equations, the corresponding points (x , y) must lie on the two lines that are the graph of the equations. ILLUSTRATION 1 A particular system of two linear equations is. { The solution set of each of the equations in the system is an infinite set of ordered pairs of real numbers, and the graphs of these sets are lines. Recall that to draw a sketch of a line we need to find two points on the line; usually we plot the points where the line intersects the coordinate axis. On the line 2x + y = 3 we have the points (3/2 , 0) and (0,3) while on the line 5x + 3y = 10 we have the points (2,0) and(0, 10/3). Shows on the same coordinate system sketches of two lines. It is apparent that two lines intersect at exactly one point. This point, (-1 , 5) can be verified by substituting into the equations as follows: 2(-1) + 5 = 3 5(-1) + 3(5) = 10 The only ordered pair that is common to the solution sets of the two equations is (-1,5). Hence the solution set of the system is {(-1,5)}. ILLUSTRATION 2 Consider the system 6푥 − 3푦 = 5 2푥 − 푦 = 4 {
  • 77. The lines having these equations appear to be parallel. It can easily to be proved that the lines are indeed parallel by writing each of the equation, we have 6x - 3y = 5 2x – y = 4 -3y = -6x + 5 -y = -2x + 4 y = 2x – 5/3 y = 2x – 4 2푥 + 푦 = 3 5푥 + 3푦 = 10 푦 = 3 − 2푥 5푥 + 3푦 = 10 77 Example 1 Use the substitution method to find the solution set of the system. Illustration 1: { Solution: We solve the first equation for y and get the equivalent system { We replace y in the second equation by its equal, 3 – 2 x, from the first equation. We then have the equivalent system { 푦 = 3 − 2푥 5푥 + 3(3 − 2푥 ) = 10 Simplifying the second equation, we have { 푦 = 3 − 2푥 −푥 + 9 = 10 Solving the second equation for x, we get { 푦 = 3 − 2푥 푥 = −1 Finally, we substitute the value of x from the second equation into the first equation and we have { 푦 = 5 푥 = −1 This system is equivalent to the given one. Hence the solution set is ( -1 , 5)
  • 78. 78 Example 2 Use the elimination method to find the solution set of the system of equations in Example 1. 2푥 + 푦 = 3 5푥 + 3푦 = 10 { Remember that our goal is to eliminate one of the variables. Observe that the coefficient of y is 1 in the first equation and 3 in the second equation. To obtain an equation not involving y, we therefore replace the second equation by the sum of the second equation and -3 times the first. We begin by multiplying the first equation by -3 and writing the equivalent system. { −6 − 3푦 = −9 5푥 + 3푦 = 10 Adding the equations given the following computations: −6−3푦 = −9 5푥 +3푦 =10 −푥 =1 With this equation and the first equations in the given system, we can write the following equivalent system { 2푥 + 푦 = 3 −푥 = 1 If we now multiply both sides of the second equation by -1, we have the equivalent system { 2푥 + 푦 = 3 푥 = −1 We next substitute -1 for x in the first equation to obtain { 2(−1) + 푦 = 3 푥 = −1 { 푦 = 5 푥 = −1 SYSTEMS OF LINEAR EQUATIONS IN THREE VARIABLES So far the linear (first degree) equations we have discussed have contained at most two variables. In this section we introduce systems of linear equations in three variables. Consider the equation 2푥 − 푦 + 4푧 = 10 For which the replacement set of each of the three variables x, y and z is the set R of real numbers. This equation is linear in the three variables. A solution of a linear equation in the three variables x, y and z is the ordered triple of real numbers (r,s,z) such that if x is replaced by r, y by s, and z by t, the resulting statement is true. The set of all solutions is the solution set of the equation
  • 79. 79 Illustration 1 For the equation 2푥 − 푦 + 4푧 = 10 The ordered triple pair (3,4,2) is a solution because 2(3) – 4 + 4 (2) = 10 Some other ordered triples that satisfy this equation are (-1, 8, 5) , (2, -6, 0), (1,0,2), (5,0,0) , (0,-6,1) , (8, 2 1) ,and (7,2 - ½). It appears that the solution set is infinite. The graph of an equation in three variables is a set of points represented by ordered triples of real numbers. Such points appear in a three dimensional coordinate system, which we do not discuss. You should, however, be aware that the graph of a linear equation in a three variables is a plane. Suppose that we have the following system of linear equations in the variables x, y and z. { 푎1푥 + 푏1푦 + 푐1푧 = 푑1 푎2푥 + 푏2푦 + 푐2푧 = 푑2 푎3푥 + 푏3푦 + 푐3푧 = 푑3 The solution set of this system is the intersection of the solution sets of the three equations. Because the graph of each equation is a plane, the solution set can be interpreted geometrically as the intersection of three planes. When this intersection consist and independent. Algebraic methods for finding the solution set of a system of three linear equations in three variables are analogous to those used to solve linear systems in two variables. The following examples shows the substitution method. Example Find the solution set of the system 푥 − 푦 − 4푧 = 3 2푥 − 3푦 + 2푧 = 0 2푥 − 푦 + 2푧 = 2 { Solution, we solve the first set of the system 푥 = 푦 + 4푧 + 3 2푥 − 3푦 + 2푧 = 0 2푥 − 푦 + 2푧 = 2 {
  • 80. We now substitute the value of x from the first equation into the other two equations , and we obtain the equivalent system 푥 = 푦 + 4푧 + 3 −푦 + 10푧 = −6 푦 + 10푧 = −4 푥 = 푦 + 4푧 + 3 푦 = 10푧 + 6 푥 = 푦 + 4푧 + 3 80 { 푥 = 푦 + 4푧 + 3 2(푦 + 4푧 + 3) − 3푦 + 2푧 = 0 2(푦 + 4푧 + 3) − 푦 + 2푧 = 2 { We next solve the second equation for y and get 푥 = 푦 + 4푧 + 3 푦 = 10푧 + 6 푦 + 10푧 = −4 Substituting the value of y from the second equation into the third gives the equivalent system. { 푥 = 푦 + 4푧 + 3 푦 = 10푧 + 6 (10푧 + 6 +) + 10 = −4 푥 = 푦 + 4푧 + 3 푦 = 10푧 + 6 20푧 = −10 { { 푧 = −1/2 Substituting the value of z from the third equation into the second equation, we obtain { 푦 = 1 푧 = −1/2 Substituting the values of y and z from the second and third equations into the first equation, we get 푥 = 2 푦 = 1 푧 = 1/2 {
  • 81. The latter system is equivalent to the given system. Hence the solution set of the given system is (2,1 ,1/2).The solution can be checked by substituting into each of the given equations. Doing this we have 81 2 − 1 + 2 = 3 4 − 3 − 1 = 0 4 − 1 − 1 = 2 { The equations of the given system are consistent and independent. Exercise 1.) { 4푥 + 3푦 + 푧 = 15 푥 − 푦 − 2푧 = 2 2푥 − 2푦 + 푧 = 4 2.) { 2푥 + 3푦 + 푧 = 8 5푥 + 2푦 + 3푧 = −13 푥 − 2푦 + 5푧 = 15 3.) { 푥 − 푦 + 3푧 = 2 2푥 + 2푦 − 푧 = 5 5푥 + 2푧 = 7 3푥 + 2푦 − 푧 = 4 3푥 + 푦 + 3푧 = −2 6푥 − 3푦 − 2푧 = −6 4.) { 5.) { 2푥 − 3푦 − 5푧 = 4 푥 + 7푦 + 6푧 = −7 7푥 + 2푦 − 9푧 = 6 3푥 − 2푦 + 4푧 = 4 7푥 − 5푦 − 푧 = 9 푥 + 9푦 − 9푧 = 1 6.) { 7.) { 3푥 − 5푦 + 2푧 = −2 2푥 + 3푧 = −3 4푦 − 3푧 = 8 푥 − 푦 = 2 3푦 + 푧 = 1 푥 − 2푧 = 7 8.) { 9.) { 3푥 − 2푦 = 1 푧 − 푦 = 5 푧 − 2푥 = 5 푥 − 푦 + 5푧 = 2 4푥 − 3푦 + 5푧 = 3 3푥 − 2푦 + 4푧 = 1 10.) { SYSTEMS INVOLVING QUADRATIC EQUATIONS In sections 6.1 and 6.2 our discussions of systems of equations was confined to linear systems. However, a number of applications lead to nonlinear systems as illustrated in exercises 25 through 36. The word problems in these exercises use concepts presented previously, but the resulting systems involve at least one quadratic equation. In this section we discussed methods of solving such systems of two equations in two variables. We consider first a system that contains a linear equation and quadratic equation. In this case the system can be solved for one variable in terms of the other, and the resulting expression can be substituted into the quadratic equation, as shown in the following example.
  • 82. 푦2 = 4(3 − 푦) 푥 = 3 − 푦 푦2 + 4푦 − 12 = 0 푦 = −6 푥 = 3 − 푦 82 Example 1 Find the solution set of the system. 푦2 = 4푥 푥 + 푦 = 3 { Solution We solve the second equation for x and obtain the equivalent system. 푦2 = 4푥 푥 = 3 − 푦 { Replacing x in the first equation by its equal from the second, we have the equivalent system { { 푥 = 3 − 푦 We now solve the first equation. (푦 − 2)(푦 + 6) = 0 푦 − 2 = 0 푦 + 6 = 0 y= 2 푦 = −6 Because the first equation of system (II) is equivalent to the equations 푦 = 2 and 푦 = −6, system (II) is equivalent to the systems 푦 = 2 푥 = 3 − 푦 { and { In each of the latter two systems we have substitute into the second equation the value of y from the first , and we have { 푦 = 2 푥 = 1 and { 푦 = −6 푥 = 9 These two systems are equivalent to system (I). Thus, the solution set of (I) is (1,2) , (9,- 6).
  • 83. SYSTEMS OF LINEAR INEQUALITIES AND INTRODUCTION TO LINEAR PROGRAMMING Systems of linear inequalities are important in economics, business, statistics, science, engineering, and other fields. With electronic computers performing most of the computation, large numbers of inequalities with many unknowns are usually involved. In this section we briefly discuss how to solve system of linear inequalities. We then give an introduction to linear programming, a related approach to decision making problems. 83 Statement of the form 퐴푥 + 퐵푦 + 퐶 > 0 퐴푥 + 퐵푦 + 퐶 < 0 퐴푥 + 퐵푦 + 퐶 ≥ 0 퐴푥 + 퐵푦 + 퐶 ≤ 0 Where A,B and C are constants, A and B are not both zero, are inequalities of first degree in two variables. By the graph of such an inequality, we mean the (x, y) in the rectangular Cartesian coordinate system for which (x, y) is an ordered pair satisfying the inequality. Every line in a plane divides the plane into two regions, one on each side of the line. Each of these regions is called a half plane. The graphs of inequalities of the forms. 퐴푥 + 퐵푦 + 퐶 > 0 and 퐴푥 + 퐵푦 + 퐶 < 0 Are half planes. We shall show this for the particular inequalities 2푥 − 푦 − 4 > 0 푎푛푑 퐴푥 + 퐵푦 + 퐶 < 0 Let L be the line having the equation 2푥 − 푦 − 4 = 0. If we solve this equation for y, we obtain 푦 = 2푥 − 4. If (x, y) is any point in the plane, exactly one of the following statements holds: 푦 = 2푥 − 4 푦 > 2푥 − 4 푦 < 2푥 − 4 Now, 푦 > 2푥 − 4 if and only if the point (x, y) is any point (x, 2x - 4) on line L; Furthermore, 푦 < 2푥 − 4 if and only if the point (x, y) is below the point (x, 2x - 4) on L; therefore the line L divides the plane into two regions. One region is the half plane above L, which is the graph of inequality 푦 > 2푥 − 4, and the other region is the half plane above L, which is the graph of the inequalities 푦 > 2푥 − 4, and the region is the half plane below L, which is the graph of inequality 푦 < 2푥 − 4. A similar discussion holds for any line L having an equation of the form 퐴푥 + 퐵푦 + 퐶 = 0 푤ℎ푒푟푒 퐵 ≠ 0.
  • 84. If B= 0, an equation of line L is 퐴푥 + 퐶 = 0, and L is a vertical line whose equation 푥 = 4. Then if (x, y) is any point in the plane, exactly one of the following statements is true: The point (x, y) is to the right of the point (4, y) if and only if x >4. Showing the graph of inequality x >4 as the half plane lying to the right of the line x = 4. Similarly, the graph of x < 4 if, and only if the point (x, y) is to the left of the point (4, y). The discussion can be extended to any line having an equation of the form Ax +퐶 = 0. By generalizing the above arguments to any line, we can prove this theorem. 84 THEOREM (I) the graph of y> 푚푥 + 푏 is the half plane lying above the line y= 푚푥 + 푏. (II) the graph of y< 푚푥 + 푏 is the half plane lying below the line y= 푚푥 + 푏. (III) the graph of (y< 푚푥 + 푏) x> 푎 is the half plane lying to the right of line x= 푎. (IV) The graph of x< 푎 is the half plane lying to the left of the line x= 푎. Example 1 Draw a sketch of the graph of the inequality 2푥 − 4푦 + 5 > 0 Solution The given inequality is equivalent to −4푦 > −2푥 − 5 푦 > 1 2 + 5/4 The graph of inequality is the half plane below the line having the equation 푦 = 1/2푥 + 5/4. A sketch of this graph is the shaded half plane. A closed half plain is a half plane together with the line bounding it is the graph of an inequality of the form. 퐴푥 + 퐵푦 + 퐶 = 0 표푟 퐴푥 + 퐵푦 + 퐶 ≤ 0
  • 85. 85 Illustration The inequality 4푥 + 5푦 − 20 ≥ 0 Is equivalent to 5푦 ≥ −4푥 + 20 푦 ≥ −4/5푥 + 4 Therefore the graph of this inequality is the closed half plane consisting of the line 푦 = −3/5푥 + 4 and the half plane above it. A sketch of the graph. Two intersecting lines divide the points of the plane into four regions. Each of these regions is the intersection of two half planes and is defined by a system of two linear inequalities.